You are on page 1of 304

MJDF Part 1 MCQ's

1 . What is the minimum platelet level with which you can do extraction?

Answer.

If the platelet level is less than 50 x 1 0 ^ 9 p e r / l , t h e n you


cannot proceed at all for surgery. Generally, any level between 50-100 is
considered to be low in which case though you can proceed but you will
have to be more careful while dealing with it e.g. you should suture the
socket, use proper packs and other precautionary measures.

If platelet count > 100, it is okay to proceed normally.

a. Regional LA blocks are given, if the platelets levels are above 30

b. Haemostasis of dentoalveolar surgery is adequate if platelet levels


are above 50x10^9/L.

c. Major surgery requires platelets level above 75x10^9/L.

2. Waste disposal ?

Answer.

Wedges, silicone syringe tips have to go on the sharps as they


can perforate the orange bag, Also the matrix band goes into sharps bin.

3.When performing operative treatment in patient's oral


c a v i t y , y o u p u t y o u r f i n g e r support on?

1- Tooth
2- Gingiva
3- Cheek
4-Chin
5- Lip

Answer.
Your finger rest is ideally on tooth.

4. EXPLAIN IN DETAIL WHY SHOULD WE GIVE 5 RESCUE


BREATHS FIRST IN CHILD FOR CPR?
Answer.

Because children have ASPHYXIAL ARREST so the most important


thing is OXYGEN

5. What is the dose of Amoxicillin oral suspension

Answer
125mg/5ml - kid dose

6. What is the year of when RIDDOR regulation came out?

Answer
1995 (Reporting of Injuries, Diseases and Dangerous Occurrences
Regulations 1995)

7. What is the normal level of Glucose on a BM stick

Answer
4-7mmol

8.What is the altered cast technique?

Answer. Special technique for mucosa and tooth-born partial denture


(Kennedy’s class1&2)?Mucocompressive/mucostatic impression.

9. A non synovial joint with dense connective tissue between two


bony surfaces

1 synchondrosis
2 syndesmosis
3 symphysis
4 suture

Answer.
Suture

Explanation:
Both Suture and Syndesmoses are connected by dense
connective tissue and is the answer for the question.
Most appropriate answer is Suture for this question.

Fibrous joints are connected by dense connective tissue, consisting


mainly of collagen. These joints are also called "fixed" or "immoveable"
joints, because they do not move. These joints have no joint cavity and
are connected via fibrous connective tissue. The skull bones are
connected by fibrous joints.

Types*

Sutures are found between bones of the skull. In fetal skulls the sutures
are wide to allow slightmovement during birth. They later become rigid
(synarthrodial).

* Syndesmoses

are found between long bones of the body, such as the radius and
ulna forearm
and the fibula and tibia in leg.Unlike other fibrous
j o i n t s , s y n d e s m o s e s a r e m o v e a b l e - amphiarthrodial), albeit
not to such degree as synovial joints.

* Gomphosis

is a joint between the root of a tooth and the sockets in the maxilla or
mandible.

Synchondroses and symphyses:

Are cartilaginous joints connected entirely by cartilage


(fibrocartilage or hyaline)

A Synovial joint, also known as a diarthrosis, is the most common and


most movable type of joint in the body of a mammal. As with most other
joints, synovial joints achieve movement at the point of contact of the
articulating bones.

Examples for Synovial joint.


The carpals of the wrist, acromioclavicular joint, Elbow (between the
humerus and the ulna). For additional info, please refer to:
http://www.teachpe.com/anatomy/joints.php

10. Ameloblastoma is the most aggressive ...........................


11. What is required to increase efficiency
a n d r e d u c e f a t i g u e o f u n i v e r s a l curettes?

12. Ideal angle required for sub gingival curettage from universal
curettes?

13. Suppuration is mainly the result of the combined action of four


factors which of the following is not one of them?

1 necrosis
2 presence of lymphocytes
3 collection of neutrophils
4 accumulation of tissue fluid
5 autolysis by proteolytic enzymes

Answer
: presence of lymphocytes

the main cells involved are neutrophils and macrophages, there’re


proteolytic enzymes and there is liquefactive necrosis and pus definitely
has tissue fluid.

14. What would over trituration of silver and mercury


c a u s e a n d w h a t w o u l d u n d e r trituration cause?

Answer:

Overtrituration causes: shrinkage on setting

undertrituration causes: porosities and voids

15. As a newly qualified dentist you are offered a contract in


which remuneration is on capitation basis.
What is the most important and well-recognized feature of this method of
remuneration?

A. Tends to encourage under-prescribing


B. Tends to encourage high technical quality of work
C. Tends to encourage high output of procedures
D. Tends to encourage over-prescribing
E. Tends to encourage low technical quality of work
Answer: B

16. A patient on examination was found to have swollen gingiva around


a crown that had been present for several years. The papillae were
particularly enlarged. What is the most important feature of a crown
that may be responsible for this?

A. Material of the Crown


B. The occlusion
C. Proximal Contour
D. Labial Contour
E. Surface finish

Answer: C

17. It is best to retain dental radiographs for how many years?

Answer:
Adults records shud be kept for: 11 years
for children: till they r 25 yrs old or 11 years, whichever is longer ......

18. Which is the most important factor to reduce dental irradiation?


(speed of film or collimation?

Answer:
Both: rectangular collimation, E or F speed films

19. After crown placement, how many months will you review it radio
graphically?

Answer:
Annually.

20. What causes Amalgam expansion? What makes Amalgam black


after sometime?

Amalgam Expansion:
1- Moisture contamination during mixing and condensation operation.
2- Moisture in the saliva is the potential contaminant for the amalgam.
3- Zinc containing amalgam, the presence of saliva on the amalgam
during condensation probably a possible source of expansion.
Amalgum Black:
1- corrosion.
2- Reaction of the phases.
3- Marginal leakage.

21. A 50 year old male patient has a Class III jaw


relationship with an anterior open bite. It is planned to restore
his lower right second molar, which has suffered tooth wear and fracture,
with an i n d i r e c t
restoration. This tooth has approximately 2
m m o f c o r o n a l h e i g h t . What would be the most suitable
approach to restore this tooth?

A. Provide an adhesively retained gold onlay


B. Provide a conventional full crown
C. Increase the vertical dimension and provide a full crown
D. Surgically crown lengthen and provide gold crown
E. Provide an adhesively retained ceramic onlay

Answer: A ( refer indications for Onlay and inlay in Master dentistry 2)

22. A patient presents with a history of a post-crown having fallen out.


The post-crown was originally placed fifteen years ago and had been
successful up until four months ago since when it has come out and
been recemented four times. At recementation there was no evidence of
any caries. The patient had been a regular attender and not needed any
restorative treatment for the last eight years. Which of the following is
the most likely cause for the failure of this crown?
A. The post was to narrow
B. The post was to short.
C. The root canal treatment was failing.
D. A vertical root fracture was present.
E. There were excessive occlusal loads on the tooth.

Answer: D
23. In dental epidemiology, indices are used to measure the oral health
of a population. The DMF index is commonly used to measure
the prevalence and severity of dental caries
24. You want to evaluate the effectiveness of tetracycline as an adjunct
to scaling & root planning for the treatment of chronic periodontitis. What
is the primary study design most appreciate for addressing this topic?
A. Cohort
B. Non randomised controlled trial
C. Randomised controlled trial
D. Control case study
E. Case series
Answer: C

24. Which LA is neurotoxic at 4%

Answer: Articane

25. The management of patients taking corticosteroid for long time with
dental infection , abscess or high temperature , do we need double the
dose of antibiotic or double the dose of cortisone ?

Answer:

Adrenal insuffiency may follow long term administration of oral


corticosteroids and
can persist for years after stopping therapy. A pt with adrenal
insufficiency can become hypotensive when under physiological stress.
Acute adrenal
insuficiency can be prevented by administration of an increased dose of
corticosteroids prior to treatment.

26. Do we need to change the antibiotic dose for patient with rheumatoid
arthritis taking 15 mg prednisolone for 6 months or 3 months had dental
abscess?

Answer: No:

Antibiotic prophylaxis: amoxicillin, cefalexin or cepharidine 2g


orally 30-60 min before procedure. If unable to take oral meds give
ampicillin 2g IM/IV 3O-60 min before procedureif penicillin allergic give
clindamycin 600 mg orally or IV 30-60 min before procedure.

27. What is the main cause of patient collapse with congestive


heart failure in the dental clinic?
Answer:
Any surgical procedures may cause undue stress resulting in
cardiac dysfunction (workload increase of the heart) which
surpasses the functional ability of the heart followed by

potential acute pulmonary edema. Pts with this condition present with
extreme dyspnea, hyperventilation, cough, hemoptysis, great difficulty in
breathing, murmurs due to cardiac asthma and cyanosis. The pt prefers
the sitting position, is anxious and might feel he/she is choking and if
death is imminent.
The preventives measures are:
1. Written consent from the pt's cardiologist and consultation is desirable
2. Oral premedication: 5-10 mg diazepam 1 hour before the surg
procedure
3. Small amounts of vasoconstrictors in LA with particular importance of
aspiration
4. Short appointments and as painless as possible

28. Why the patient with heart failure bleed easily and how to manage
such bleeding?

Answer:
Patients with heart failure are on usually (but not exclusively) on blood
thinners (Aspirin or Warfarin) that help prevent clot formation in the
vessels. Blood thinners may cause to bleed easily or bruise easily.
Consult the cardiologist & check INR

29. Which immunoglobin exists as both a monomer and diamer?

Answer:
IgA

30. Appointment for asthmatic patient should be given in?

A morning
B Afternoon
C Evening
D Anytime

Answer
: because the highest risk for asthma attack is in the morning and at
night...so the better option would be in the
afternoon

30. According to the water regulations, all dental equipment must be


protected by back-syphonageand between the water ingress and
drainage points there should be a:
a. “Type A” gap of 20mm
b. “Type B” gap of 30mm
c. “Type C” gap of 40mm
d. “Type D” gap of 50mm.

Answer:
B
31. Fire certificates are only required for buildings with 20 people
working in them or if more than ten people work on floors other
than ground floors (same applies to dental practices). Fire
risk assessment should be carried every
a. 3 months
b. 6 months
c. 9 months
d. one year
Answer:D

32. Discharge in vesicullo bullous lesion - serous or suppurative?

Answer : Serous

33. Rushton bodies are commonly seen in which of the following:


1.ultraviolet rays
2.ultrasonic vibrations
3.lichen planus
4.lupus erythematosus
5. Radicular cyst

Rushton bodies:
Translucent or pink staining lamellar bodies are formed by cyst lining
epithelium and indicate the odontogenic origin of a cyst.
Answer:
Radicular cyst

34. If a patient has severe gag reflex which kind of sedation will
you use?

Answer:
Inhalation IM, IV
35. Sedation for pregnant women?

Answer:
Best to avoid sedation

Contraindications and special caution


Benzodiazepines require special precaution if used in the elderly, during
pregnancy, in children, inalcohol or drug-
dependent individuals or individuals withcomorbid psychiatric di
sorders. Additional caution is required in critically ill patients, as
accumulation of midazolam and its activemetabolites may occur.
[43]
Kidney or liver impairments may slow down the eliminati
on of m i d a z o l a m l e a d i n g t o p r o l o n g e d a n d e n h a n c
ed effects.
[5][44]
Contraindications includehypersensitivity, acute narrow angle
glaucoma, shock, hypotension or head injury. Most are
relativecontraindications.
Pregnancy
Midazolam, when taken during thethird trimester of pregnancy,
may cause severe risk to
theneonate, including benzodiazepine withdrawal syndromewith pos
sible symptoms includinghypotonia, apnoeicspells, cyanosis, and
impairedmetabolic responses to cold stress. Symptoms of hypotonia and
the neonatal benzodiazepine withdrawal syndrome have been reported
to persist fromhours to months after birth.
[45]
Other neonatal withdrawal symptoms include
hyperexcitability,tremor and gastrointestinal upset (diarrhea or
vomiting).Breast feeding b y m o t h e r s u s i n g midazolam is not
recommended.
[46]
Neonates
Midazolam is sometimes used in neonatal intensive care unit care.
When used, additional caution isrequired inneonates; midazolam
should not be used for longer than 72 hours due to risks of
tachyphylaxis
, and the possibility of development of a benzodiazepine
withdrawal syndrome, aswell as neurological complications.Bolus
injections should be avoided due to the increased risk of cardiovascular
depression, as well as neurological complications.
[47]
Elderly
Additional caution is required in the elderly, as they are more
sensitive to the pharmacologicaleffects of benzodiazepines and
also metabolise them more slowly, and are more prone to
adverseeffects, including drowsiness, amnesia (especially anterograde
amnesia),ataxia, hangover effects,confusion and falls.
[13]

36. Which immunoglobulin associated to MALT (mucosa associated


lymphatic tissue?)

Answer: Predominantly IgA (IgA & IgM)

Mucosa-associated lymphoid tissue (MALT) is scattered along mucosal


linings, measuring roughly 400 m2.

It is the most extensive component of human lymphoid tissue.


These surfaces protect the body from an enormous quantity and
variety of antigens. The tonsils, Peyer patches within the small intestine,
and the vermiform appendix are examples of mucosa-associated
lymphoid tissue (MALT).

Mucosa-associated lymphoid tissue (MALT) includes gut-


associated lymphoid tissue (GALT), bronchial/tracheal-
associated lymphoid tissue (BALT), nose-
associated lymphoid tissue (NALT),and vulvovaginal-
associated lymphoid tissue (VALT). Additional mucosa-
associated lymphoidtissue (MALT) exists within the accessory organs
of the digestive tract,
predominantly the parotid gland.
The germinal center is key to the generation of a normal immune
response. The location of mucosa-associated lymphoid tissue (MALT) is
key to its function.
Stimulation of B lymphocytes leads tothe production of
immunoglobulin A (IgA) and IgM within the Peyer patches,
preventingadherence of bacteria and viruses to the epithelium,
thus blocking entry to the subepitheliallayers of the intestine.
The direct secretion of secretory IgA onto
mucosal epithelia represents the major effector mechanism of
mucosa-associated lymphoid tissue (MALT).

Major accumulations of lymphoid tissue are found in the lamina


propria of the intestine. M cells in the intestinal epithelium
overlyingPeyer patches allow transport of antigens to the lymphoid
tissue beneath it.The complex interplay among antigens, cells, and
cytokines results in a very efficient immuneresponse. The
efficiency of mucosa-associated lymphoid tissue (MALT) also
depends on theadequate function of IgA. Individuals with
selective IgA deficiency are prone to infections alongmucosal
surfaces in the respiratory, gastrointestinal, and genitourinary tracts.
Adequate function of IgA depends on the production and acquisition of a
joining (J) chain. This glycoprotein is
produced by plasma cells and is important in the formation of IgA dimers
and IgM pentamers. It has beenshown that in children who have
recurrent tonsillitis, B lymphocytes in tonsillar crypts do
not produce the J chain. The J chain is key in permitting secretory
IgA and IgM to function as the first line of defense in mucosal epithelium.

37. What will be the answer?

Stephan's curve represents


a) change in pH of saliva with time
b) change in pH of plaque with time
c) change in pH of saliva with sugar intake.

Answer: B

Explanation:
Beneficial effects of saliva in reducing plaque pH and therefore
caries
Dr Helen Whelton, Director Oral Health Services Research Centre,
University Dental School andHospital, Wilton. Cork, Ireland.
Email: H.Whelton@ucc.ie

Dental caries is an infectious, communicable disease resulting in


demineralisation and destruction of tooth structure by acid-forming
bacteria found in dental plaque, an intraoral biofilm, in
the presence of sugar. Far from being static, dental enamel is in a
constant state of change in that it undergoes cycles of
demineralisation and remineralisation. Acid produced by oral
bacteria in the
glycolysis of carbohydrates is the driving force towards
enamel demineralisation and caries progression. Saliva on
the other hand is the driver of repair and remineralisation.
In the absence of the protective properties of saliva, acid
dissolution of the teeth would progress unchecked in
susceptible individuals with cariogenic bacteria and unfavourable
diets. The fundamental aims of strategies for caries prevention are to
reduce demineralisation and/or to increase remineralisation.

The caries process is illustrated in Figure


1. Interaction between three factors over time is neededfor caries
to develop; a susceptible host i.e. a tooth surface, microflora with
cariogenic potential i.e. plaque, and fermentable carbohydrate i.e. sugar
The presence of saliva is vital to the maintenance of healthy
oral tissues. Its importance
in preventing caries was first illustrated by feeding a cariogenic diet to
desalivated rats which developed over four times as much caries as rats
with intact salivary glands who were fed the same diet. In humans,
although there is a lot of variation in saliva secretion rates,
the average is 0.3ml/minute for unstimulated and 1-2ml
per minute for stimulated saliva. The rampant caries seen in some
xerostomia individuals (<0.1ml saliva produced per minute,
unstimulated salivary flow rate) is indicative of the devastation of
the dentition that occurs in the absence of saliva. In healthy subjects
the teeth are constantly bathed in 0.5ml of unstimulated saliva which
coats them with a film approximately 10µm thick and which moves
as the muscles of the mouth are used. The impact of saliva in
neutralising plaque pH was illustrated by Stephan
2
who measured the changes in plaque acidity following sugar intake. He
used indwelling antimony microelectrodes to monitor changes in plaque
pH in situ following a sucrose rinse. The characteristics of the Stephan
Curve are illustrated in Figure 2.

Figure 2

Changes in plaque pH over time following a sucrose rinse

Characteristically, the Stephan Curve shows a rapid drop in plaque pH


which is followed by a slow rise until resting pH is attained. The critical
pH below which demineralisation of enamel generally takes place is 5.5.
Thus the shaded part of the curve indicates the period of
demineralisation. The time taken for these changes to occur
varies between individuals and also varies according to the nature of the
challenge. The drop in pH usually takes no more than five minutes
whereas the recovery for the resting pH can take between fifteen
and forty minutes depending on the acid neutralising properties of an
individual’s saliva. The initial sharp drop depends upon the speed with
which plaque microbes are able to metabolise sugar. Large molecules
like starch for example, diffuse into the plaque more slowly and take
longer to be broken down resulting in a less steep drop in pH. The rise in
pH is dependent upon diffusion of acid by-products out of the plaque
and, secondly salivary bicarbonate diffusing into the plaque buffering the
acid by-products. One of the most important factors governing the
overall shape of a Stephan Curve, but particularly the pH
recovery, is saliva flow rate.
Saliva exerts two effects
.
First,
it dilutes and carries away metabolites diffusing out of the plaque.

Second it supplies bicarbonate ions


which diffuse into plaque and neutralize the by-products of fermentation
(organic acids)
in situ
.The bicarbonate-
mediated acid neutralisation effect is enhanced by the increa
se in salivary bicarbonate associated with increased saliva flow which
coincides with eating or chewing e.g.chewing gum.
Source:
http://betteroralhealth.info/orbit_complete/professional-
area/resources/cpd/saliva- benefits/beneficial-effects-of-saliva-in-
reducing-plaque-ph-and-therefore-caries/index.htm38. Cause of early
child hood caries? (ECC)
Answer: Fruit based drink
39. Reducing cariogenicity1. sucrose, glucose, fructose, maltose
(honey)2. Galactose, lactose3. Complex carbohydrate (e.g. starch in
rice, bread, potatoes)
Foods that stimulate salivary flow and can speed the return of plaque PH
to normal E.g cheese, sugar free gum, salted peanuts.

40. Which antibiotic we can give to patient who is on warfarin? (not


allergic to penicillin)

Answer: Penicillin
41. Which antibiotic we can give to patient who is on warfarin? and if he
is also allergic to penicillin?

Answer
: Clindamycin
Avoid Metronidazole and Erythromycin as they potentiate the action of
warfarin (metro) or induce unpredictable effects (erythromycin). Note:
Paracetamol is safer than Aspirin and other NSAIDS which should be
also avoided in such patients.

42. Drug-induced lichenoid reaction


Gold salts, beta blockers, antimalarials, ACE inhibitors, NSAID’s thiazide
diuretics, furosemide,spironolactone, and penicillamine. As well as
systemic side effects of hypoglycaemic drugs such asMetformin.

43. Which drug causes angioedema?

i)Which drug causes lichenoid reaction


ii)Which drug causes dry cough?

Answer:
Phenytoin - Gingival hyperplasia
Ramipril (ACE inhibitors) – dry cough and angioedema – is type 1
hypersensitivity
Penicillin – anaphylaxis (type I hypersensitivity)
Furosemide – lichenoid reaction

Top 10 drugs/drug classes associated with angioedema.

ACE-inhibitors
bupropion
vaccines
selective serotonin reuptake inhibitors (SSRIs)
COX-II inhibitors
angiotensin II antagonists
other antidepressants
non-steroidal anti-inflammatory drugs (NSAIDs)
Statins
PPI's

Non-steroidal anti-
inflammatory drugs (NSAIDs), another major cause of the condition, pro
voke angioedema in 0.1–0.3% of patients. Facial angioedema is the
most frequent presentation. Both COX-I and COX-II inhibitors can
cause angioedema.

Other drugs
that can cause angioedema include aspirin, amoxicillin, and
proton-pump inhibitors(PPIs).
44. Which clotting factor is related to disseminated intravascular
coagulopathy?

Answer:

Factor VII, XII to XI to IX

TF binds with coagulation factors that then triggers the extrinsic pathway
(via Factor VII) which subsequently triggers the intrinsic pathway (XII to
XI to IX) of coagulation.
In DIC, the processes of coagulation and fibrinolysis are
dysregulated, and the result is widespread clotting with resultant
bleeding.

Under homeostatic conditions, the body is maintained in a finely tuned


balance of coagulation and fibrinolysis. The activation of the coagulation
cascade yields thrombin that converts fibrinogen to fibrin; the stable
fibrin clot being the final product of hemostasis. The fibrinolytic
system then functions to break down fibrinogen and fibrin.
Activation of the fibrinolytic system
generates plasmin (in the presence of thrombin), which is responsible fo
r the lysis of fibrin clots. The breakdown of fibrinogen and fibrin results in
polypeptides called fibrin degradation products(FDPs) or fibrin split
products (FSPs). In a state of homeostasis, the presence of plasmin is
critical,as it is the central proteolytic enzyme of coagulation and is
also necessary for the breakdown of clots, or fibrinolysis.
In DIC, the processes of coagulation and fibrinolysis are dysregula
ted, and the result iswidespread clotting with resultant bleeding.
Regardless of the triggering event of DIC, once initiated, the
pathophysiology of DIC is similar in all conditions. One critical mediator
of DIC is the release of a transmembrane glycoprotein called tissue
factor (TF). TF is present on the surface of many cell types
(including endothelial cells, macrophages, and monocytes) and is
not normally in contact with the general circulation, but is exposed
to the circulation after vascular damage. For example, TF is
released in response to exposure to cytokines (particularly
interleukin 1), tumor necrosis factor , a n d endotoxin.
[5]

This plays a major role in the development of DIC in septic


conditions.
TF is also abundant in tissues of the lungs, brain, and placenta.
This helps to explain why DIC readily develops in patients with
extensive trauma. Upon activation,
TF binds with coagulation factors that then triggers the extrinsic
pathway (via Factor VII) which subsequently triggers the intrinsic
pathway (XII to XI to IX) of coagulation.
Causes
DIC can occur in the following conditions;

Cancers of lung, pancreas, prostateandstomach, as well asacute
myeloid leukemia (particularlyAPML)

Obstetric:abruptio placentae,pre-eclampsia,amniotic fluid embolism

Massive tissue injury: Trauma, burns, extensive surgery

Infections:Gram-negativesepsis, Neisseria meningitidis,Streptococcus
pneumoniae, malaria,histoplasmosis,aspergillosis,Rocky mountain
spotted fever

Miscellaneous:Liver disease,snake bite, gianthemangioma,shock ,heat
stroke,vasculitis,aortic aneurysm,Serotonin syndrome
[9]

Viral:ArenavirusescausingArgentine hemorrhagic fever or Bolivian
Hemorrhagic Fever
Diagnosis
Diagnosis is usually suggested by following conditions:
[8]

Severe cases with hemorrhage:
The PT and APTT are usually very prolonged and thefibrinogen level
markedly reduced. High levels of fibrin degradation products, includingD-
dimer , are found owing to the intense fibrinolytic activity stimulated by
the presence of fibrin in the circulation. There is severe
thrombocytopenia. The blood film may showfragmented red blood cells
(schistocytes).

• Mild cases without bleeding:


There is increased synthesis of coagulation factors and platelets. PT,
APTT, and platelet counts are normal. fibrin degradation products are
raised. Definitive diagnosis depends on the result of:
- Thrombocytopenia
- Prolonged PT/APTT
- Low Fibrinogen
- Increased FDP

45. Which one is the most benign tumor

A. Kaposi sarcoma
B. Adenolymphoma
C. Burkitt's Lymphoma

Answer: Adenolymphoma

A benign glandular tumor usually arising in the parotid gland


and composed of two rows
of e o s i n o p h i l i c e p i t h e l i a l c e l l s w i t h a l y m p h o i d s t r o m a . A
l s o c a l l e d papillary cystadenoma lymphomatosum,Warthin's tumor

adenolymphoma 10% occur bilaterally.

a) Pleomorphic adenoma - 75% of parotid tumors, its mixed tumour (


fibrous, myxoid, cartilaginous )***

Common intraoral site


– junction of hard and soft palate usually feels rubbery and lobulated
on palpation.

b) mucoepidermoid carcinoma
- solid and more aggressive, can be invasive and occasionally
metastsise. 3-9%****

c) adenocystic carcinoma - cribriform or Swiss cheese pattern


, infiltrate along nerve sheaths. Cause multiple cranial nerve lesions –
especially lingual, facial or hypoglossal ***

d) Acinic cell carcinoma 1%

46. A curette may be inserted to the level of the attached gingiva with
minimal trauma to the tissues because of

A. Has a round base


B. Is easy to sharpen
C. Has rounded cutting edges
D. Provides good tactile sensitivity
E. Has two cutting edges

Answer:C

47. What type of hypersensitivity reaction causes amalgam restoration?

Answer:
Type IV

48. Kiwi causes anaphylaxis & urticaria,

Answer:
type I hypersensitivity.

48. As far as localised alveolar osteitis is concerned; which one of the


following is true

A. The incidence in the mandible and maxilla is similar


B. The prophylactic prescription of antibiotics prior to extraction reduces
the incidence.
C. Excessive fibrinolysis is the likely aetiology
D. Purulent exudate must be seen for a diagnosis and irrigation is
mandatory
E. Zinc oxide eugenol and alvogyl dressing promote a rapid bone growth

Answer: C

Explanation:
loss of cot was traditionally ascribed due to bacterial proteolytic
enzymes. However, it appears frequently due to excessive local
fibrinolytic activity. The alveolar bone and other oral tissues have a high
content
of fibrinolysin activators (plasmin) which are released when the bone is
traumatized.** Oestrogen component of oral contraceptives enhances the
serum fibrinolytic activity.

Alveolar osteotitis: Affects females, few days after extraction may be


delayed for week or more. Pain is deep seated, severe and aching or
throbbing.

Etiology:
• In healthy person, it only affects lower molar region where the bone is
more dense and lessvascular than elsewhere.

• Its an expected complication of extractions in Paget’s disease and after


radiotherapy where endarteritis causes ischemia of the bone.

•LA

49. How do you prepare floor of pulp chamber in molars

A. Swab and dry with cotton wool and excavate


B. Use round bur to flatten the floor
C. Under cut walls
D. Use flat end fissure bur to make it leveled

Answer: A

50. Which immunoglobin is the first to be found when the child is born?

Answer: IgM ( first to be produced by the neonate and produced


in primary immune response, 5-10% inserum). IgG crosses placenta and
protects fetus, produced in secondary immune response. Most abundant
immunoglobulin 80% in serum. IgA major secretory immunoglobulin (10-
15%)

51. Distance of maxillary sinus from premolars and molars

Answer:

Canine 6.9 mm
1st PM – 3.8
2nd PM – 1.9
1st M – 1.8
2nd M – 1.7
3rd M – 2.8

The bony lamella separating the maxillary sinus and the teeth decreases
in thickness from the upper canine (6.9 mm) to the 2nd molar (1.7 mm)
and further increases to the 3rd molar (2.8 mm).
52. Which of the following anomalies occurs during the initiation and
proliferation stages of tooth development

A. Amelogenesis imperfecta - histodifferentiation


B. Dentinogenesis imperfecta- histodifferentiation
C. Enamel hypoplasia – apposition stage
D. Oligodontia – initiation stage ( absence of single or multiple teeth)
E. AnkylosisHypocalcified/hypomineralised – normal organic matrix but
defective mineralization.

53. How to disinfect a bridge before sending to lab?

54. Best treatment planning- to replace missing lateral incisor

55. 15yr old boy


26 year old with missing lateral

with good periodontium, bone levels, good oral hygiene, absence of


Dental Caries ?

Answer
15 yrs- Resin bonded bridge
26years- Implant

56.Treatment for peg shaped laterals? With Hybrid composite or


laminate veneer the best option?

57.Type of crown placed after Nayyar core tec

Answer: Amalgam

58.Cement used for Temporary cementaion? Is it Zincpolycarboxylate


cement or Zinc oxide eugnol or both?

Answer:
Zinc oxide eugnol ( temp bond)

59. How many mg of lidocaine in 3 cartridges of 2.2?


please explain

Answer:
A. 2.2 ml of cartridge has 44mg of lidocaine.132mg of LA in 3 cartridges.
The max dose of L.A. is 4.4mg/kg body weight.eg. weight of a young
male 70 kg, hence max dose of LA will be 4.4 x 70 = 308mg.2%
Lignocaine comes to 20mg Lignocaine per ml.1 cartridge = 1.8ml = 1.8 x
20 = 36mg of lignocaine in one cartridge. Hence now 308 divided by 36
= 8.55which means about 8 cartridges of L.A. are safe in this patient (wt.
70kg)

60. What is the most toxic LA?

Answer:

Cardiotoxic- bupivacaine also ( long.acting)

61. LA for a patient with congenital heart disease in a dental emergency

Answer:
for cardiac arrhythmias- Lignocaine causes methhaemoglobinaemia -
Answer is Prilocaine

62 cells in acute and chronic infection

Answer:
Acute- PMN
chronic- Lympocytes, plasma cells, macrophages

63. Cells in granulomatous infections

Answer
.Granulomatous infection- epitheloid cells
64. What is the concentration of lignocaine in a topical anesthetic?

Answer:

Intra oral 4% is available as an ointment or gel. Spray formulations –


10% lidocaine is available for introral use. Skin: lidocaine 2.5%, prilocine
2.5% or 4% tetracaine. Lidocaine is an effective LA and consequently
the most commonly used in dentistry in the UK. It is available in dental
cartridges as a plain 2% solution or with adrenaline (epinephrine)
added in a concentration of 1:80 000.

Prilocaine
Prilocaine is available as a 4% plain solution or as a 3% solution with
0.02 lU/ml felypressin. The later is the usual alternative to lidocaine with
adrenaline (epinephrine) in the UK. If a vasoconstrictor must be avoided,
then plain 4% prilocaine is more effective than plain 2% lidocaine.
2) What is the depth of action of a topical anesthetic?
3) Which is the most toxic anesthetic agent?
Most toxic – Bupivacine
Less toxic – prilocine compared to lidocaine

65. Bur used to remove enamel?


2. Bur used to remove dentine?
3. Treatment options for root canal treated upper molar wt buccal root
canal failure and periapical infection?
4. Percentage of children getting cavities after application of sealants?
5. Best restoration material for class 5 in sjogrens? - GIC

6. Cavity in post teeth in parkinsonism patient,restorative material of


choice? -
Patients with Parkinson's disease are usually given anti-cholinergics,
which causes dry mouth, hence increased risk of caries.. GIC

66. What antibiotics and pain reliever would you prescribe for a patient
who has prosthetic heart valve and is on Warfarin anticoagulant
after having a surgical extraction of an infected tooth?
Ans: Penicillin and Paracetamol.

How should postoperative pain control be managed?

Patients should follow the advice of their anticoagulant clinic with regard
to the choice of analgesia for short-term mild to moderate pain.
Generally paracetamol is considered the safest simple analgesic
for patients taking warfarin and it may be taken in normal doses if
pain control is needed and no contraindication exists. Patients
should be advised not to take Aspirin,
aspirincontaining compound analgesic preparations or non-
steroidal anti-inflammatory drugs(NSAIDs) e.g. ibuprofen, which
are considered less safe than Paracetamol in patients taking
warfarin.
If analgesia is to be prescribed additional options include;

• Dihydrocodeine – an opioid analgesic with similar


analgesic efficacy to codeine that
can be prescribed on an NHS prescription. Its use should be considered
second line and only when other drugs are unsuitable. It is effective
for mild to moderate pain but has no anti-inflammatory activity and is
of limited value in pain of dental origin.

Are there any drug interactions that are relevant to this patient
group undergoing dental surgical procedures?

Amoxicillin - There are anecdotal reports that amoxicillin interacts with


warfarin causing increased INR and/or bleeding but documented
cases of an interaction are relatively rare considering how
frequently the drug is used; the broad picture is that no clinically
relevant interaction
normallyoccurs with amoxicillin and most penicillin’s. A single
3 gram dose given for endocarditis prophylaxis has not
been shown to produce a clinically relevant interaction. Prophylactic
antibiotics do not appear to affect the bleeding risk post operatively.
Patients taking warfarin requiring a course of amoxicillin should be
advised to be vigilant for any signs of increased bleeding and
concurrent use should be monitored so that the very occasional and
unpredictable cases INR increase or decrease can be identified.

Clindamycin - Clindamycin does not


interact with warfarin when given as a single
dose for endocarditis prophylaxis. Prophylactic antibiotics do
not appear to affect the bleeding risk postoperatively.
Metronidazole

CAUTION:
Metronidazole interacts with warfarin and should be avoided
wherever possible. If it cannot be avoided the warfarin dose may need
to be reduced by a third to a half by the
GP or anticoagulant clinic. The patient must seek advice
f r o m t h e p e r s o n m a n a g i n g t h e i r anticoagulation before taking
Metronidazole.
Erythromycin -
Erythromycin interacts with warfarin unpredictably and only
affects certainindividuals. Most are unlikely to develop a
clinically important interaction. Patients should be advised to be
vigilant for any signs of increased bleeding, concurrent use should
be monitored especially in the elderly. Paracetamol - The anticoagulant
effect of warfarin is normally not affected, or only increased by a small
amount, by occasional doses of paracetamol (no more than
about 2.5 to 3 g weekly).Paracetamol is considered to be safer than
aspirin as an analgesic in patients taking warfarin and is the analgesic
advised by anticoagulant clinics and the patient held ‘Oral
Anticoagulant
Therapy’ booklet. The anticoagulant effect of warfarin may be enhanced
by prolonged regular use of paracetamol.

Aspirin - AVOID use as an analgesic and anti-inflammatory agent.


Concurrent aspirin increases the likelihood of bleeding by 3-5
times, increases the bleeding time and may damage the
stomach lining. The interaction is well documented and clinically
important.

Non-Steroidal Anti-Inflammatory Drugs (NSAIDs)e.g. ibuprofen,


diclofenac. – AVOID:
Care should be taken when using NSAIDs because, to a
greater or lesser extent, they irritate the stomach lining, whic
h can result in gastrointestinal bleeding, which will be more severe
in anticoagulated patients. Although no interaction usuallyoccurs with
normal doses of ibuprofen and probably diclofenac, isolated cases of
raised INRs have been described. Some NSAIDs have effects on
platelet activity, which can affect bleeding
times. http://www.dundee.ac.uk/tuith/Static/info/warfarin.pdf

67. What do we do with used files? are they disp


o s e d o f i n a s h a r p s c o n t a i n e r o r are they sterilized ?

Answer:
According to new guidelines you reuse it for the same
p a t i e n t o n c e o n l y ( f r o m start to finish of rct) and have to dispose in
sharps bin.

68. Which tooth if extracted can spread infection to cavernous sinus?

Answer:
Mid facial infection spread infections to cavernous sinus

69. Patient, a known diabetic collapses in your dental chair, what would
is your first line of treatment?

Answer:

If patient is conscious give glucose orally or if unconscious give 1mg


glucagon IM.70. Which measurement is taken with a single record block
in the mouth??

Answer:
Resting Vertical Dimension: taken with only the lower denture.
Willis gauge used for measurement.

71. Uses of face bow?

Answer:
a device used in dentistry to record the positional relations of the
maxillary arch to the temporomandibular joints and to orient dental
casts in this same relationship to the opening axis of the articulator.

Or Used to transfer the relationship of Maxilla to the intercondylar axis of


the patient to the Articulator and the hinge axis of articulator.

72. Cantilever Bridge


Answer:
Suitable for Anterior teeth only

A Cantilever Bridge is a fixed bridge that is attached to adjacent


teeth on one end only. They are u s e d i n c a s e s w h e r e a
bridge can only be anchored to a tooth on one side of
t h e g a p . T h u s , Cantilever Bridges are best suited to areas of
your mouth that are less prone to stress, such as the front teeth.
This dental procedure involves anchoring the false tooth to one
side over one or more natural and adjacent teeth. As a result,
they are best suited and ideal for situations where a traditional
dental bridge or a dental implant isn’t an option.
73. Role of NaCl in saliva?
Answer: Preserve salt taste
2: role of Hco3 in saliva?
Answer: Acid-base balance
3: what makes saliva isotonic/hypotonic?
Answer: reduced concentration of HCo3- makes saliva hypotonic and
has little buffering capacity. Isotonic: Rich Na+ and HCO3- makes saliva
isotonic produced by acinar cells (Primary saliva)Final Saliva is less
hypotonic and with greater acid buffering capacity
74. Saliva Normal resting saliva flow:
0.1-0.6ml/min
Less than 0.1 cause xerostomia
75. A patient on examination was found to have swollen gingiva around
a crown that had been present for several years. The papillae were
particularly enlarged. What is the most important feature of a crown
that may be responsible for this?
A. Material of the Crown
B. The occlusion
C. Proximal Contour
D. Labial Contour
E. Surface finish
Answer: C
Explanation: The keyword in the question is enlargement of the
papilla. This enlargement is the result of lack of space (under
preparation) for the crown's material (MCC or GSC or Full porcelain)

and resulted in over contoured crown which in turn irritated


the papilla and invaded its spacecausing it to hypertrophy.
Because the papilla are anatomically located proximal (mesial or distal)
to teeth, the best answer would then be PROXIMAL CONTOUR
Hypertrophy: increase in size of cells

Hyperplasia: increase in number of cells (neoplasm=cancer)

76. Dental fee exemption: You do not have to pay for NHS dental
treatment if, when the treatment starts, you are:-

Under 18
-Under 19 and receiving full-time education
-Pregnant or a mother who has had a baby in the previous 12 months
-Staying in an NHS hospital and your treatment is carried out by the
hospital dentist.
-An NHS hospital dental service outpatient (however, you may have to
pay for your dentures or bridges)
-Included in an award of:
- Income support
- Income-related Employment and Support Allowance
- Pension Credit guarantee credit
-You are named on a valid NHS tax credit exemption certificate, or
you are entitled to an NHS tax credit exemption certificate
-You are named on a valid HC2 certificate.

Ref :- Pamphlet: NHS dental services in England-


www.dh.gov.uk/dentistry

77. A 55 year old female patient is missing her upper right second
premolar and upper right first molar and also is missing the upper left
second molar. The upper right second molar is functional and has an
amalgam restoration (MOD and buccal wall) that requires replacing. The
patient has no functional or aesthetic concerns. What would be the
treatment of choice in this situation?

A. Provide an upper removable partial denture


B. Replace the amalgam in the upper right 7 only
C. Provide a full coverage crown in the upper right 7
D. Provide a fixed bridge in the upper right quadrant
E. Provide a full coverage crown in the upper right 7 with guide planes
and occlusal rests.

Answer:

B78.
A new filling material has been developed by the Dental School. After
publication of laboratory results, the researchers conducted a
randomized clinical trial in general dental practice
where patients requiring one filling were randomly allocated either to old
or new filling material group.After five year follow up, the mean
survival time between two materials was compared, and P-
value of 0.125 was reported.

Which answer is correct?

A. There are no important differences in the properties of the materials


B. There is no difference in restoration mean survival time between the
two materials
C. The difference between two materials is not clinically important
D. The difference between the materials is very small
E. The new material is useless

Answer :B

Explanation: traditionally, one rejects the null hypothesis if the p-value


is smaller than or equal to the significance level,
[2] often represented by the Greek letter α (alpha).
If the level is 0.05, then
results that are only 5% likely or less, given that thenull
h y p o t h e s i s i s t r u e , a r e d e e m e d extraordinary. When we ask
whether a given coin is fair, often we are interested in the deviation of
our result from the equality of numbers of heads and tails. In such a
case, the deviation can be in either direction, favoring either
heads or tails. Thus, in this example of 14 heads and 6 tails,
we may want to calculate the probability of getting a result deviating
by at least 4 from parity (two-sided test). This is the probability of
getting at least 14 heads or at least 14 tails. As
the binomial distribution is symmetrical for a fair coin, the two-sided p-
value is simply twice the above calculated single-sided p-value;
i.e.
, the two-sided p-value is 0.115.In the above example we thus have:

• null hypothesis (H0): fair coin;

• observation O: 14 heads out of 20 flips;

and
• p-value of observation O given H0 = Prob (≥ 14 heads or ≥ 14 tails) =
0.115. The calculated p-value exceeds 0.05, so the observation is
consistent with the null hypothesis — that the observed result of
14 heads out of 20 flips can be ascribed to chance alone — as it
falls within the range of what would happen 95% of the time were
this in fact the case. In our example, we fail to reject the null
hypothesis at the 5% level. Although the coin did not fall
evenly,
thedeviation from expected outcome is small enough to be re
ported as being "not statisticallysignificant at the 5%
level".H o w e v e r , h a d a s i n g l e e x t r a h e a d b e e n o b t a i n e d , t h
e r e s u l t i n g p - v a l u e ( t w o - t a i l e d ) w o u l d be 0.0414
(4.14%). This time the null hypothesis –
that the observed result of 15 heads out of 20flips can be ascribed to
chance alone – is rejected when using a 5% cut-off. Such a finding
would be described as being "statistically significant at the 5% level".

79. A 23 year old male presents to your surgery. He lost his upper lateral
incisors some 10 years ago in a swimming pool accident. Since then he
has been wearing a `spoon' denture which he now feels in aesthetically
unacceptable. He has sought an opinion on dental implants but has
been told that he would need bone grafting for this to be
successful and he is not prepared to undergo this. His
dentition is excellent with no restorations and a Class I occlusion. He
wants some advice on what the best treatment might be. Which option
would you put first on your list of possibilities?
A. Two fixed resin bonded bridges using the central and canine teeth
B. Two cantilever resin bonded bridges from the central incisors and
canines.
C. Two conventional fixed bridges from the canine
D. Conventional cantilever bridges from the canines
E. Cobalt chrome partial denture

Answer: B

80. Universal curette and Gracey curette

Rake angle 90 and 70 degree.

•A universal curette
has a blade that is perpendicular to its terminal shank. This orientation
allows the blade to be used against either the mesial or distal surface of
a tooth. Because this type of periodontal curette was developed at
the Columbia University College of DentalMedicine, it is also known
as a Columbia curette.

•The Gracey curette


, invented by Dr Clayton Gracey with the help of Hugo Friedman
of Hu-Friedy Manufacturing company in the early 1940's, has a blade
that is laterally offset by70 degrees relative to the shank.
Consequently, a Gracey curette has a lower
cutting edge and an upper non-cutting edge. Because only one side of
each blade can cut, Gracey curettes are site-specific, and a posterior
instrument used to clean mesial surfaces of teeth won't work on distal
surfaces , and vice versa. "Gracey" Christina Patrick blades
used for mesial surfaces of anterior teeth from the facial are only
suitable for the distal surfaces of the same teeth when access is
performed from the lingual. Gracey Curettes 1/2 ,3/4 ,5/6 are used
on the anterior sextants of teeth.7/8 and 9/10 are used on the
buccal and lingual portions of posterior teeth.11/12 and 15/16
are used on the mesial portions of posterior teeth.13/14 and17/18 are
used on the distal portions of posterior teeth.

81. How to disinfect a bridge before sending to lab?

Answer: ultra sonic bath

82. Gingival enlargement due to phenytoin?


1. Affects males more than females
2. More in edentulous areas
3. Dose dependent
4. Occurs within 4-12 months after beginning of therapy

Answer:
A

No sexual predilection
exists for drug-induced gingival overgrowth, although in one study,
males were 3 times more likely than females to develop gingival
overgrowth with calcium antagonists.
•P h e n y t o i n h a s b e e n s h o w n t o i n d u c e g i n g i v a l o v e r g r o w t
h b y i t s i n t e r a c t i o n w i t h a subpopulation of sensitive fibroblasts.

•Cyclosporine has been suggested to affect the metabolic function of


fibroblast (eg, collagen synthesis, breakdown), whereas nifedipine,
which potentiates the effect of cyclosporine, reduces protein
synthesis of fibroblasts

82. Which antibiotic causes diarrhoea and abdominal cramps at a high


dose?

Answer:
Mainly: clindamycin causes abdominal cramps and diarrhea. (Antibiotic
induced colitis) ref- churchil book ( page no:497)
Others: less commonly erythromycin, penicillin, cephalosporin ( scully 5
th edi, page no:113)

83. which anion is present in bone?

Anion – phosphate and hydroxyl

Cation - calcium

2 significance of surface markers in hep B? – indicates acute or chronic


infection, immunity etc.

3 which virus is stable outside the body? – hep A

4 how to treat a spillage of less then 30 ml of blood? – 10,000ppm of


sodium hypochlorite.

5 which drug causes a fixed ulcer? – Nicorandil


6 who can give dental education?

84.GDC regulations

The GDC is an independent healthcare regulator established


under the Dentists Act 1956 (since replaced by the Dentists Act
1984, as amended). Its purpose is to protect the public
by regulating dentists and dental care professionals in the United
Kingdom. In particular, the GDC:
•Registers qualified professionals
(dentists, dental hygienists, dental therapists, dental nurses,
orthodontic therapists, dental technicians and clinical dental technicians)

• Sets standards of dental practice and conduct.


• Assures the quality of dental education.
•Ensures professionals keep up-to-date. (CPD)
• Helps patients with complaints about a dentist or a dental care
professional.
• Works to strengthen patient protection.

85. How often should routine radiographs be taken?

Posterior bitewing radiographs

High caries risk : 6 monthly interval


Moderate risk : Annually
Low risk: primary dentition: 12-18 month
Permanent dentition: 2 years

86. Which x-ray for implant? Conventional tomography or CT?

Answer:
CT is best for implant.
87. What type of cells are seen in the early and later stages of
periodontitis?

Answer:
Initial lesion: polymorphs
Early lesion: lymphocytes and polymorphs
Established: lymphocytes and plasma cells
Advanced: plasma cells dominate

88. The Isthmus diameter in cavity prep?

Answer:
Dimensions of balance between adequate depth without risking pulpal
exposure (1.5-2mm) and adequate width without weakening the
cusps(1/3-1/2distance between the cusps) in pedo pg 84 pink book.
However in class II adults the width of the isthmus should not overcut
(ideally 1/4 to1/3 inter cuspal width) pg 230 pink book.
89. Which area is more prone to develop cancer?

1 floor of the mouth is the single most common site followed by lingual
sulcus and ventral surface of tongue ( accounting for 75% of Cancer
seen in European and America)

90.What is the pressure of BPE probe?

Answer: 25 grms

91. The management of patients taking corticosteroid for long time with
dental infection, abscess or high temperature, do we need double the
dose of antibiotic or double the dose of cortisone ?

Answer;
Adrenal insuffiency may follow long term administration of oral
corticosteroids and
can persist for years after stopping therapy. A pt with adr.insuf. can beco
me hypotensive when under physiological stress. Acute adr.insuf. can b
e prevented by administration of an increased dose of corticosteroids
prior to treatment.

92. Do we need to change the antibiotic dose for patient with


rheumatoid arthritis taking 15 mg prednisolone for 6 months or 3
months had dental abscess?

Answer:
Antiobiotic prophylaxis- is not recommended for prevention of infective
endocarditis in adults or children undergoing dental procedures (scully )
except PATIENTS WITH PROSTHETIC JOINTS undergoing ( pg 396
Scully -6th edition )
1. dental at risk procedures - ( tooth removal, flaps, perio surgery - sub
gingival scaling, root planing, intra ligament inj, reimplant avulsed teeth,
endo beyond root apex, ortho banding )
2. Joint at risk- placed within 2 yrs, h/0 0f infection, joints in
hemophiliacs, type1 diabetics, Rheumatoid Arthritis, under
immunosuppressive therapy.
3. Antibiotic prophylaxis- 1-1.5hours AMOXICILLIN 2G CLINDAMYCIN
600 mgCEFALEXIN 2 G
AZITHROMYCIN 500 mg

CLARITHROMYCIN 500 mg

93. Why the patient with heart failure bleed easily?. How to manage
such bleeding?

Answer:
Pts with heart failure are on blood thinners that help prevent clots
forming in the blood. Blood thinners may cause to bleed easily or bruise
easily. Refer to cardiologist, check INR

.94. What is the main cause of collapsing a patient with congestive heart
failure in the dental clinic?

Answer:

any surgical procedures may cause undue stress resulting in


cardiac dysfunction(workload increase of the heart) which
surpasses the functional ability of the heart followed
by potential acute pulmonary edema. Pts with this condition present with
extreme dyspnea,hyperventilation, cough, hemoptysis, great difficulty in
breathing, murmurs due to cardiac asthma and cyanosis. The pt prefers
the sitting position, is anxious and might feel he/she is choking and if
death if imminent. The preventives measures are:

1. Written consent from the pt's cardiologist and consultation is


desirable
2. Oral premedication: 5-10 mg diazepam 1 hour before the
surg procedure
3. Small amounts of vasoconstrictors in LA with particular importance
of aspiration
4. Short appointments and as painless as possible

95. Which immunoglobin exists as both a monomer and diamer?

Answer:
IgA

96. about k files all are true except

a. has more number of flutes then reamer


b. is more flexible then reamer
c. used to machine the dentin
d. made up of triangular/square blank

Answer: b
k-files

1. have more flutes than reamers.


2.made up of triangular or square blank ( Reamers - triangle)
3. Used to machine dentine (like reamers) k-flex files are more flexible.

99. Fluoride in blood is carried by - calcium ions

Answer:
The mechanism of toxicity involves the combination of the fluoride anion
with the calcium ions in the blood to form insoluble calcium fluoride,
resulting in hypocalcaemia

100. What is the extra cranial cause of nerve palsy?


a. cerebral malignancy (Intracranial palsy)
b. stroke (Intracranial palsy)
c. MS (Intracranial palsy)
d. Ramsay hunt syndrome (Intracranial palsy)
e. Melkersson-Rosenthal syndrome

Answer: E

101. A young girl complains of swelling of cheeks both sides. On doing


various tests the salivary glands were found normal. What may be the
reason?

Answer: Cherubism

102. A 43year old patient is missing on the upper right first premolar and
molar. He has good oral hygiene and requests a fixed replacement
for these teeth. The other teeth on the same side are all
moderately restored with MOD amalgam restorations and are vital,
except the canine, which has a very large restoration and is root-
filled. He has group function. Radiographs show a large sinus
cavity and no peri-apical pathology. What would be the restoration of
choice for replacement of the missing teeth?

A. Implant supported crowns


B. A conventional fixed bridge using the 7 and 5 as abutments
C. Two conventional cantilevered bridges, using the 7 and 3 as
abutments
D. A resin-bonded bridge, using the 7 and 5 as abutments
E. A conventional fixed-moveable bridge using the 7 and 5 as abutments

Answer: B

Explanation:
a fixed bridge with 7 and 5 as abutments.

Cantilever bridges

are not preferred because they put a heavy load on the retainer and
distal cantilever bridges are rarely indicated due to high occlusal loads.

Resin bonded bridges


are generally for the anterior teeth and they again cant take up high
loads.

Fixed movable bridges


..in this case the cantilever effect of the non rigid component will put
extra load on the major retainer and these are also not indicated for
posterior teeth with heavy loading .

Implants
are best for single tooth replacement.

103. An 80 year old patient presents with an ulcer in the floor of the
mouth. This has been present for several months and has not
responded to conventional treatment. An incisional biopsy is taken.
Which of the following histological changes in the epithelium confirm a
diagnosis of squamous cellcarcinoma?

A. Hyperkeratosis
B. Acanthosis
C. Dysplasia
D. Invasion
E. Discontinuous epithelium

Answer: D
104. Cigarette smoking is considered to be the most important factor
next to microbial plaque in periodontal disease progression. Which of the
following is the most important factor in the disease progression in
smokers?

A. Smokers have drier mouths than non-smokers


B. Smokers have poorer oral hygiene than non-smokers
C. Nicotine will impair the chemotactic and phagocytic properties of
PMNs.
D. The gingival blood flow is reduced in smokers.
E. Smokers alter the oral environment encouraging the growth of
anaerobic bacteria.

Answer: D
http://www.tobaccoinduceddiseases.com/content/pdf/1617-9625-8-
4.pdf

105. A 35 year old male patient who admits to grinding his teeth at night
has a number of wedge-shaped cervical (Class V) lesions on his upper
premolar teeth. These are causing some sensitivity and are
approximately 3mm deep. What is the correct management option?

A. Provide tooth brushing instruction and fluoride


B. Restore the lesions with compomer
C. Restore the lesions with micro-filled composite
D. Restore the lesions with a hybrid composite

Answer: D

106 Cell which not present in healthy periodontium?

Answer:
Cementoclasts are found only in pathological conditions such as
application of excessive orthodontic forces, roots of deciduous teeth
undergoing resorption prior to exfoliation. Blue book pg 103
107. A patient gives a history of rheumatic fever. Which of the following
procedures require prophylactic antibiotic cover.

a. scale and polish


b. extraction of tooth
c. inferior dental nerve block
d. impression for a new lower complete denture
e. placing a class 1 amalgam restoration
Answer: none

108. Best method to diagnose below caries

1) caries half away through enamel on proximal surface - bitewing


2) caries halfway through enamel on occlusal surface - straight probe
3) caries half way through dentin not cavitated – bitewing
4) stained fissure – straight probe
5) hidden occlusal caries – wet tooth

Option: traslumination, bitewing, dry tooth, wet tooth, briault prob,


Williams’s prob, straight sharp prob

Answer: Bitewing

109. What is required to increase efficacy and reduce fatigue of


universal curettes?

Answer:
Sharpness of the instrument, Chair position, and instrument Grasp and
finger rest all are required to increase efficiency and reduce fatigue of
universal curettes.

110. Ideal angle required for sub gingival curettage from universal
curettes?

Answer: Ideal angle required for sub gingival curettage from universal
curette is 45 degrees. If 0 degrees then embedding the calculus to root
surface. If 90 degrees Laceration of soft issues will occur.

111. Suppuration is mainly the result of the combined action of four


factors which of the following is not one of them?

1. necrosis
2. presence of lymphocytes
3. collection of neutrophils
4. accumulation of tissue fluid
5. autolysis by proteolytic enzymes

Answer:
is 2. Presence of lymphocytes
112. What make scaling easier and less exhausting?

A. length of instrument
B. breadth
C. grip

Answer:
C

2) Same question about tiredness?


A. diameter of curette
B length
C thickness

Answer:
C

113. Which immunoglobin will increase in gingival inflammation?


Answer:
IgG

114. 8 year old, high caries rate prescribe fluoride content


500ppm, 800ppm, 1000ppm, 1250 ppm.
Answer: 1250ppm (1350-1400ppm)

115. What is gingival index?

Score 0- normal gingiva


score 1- Mild inflammation, slight change in colour, slight oedema, No
bleedingon Probing
Score2- Moderate inflammation, redness, oedema, Bleeding on probing
score3- Severe inflammation, marked redness, oedema, ulceration.
spontaneous bleeding
116. What would over trituration of silver and mercury cause and what
would under trituration cause?

Answer:
over trituration causes: hard and hot amalgamunder trituration causes:
grainy amalgam.

117. A patient on examination was found to have swollen gingiva around


a crown that had been present for several years. The papillae were
particularly enlarged. What is the most important feature of a crown
that may be responsible for this?

A. Material of the Crown


B. The occlusion
C. Proximal Contour
D. Labial Contour
E. Surface finish

Answer: C118.

What causes Amalgam expansion? What makes Amalgam black after


sometime?

Answer:

Amalgam Expansion:
1- Moisture contamination during mixing and condensation operation.
2- Moisture in the saliva is the potential contaminant for the amalgam.
3- Zinc containing amalgam , the presence of saliva on the amalgam
during condensation probably a possible source of expansion.

Amalgam Black:
1- corrosion.
2- reaction of the phases.
3- marginal leakage.
119. A patient presents with a history of a post-crown having fallen out.
The post-crown was originally placed fifteen years ago and had been
successful up until four months ago since when it has come out and
been recemented four times. At recementation there was no evidence of
any caries. The patient had been a regular attendee and not needed any
restorative treatment for the last eight years. Which of the following is
the most likely cause for the failure of this crown?

A. The post was to narrow


B. The post was to short.
C. The root canal treatment was failing.
D. A vertical root fracture was present.
E. There were excessive occlusal loads on the tooth.

Answer: D

Explanation
: Post crown was fixed 15 years ago & was fine till 4 months ago which
means there is s o m e d r a s t i c c h a n g e a f t e r s u c h a l o n g t i m e
w h i c h i s n o t g r a d u a l . C h a n g e i n o c c l u s a l l o a d i s normally a
gradual process & occurs especially after some restorations or
extraction are done or due t o s o m e p e r i o i s s u e s . P t h a d n o
treatment in mouth for last 8 years which indicates very
g o o d hygiene. The only reason in this case is a fracture in root
& is also supported by the fact that post crown needed
recementation 4 times in last 4 months coz root is not supporting
the post due tofracture.

120. A 50 year old male patient has a Class III jaw relationship with an
anterior open bite. It is planned to restore his lower right second molar,
which has suffered tooth wear and fracture, with anindirect restoration.
This tooth has approximately 2mm of coronal height.What would be the
most suitable approach to restore this tooth?

A. Provide an adhesively retained gold onlay


B. Provide a conventional full crown
C. Increase the vertical dimension and provide a full crown
D. Surgically crown lengthen and provide a gold crown
E. Provide an adhesively retained ceramic onlay

Answer:
A121. In dental epidemiology,
Indices
are used to measure the oral health of a population. The
DMF index is commonly used to measure the prevalence and severity
of dental caries.

122. You want to evaluate the effectiveness of tetracycline as an adjunct


to scaling & root planning for the treatment of chronic periodontitis. What
is the primary study design most appreciate for addressing this topic?

A. Cohort
B. Non randomised controlled trial
C. Randomised controlled trial
D. Control case study
E. Case series

Answer:
C

123. Which LA is neurotoxic at 4%


Answer:4% Articane is neurotoxic

124. .Which LA will u use in a child to do pulpotomy?

Answer:
2% LIGNOCAINE

125. Cyst in which white paste is seen?

Answer:OKC

126. What is the amount of gas in lungs at the end of the tidal volume?

Answer:
1000ml is residual vol

127. Anesthetic for patient with congenital heart disease in an


emergency appointment?

Answer:
2%LIGNOCAINE WITHOUT 1:80 000 ADR
128. Pt. with dentures that don't fit,

Answer:
macroglossia+ space in lower central incisors?

129. Minimum percentage of leucocytes for an extraction?

Answer:
50,000

130. Cement used for cantilever with little preparation?


Answer:
GIC LUTING

131. % of fissure sealants that fail and get carious?

Answer:
RETENTION OF F/S AT 48 MTHS= 52%
SO FAILURE = 48% (Nunn et al 2000,cochrane review, beauchamp
2008)
132. What is the % of transmission of these viruses?
Is it HBV 30%

HCV 3%

HIV 0.3%

which is the most stable virus outside the body?

Hepatitis A Virus

The hepatitis A virus, or HAV, is relatively hardy. In good conditions, it


can survive outside the body for months. HAV can survive certain acids
and some heat. For a period of time and under certain conditions, HAV
can survive in sea water, dried faeces and live oysters.

Hepatitis B VirusThe hepatitis B virus, or HBV, can still be infectious for


up to a week outside the body.

How long does thehepatitis C virus survive outside the body?


The hepatitis C virus can survive outside the body at room temperature,
on environmental surfaces, for at least 16 hours but no longer than 4
days.

Hepatitis D Virus: This virus needs help from the hepatitis B virus to
be infectious, so it only infects someone with hepatitis B.

Hepatitis E Virus:This virus is spread similar to the hepatitis A virus and


causes acute disease similar to the others.

3 in 1000 get HIV after needle stick injury

4 in 1000 get HEP c after needle stick injury

the risk is less than that in hep b because we are immunized that from
odell book 2010.

** HIV viruses has the least ability to survive outside the body as it
cannot survive outside the body for more than a maximum of 30 to 60
seconds

133. Which have most smooth muscle??


Answer:
arterioles

134. Child dental survey 2003

Answer:
There has been a decrease in the average
number of filled primary teeth
in both five and eight year-olds. The average number of filled primary
teeth
in five-year-olds
decreased from 0.5 teeth in1983, to 0.3 teeth in 1993
and 0.2 teeth in 2003
And
in eight-year-olds
decreased from 1.2 teeth in 1983 to 0.7 teeth in 1993
and 0.5 teeth in 2003

133. Which have most smooth muscle??


Answer:
arterioles

134. Child dental survey 2003

Answer:
There has been a decrease in the average
number of filled primary teeth
in both five and eight year-olds. The average number of filled primary
teeth
in five-year-olds
decreased from 0.5 teeth in1983, to 0.3 teeth in 1993
and 0.2 teeth in 2003
And
in eight-year-olds
decreased from 1.2 teeth in 1983 to 0.7 teeth in 1993
and 0.5 teeth in 2003.

• In 2003, 62% of 12-year-olds and 50% of 15-year-olds


w e r e f r e e f r o m a n y o b v i o u s d e c a y experience in the
permanent teeth. Among both 12 and 15-year olds 87% were
free from cavities into dentine in the permanent teeth.

135. A 30 year old man with unknown allergy to latex goes into
anaphylactic shock whilst being treated in the dental surgery. Which
drug and route of administration is of most benefit in this situation?

A. Hydrocortisone - orally
B. Chlorphenamine – intramuscularly
C. Chlorphenamine - orally
D. Epinephrine - intravenously
E. Epinephrine – intramuscularly

Answer: E

Explanation:

The first line of management for anaphylaxis is 500micrograms


or 0.5ml of epinephrine 1:1000given intramuscularly. It is fast
acting whilst hydrocortisone and chlorpheniramine are slower.
Hydrocortisone and chlorpheniramine are not relevant to a dentist in
general dental practice. When the paramedics arrive, the patient may be
given these other two drugs which give a longer duration of action than
epinephrine. Refer rhesus guidelines and Scully for more explanation
on this.136. What causes difficulty in pronouncing the letters?

F, V
– Incisors too far palatally
D,S,T
– incorrect overjet or bite, alteration in palatal contour

S becomes ‘th’
- Incisors too far palatally, palate too thick.

Whisteling
– palatal vault too high behind incisors

Clicking teeth – OVD, lack of retention


137. What is the irrigant that could be used in root canal
irrigation?4What concentration?

Answer: sodium hypochlorite - 2.5%

138. What combines irreversible to Hb (hemoglobin)?

Answer: Carbon monoxide (compound formed is carboxyhaemoglobin)

139. What are the success rate of first time RCT and the success rate of
Re-RCT?

Answer:
Success rate for first time RCT is 90% whereas for re RCT is 70%.

2- Should a re-RCT always be referred? Even if the patient is otherwise


fit and well?

Answer:
Safe bet would be to refer but at the same time explain the patient what
the endodontist will do.

140. BPE score * : is that pocket >7 mm

Answer:
when attachment loss (recession and pocket depth) is more than 7mm
or there is furcation involvement.
141.What type of fluoride (topical or systemic) you will prescribe for a 6
year and a 12 year old living in an area with water fluoridation of less
than 0.3 ppm.

Answer:
For <3- 0.25mg
For 6 year old - 0.5mg
For 12 year old - 1mg
For > 3 years 1350 – 1400ppm of toothpaste is advised.

142. .best method to detect sickle cell anemia:

a. full blood count – iron def


b.siderex - if results needed urgently
c. Hb electrophoresis – sickle cell anaemia.(on all afro Caribbean,
Mediterranean, middle easternand Indian)

Answer:
C

143. Iron def anemia is: Microcytic hypochromic - iron def (chronic blood
loss, inadequate diet)

A.Macrocytic anaemia (MCV > 100fl)


– B12 (pernicious anemia -deficit intrinsic factor). Folate deficiency –
usually dietary, illness (e.g celiac disease, skin disease) or drugs
(phenytoin, methotrexate, trimothoprim and co-trimoxazole)
Management: IM hydroxocobalamin 1mg 3monthly to treat B12
deficiency and folic acid 5mg odfor folate deficiency

B. Microcytic hypochromic (MCV <78fL)


- iron def (chronic blood loss, inadequate diet)Diagnosis: FBC (low
serum iron and increased total iron binding capacity (TIBC))Increased
RBC and zinc protoporphyrin is a fast and sensitive early test.
Management: ferrous sulfate 200mg tds.

C. Noromochromic noromocytic – anaemia of chronic disease. Other


causes pregnancy, acute bloodloss, hemolytic anemia and aplastic
anaemia.Diagnosis TIBC is reduced

Answer: B
144. Last deciduous tooth to be replaced by permanent one:
a. max canine
b. max 2nd molar
c. mand canine

Answer: A

145. Elongation of pulp chamber seen in:


a.gemination
b.turner teeth
c.taurodontism
d.dilaceration

Answer: C

Taurodontism
is found in association withamelogenesis imperfecta, ectodermal
dysplasia and tricho-dento-osseous syndrome. The term
means "bull like" teeth derived from similarity of these teeth to
those of ungulate or cud-chewing animals.

Turner's hypoplasia
is an abnormality found in teeth. Its appearance is variable, though
usually is
m a n i f e s t e d a s a p o r t i o n o f m i s s i n g o r d i m i n i s h e d enamel
o n p e r m a n e n t t e e t h . U n l i k e o t h e r abnormalities which affect a
vast number of teeth, Turner's hypoplasia usually affects only
one tooth in the mouth and, it is referred to as a
Turner's tooth. If Turner's hypoplasia is found on a canine or a premolar ,
the most likely cause is an infection that was present when the
primary (baby) tooth was still in the mouth. Most likely, the
primary tooth was heavily decayed and an area of inflamed tissues
around the root of the tooth (called a periapical inflammation), affecting
the development of the permanent tooth. The tooth most likely affected
by this cause is the canine tooth. The appearance of the
abnormality will depend on the severity and longevity of the
infection.

Gemination
arises when two teeth develop from one tooth bud
and, as a result, the patient has an extra tooth, in contrast to
fusion, where the patient would appear to be missing one tooth.
Fused teeth arise through
union of two normally separated tooth germs
, and depending upon the stage of development of the teeth at the
time of union, it may be either complete or incomplete. On some
occasions, two independent pulp chambers and root canals can be
seen. However, fusion can also be the union of a normal tooth bud to a
supernumerary tooth germ. In these cases, the number of teeth is also
normal and differentiation from gemination may be very difficult,
if not impossible. Ingeminated teeth, division is usually incomplete and
results in a large tooth crown that has a single r o o t a n d a s i n g l e
c a n a l . Both gemination and fusion are prevalent in primary
dentition, with incisors being more affected
Concrescence
is a condition of teeth where the cementum overlying the roots of at
least two teeth join together. The cause can sometimes be attributed to
trauma or crowding of teeth. Surgical separation of the teeth may
be necessary if one is to beextracted.

146. Potassium sulfate is added to gypsum to


a. increase setting expansion
b.decrease working time

Answer: B

147. The component of bacteria that attaches to cell process:

a. propionic acid
b. lipopolysachharide
Answer: B

148. the maximum cartridges of lignocaine that can be given to a pt

a.7
b.14
C.6

Answer: A

149. Hb e antigen means:


a. high risk infection
b. recent infection
c. immune to infection

Answer: A

150.question of candidiasis which is caused in particular group/races


(don't remember the exact ethnic group)

a.c.tropicalis
b.c.albicans

151.the approach for Gilles method in zygomatic #,

A.vertical incision is made in the temporal hairline, and the elevator is


then tunneled underneath thetemporalis fascia,
b.the same between the temporal fascia and temporalis muscle

Gilles method
A 3-cm incision placed 4 cm superior to the zygomatic arch
and posterior to the temporal hairline can be fashioned to allow
direct access to the arch. This approach (ie, Gillies approach) allows
accurate fracture reduction by means of a bimanual technique. The
surgeon creates a skin incision, the surgeon carries down a dissection
through the superficial temporal fascia and the Temporalis muscle fascia
(deep temporal fascia). A plane is carried forward, superior to the
temporalis muscle to the zygomatic arch. See the image below

Gillies approach to reduction of a zygomatic arch fracture.Once this


conduit is created, a periosteal elevator is positioned beneath the
zygoma. Lateral traction is placed on the elevator while the surgeon's
free hand palpates the fracture site during reduction. Once hemostasis is
ensured, the fascia and skin are closed in the usual fashion. Take care
to closethe wound with all layers reanastomosed to their respective
anatomic partners. Advantages of the Gillies approach include a scar
camouflaged by the patient's hair, accurate bimanual fracture reduction,
and a remote chance of injury to the temporal branch of cranial nerve
VII. Fluroscopy may aid in proper fracture reduction with closed
reduction

152.double blind method:

12.lot of ques based on histological features of pemphigus

Pemphigus vulgaris:

pathology
• Loss of intercellular adherence of suprabasal spinous cells
(acantholysis)• Formation of clefts immediately superficial to the basal
cells• Extension of clefts to form intraepithelial vesicles (Fig. 13.22)•
Rupture of vesicles to form ulcers• High titre of circulating antibodies to
epithelial 'intercellular cement substance' (desmoglein 3)• Binding of
antibodies to intercellular substance detectable by fluorescence
microscopy

13.trigeminal neuralgia features:

a.trigger zones

b.night pain

153.the strength of adrenaline used

1:X

where X is

a.100
b.1000
c.10000

Answer: B
154.ques based on TNM classification

THE TNM SYSTEM OF STAGING

T1 less than 2 cm greatest dimension


T2 2- 4 cm greatest dimension
T3 > 4 cm greatest dimension
T4 extending to adjacent structures, e.g. bone, sinus, skin

N0: No regional lymph node metastases


N1: One ipsilateral node < 3 cm diameter
N2: Ipsilateral or contralateral nodes 3-6 cm diameter
N3: Lymph node metastasis > 6 cm diameter

M0: No distant metastases


M1: Distant metastasis (e.g. liver, lung)

Scores are compiled to designate the stage as follows:


Stage1: T1 N0M0
Stage 2: T2N0M0
Stage 3: T3N0M0
Stage 4: any T4 any N2 or N3 any M1

SITES OF ORAL CANCER


The lower lip is the most frequent site of oral cancer overall, while the
tongue is the most frequently affected site
within the mouth. In the oral cavity, the majority of cancers are
concentrated in the lower part of the mouth, particularly the
lateral borders of the tongue, the adjacent floor of the mouth and
lingual aspect of the alveolar margin, forming a U-shaped area
extending back towards the oropharynx (Fig. 17.2). This accounts for
only about 20% of the whole area of the interior of the oral cavity, but
70% of oral cancers are concentrated there. This distribution
may be due to the likelihood that carcinogens could pool and
concentrate in the lower mouth before swallowing. F o r t h e s a m e
reason, the hard palate and central dorsum of tongue are
very rarely affected.
16.Ques based on BPE
155.pemphigus shows:

a.acantholysis

b..acanthosis

Answer:A

156. IgG and C3 are seen in:

a. lichen planus
b. phemphigoid
c. erythema multiforme
d. phemphigus vulgaris

Answer: D

157. Commonly used treatment of candidiasis:


a. flucanozole
b. micanazole
c. nystatin

Answer: C
158. actinomycosis is differentiated from osetomyelitis by the presence
of:
a. sequestrem
b. involucrum
c. sulfpur granules
d. pain

Answer: C

159. wharf’s assessment is used in

a. fractures
b.3rd molar assessment

160.AVPU is used in

a.ABCDE
b.ABC
161.define epidemiology

Epidemiology is defined as the study of distribution and determinants of


health related states in populations and use of this study to address
health related problems.

162.histopath lesion closest to advancing edge in enamel caries:

a. dark zone
b. surface zone
c. translucent zone

Answer: C

163. pogonion, menton,and gonion are the bony landmarks in

a. mandible
b. max
c. bony chin
Answer:
A

164. best method of fluoridation – systemic

165.simplest way to correct a lateral incisor cross bite :

a.removal appliance
b.fixed appliance

Answer: A

167. case scenarios where one central incisor is missing in a child who
has had a previous historyof trauma with an avulsed primary incisor, the
cause is

a.scar tissue
b.supernumerary teeth
c.tubercle
d.frenum

168. based on AAA (American anesthesia association) when can u treat


a dental pt:
a. type /class 1
b. type 1 or 2
c.type 5
d.type 4 and 5

ASA Physical Status Classification SystemASA Physical Status 1


- A normal healthy patient

ASA Physical Status 2


- A patient with mild systemic disease

ASA Physical Status 3


- A patient with severe systemic disease

ASA Physical Status 4


- A patient with severe systemic disease that is a constant threat to life

ASA Physical Status 5


- A moribund patient who is not expected to survive without the
operation

ASA Physical Status 6


- A declared brain-dead patient whose organs are being removed for
donor purposes

169.ques on pulpotomy
170.pontic
171.when can u say lkp is turning malignant:
172.ques on epi dysplasia
173.What is the concentration of chlorhexidine in a gel?
0.2%, 0.5%, 1%, 5%
Answer: Chlorhexidine as a mouthwash is 0.2% and gel is 1%
(Corsodyl)Ref : pink book page no. 32
3 1 3
1 4 3

174. A 46-year-old male smoker presents as a new patient complaining


of bleeding gums, bad breath and his BPE score is given as above:

Select the most appropriate initial radiographic examination.


A. Bitewings
B. Bitewings and periapical views of selected teeth
C. Full mouth periapicals
D. Periapicals of the lower incisors
E. Vertical bitewings
Answer: B

175.

A.1 month
B. 3 months
C. 6 months
D. 12 months
E. 24 months
F. 36 months

Choose from the options above the period of time which should elapse
before the next radiographic review in the scenarios below. Each option
may be used once, more than once, or not at all.
1. A 13-year-old patient designated as having a high caries risk. - 6
months
2. A 15-year-old patient considered to be at moderate risk of
futurecaries. - 12 months
3. A 32-year-old patient still considered as at high risk of future caries. -
6 months
4. A 9-year-old patient at low caries risk. 12-18 month
5. A 25-year-old patient at moderate risk of future caries. Annually
6. A 38-year-old patient who has had a full coverage crown placed.
Annually
7. A 27-year-old patient who has had orthograde endodontic treatment
to UL6. Annually
8. A 7-year-old who has had a vital pulpotomy following trauma to UL1.
Annually

High caries risk


: 6 monthly interval
Moderate risk
: Annually
Low risk:
primary dentition: 12-18 month. Permanent dentition: 2 years
176.
A. Clubbing
B. Erythematous palms
C. Evidence of widespread scratching
D. Flattened nails (koilonychias)
E. Keratotic striations
F. Pitted nailsG. Purpura
H. 'Target' lesions.

For each of the following clinical scenarios identify the most appropriate
skin/nail condtion fromthose provided. Each option may be used once,
more than once, or not at all

1. A 45 year old woman with known liver disease. - H


2. A 56-year-old psoriasis sufferer. B (can also be ‘C’ Due to itching)
3. A woman with a hypochromic microcytic anaemia. - D
4. A heavy smoker with haemoptysis. - A
5. A patient with a history of gallstones presenting with dark urine
and jaundice. - C

Explaination for Q 5
-
Post-hepatic
jaundice, also called obstructive jaundice, is caused by
a n interruption to the drainage of bilein the biliary system. The most
common causes aregallstonesinthe common bile duct, and pancreatic
cancer in the head of the pancreasPatients also can present
with elevated serum cholesterol, and often complain of severe
itching or "pruritus" because of the deposition of bile salts.

177. Ankylosis
Our teeth stay anchored to our jawbone, thanks to healthy tissue
and ligaments. Normally, the roots and ligaments for primary ("baby")
teeth will dissolve, allowing the baby teeth to come out
and the permanent teeth to emerge from underneath. Sometimes the
roots don't dissolve properly, and instead they fuse directly to the
jawbone. The fusion may occur because the ligament that normally
surrounds the tooth in the jawbone is lost. This condition is called
ankylosis
(ang-kil-LO-sis).

Causes of ankylosis
The causes for ankylosis vary; for example, the healthy ligament cells
may dry up and die because of:
• Inflammation or infection
• Problems with metabolism or normal bone growth
• Congenital tendencies
• Gaps in the membranes around the tooth
• Abnormal pressure from the tongue
• Crushing or other damage

Ankylosis also may develop in children or adults when an injury (such as


with sports or accidents)causes the top of the tooth to be broken off,
leaving the roots behind.

Some studies have indicated that 1 to 3 percent of children may have


ankylosis with one or more primary teeth, while other studies have
indicated more than 38 percent of children may have ankylosis. Studies
also have shown that children or adolescents with 1 or 2 ankylosed teeth
are more likely to have additional ankylosis later. These variances in
statistics may be due to different ways of diagnosing ankylosis, ethnic
factors, or other reasons. (For example, a study at the Pediatric
Dentistry Clinic of the University of Minnesota's School of Dentistry
focused only on Caucasian children.) But the studies all come to the
same conclusion: while it may not be an unusual condition, especially
with lower first molars, ankylosis is a serious condition. To ensure the
healthy development of your child's teeth, ankylosis must be addressed.

Signs and sounds of ankylosis


Some of the signs of ankylosis may include:

•The ankylosed tooth appears submerged because its roots don't grow
at same rate as other teeth.

•The opposing tooth visibly loses alignment as it grows out of its socket.

•The permanent tooth is blocked out of position because the primary


tooth's roots failed to dissolve normally.

Often we can diagnose ankylosis simply through observation. For


example, a tooth noticeably lower in growth is a tell-tale sign of
ankylosis. Also, with x-rays taken at regular checkups, we can view the
development of permanent teeth that may be blocked by the primary
teeth's roots. Special, sophisticated instruments can be used to identify
ankylosis. But one of the most reliable instruments is our ears! Simply by
tapping on the teeth, we can hear a distinct difference between
anankylosed tooth and a normally growing tooth. That's why we may use
the clean handle of a dental mirror to tap lightly on children's teeth as
part of their regular dental exam. (An ankylosed tooth has a higher
pitched or dulled sound as opposed to the more cushioned sound of a
normal tooth.)

Solutions for ankylosis

We may recommend several solutions for ankylosis, depending on the


specific tooth and your child's situation. For example, with younger
children, we may simply recommend monitoring the situation for a period
of time, to watch how the primary and permanent teeth develop. Sooner
or later we may recommend removing the ankylosed tooth, to ensure
that your child's permanent teeth can develop straight and strong. We
also may recommend surgery to expose, protect, or reposition the
emerging tooth. If we must remove a primary tooth before the permanent
tooth is ready to emerge, we may use space maintainers to ensure that
surrounding teeth do not crowd out the emerging permanent tooth.
Sometimes orthodontic steps may be taken to ensure that your child's
teeth align and the bite is
correct. In general, the sooner we can deal with your child's
ankylosed tooth, the fewer problems that are likely to develop later.

178. A mother brings her child to the out of hours clinic suffering
spontaneous excess bleeding from his gingival, what is the
expected diagnosis?

- acute leukemia
- injury
- stress
- scurvey

Answer: A

179.
The most common type of injury caused by a Non accidental Injury is:
- burnt tip of tongue
- ulcer on the gingiva
- lacerated labial frenum

Answer: B

180.
The most common scale used for diagnosing a brain injury is:
Answer:
Glasgow Coma Scale (GCS).

181. The most important feature to differentiate between an upper


neuron motor lesion and a lower neuron motor lesion is:

- eye involvement
- ear involvement
- forehead involvement
- anaesthesia of the facial nerve

Answer:

C
Main difference:
frontalis and orbicularis oculi muscle is less paralysed, the facial muscle
may appear non-paralysed during emotional reactions and there is
usually a degree of paralysis of ipsilateral arm and leg or aphasia.
(Scully page no – 578)

182.
A supernumerary tooth next to the maxillary lateral incisor is called

a:- conical tooth


- supplementary
- tuberculate

Answer: B

183.
A patient attends your clinic for swollen lumph nodes, after examination,
the diagnosisrevieled T2N1M0, what does that mean?

- A tumor of 1 cm, with one lymph node involved and no metastasis.


- A tumor of 2 cm, with one lymph node of 1 cm involved and
no metastasis.
- A tumor of 2-4 cm, with ipsilateral lymph nodes involved and no
metastasis.

Answer: C

184.
You suspect the patient suffers hepatitis B symptoms, after investigation,
the test show HBe Ag antibodies, what does that indicate?

- the patient is getting better


- the patient is in a transmitting state of the disease

Answer:B

185.
Peptic ulcers are caused by the following type of bacteria:

- mycobacterium tuberculosis
- sterptococus oralis
- helicobacter pylori

Answer: C

186.According to the modified Anxiety Scale, a dental phobic patient is


on a scale of:
- 9-14
- 15-20
- 20-27
- 27-35

10- The age range for the following diseases are: pemphigus vulgaris,
mucouse membrane pemphigoid,lichean planus
Pemphigus Vulgaris
Management

The diagnosis must be confirmed as early as possible.


B i o p s y i s e s s e n t i a l a n d t h e c h a n g e s a r e sufficiently
characteristic to make a diagnosis. Immunofluorescence microscopy
should be used to
exclude similar but less common diseases. Once
t h e d i a g n o s i s h a s b e e n c o n f i r m e d , immunosuppressive
treatment is required. There is little consensus about dosage but
a typical regimen is 80-100 mg/day of predisolone plus
azathioprine (1-1.5 mg/kg daily). Azathioprine is given to allow
doses of the corticosteroid to be lowered and reduce their side-effects.

Pathology

Histologically there is loss of attachment and separation of the full


thickness of the epithelium from the connective tissue at basement
membrane level. Epithelium, though separated, remains for a time intact
and forms the roof of a bulla (Fig. 13.26). The floor of the bulla is
formed by connective tissue alone, infiltrated with inflammatory cells.
The disease is immunologically mediated, and binding of
immunoglobulin or more frequently of complement components along
the basement membrane zone can be demonstrated (Fig. 13.27).
Circulating autoantibodies are detectable by sensitive techniques.

Management
The diagnosis is confirmed by biopsy and immunofluorescence
microscopy but it is preferable to obtain an intact vesicle or bulla.
Oral mucous membrane pemphigoid can often be effectivelycontrolled
with topical corticosteroids. Doses are small and without systemic
effects. Because of the possible risk to sight, ocular examination is
necessary if early changes in the eyes are
suspected. If t h e e y e s b e c o m e i n v o l v e d , s y s t e m i c c o r t i c o
steroids have to be given and are effective.

11- The percentage of people surviving a cardiac arrest in the UK is…

12- Winters lines on an OPT is used to:….

13- Verifiable and non-verifiable CPD hours, and core CPD hours

14- Maximum dose allowed for: amoxicillin, aspirin, ibuprofen,


paracetamol

15- The action of these drugs

16- What do the following trade names indicate: Panadol, Dactarin,


Fusidin, Augmentin..

17- Endodontics: K-files, Hedstrom files, EDTA, Gutta percha,


Chloroform

18- Root filling techniques: step down, step back, vertical condensation,
thermal condensation
19- Kennedy classification,

20- Eruption times and numbering systems for teeth

21- Medical emergencies: anaphylactic shock, hypoglycaemia, MI,


asthma

22- A patient suddenly collapses, what is the first thing you need to do:
call 999, check the airway isclear, Give him glucagon..

23- Types of suturing needles, their sizes and for which wounds they are
used

24- You are a VT working in a dental clinic, to which departments do you


refer the following patients: a patient suffering Addison's disease, a
patient suddenly becomes confused with slurred speech and not had
breakfast, a patient needs his third molar to be extracted under sedation.
A patient who looks anaemic..

25- Which of the following materials has an alkenoic reaction: calcium


hydroxide, ZOE, GIC

26- To increase the strength of the amalgam and decrease its corrosion
which of the following needsto be added: mercury, copper, silver..

204. A man has diet-controlled type 2 diabetes. He consumes


30 glasses of alcohol and smokes 40cigarettes per day. He is otherwise
healthy. What may be the reason of dry mouth in this person?

Sjogren's sydrome,
candidosis,
dehydration,
drug-induced.

Answer. C

205.
Which among the following is least important in taking consent?The
trainee understands the procedure of taking consent, he has done it
previously, he understandsthe risks associated with the procedure, he
explains the risk to the patient, he follows the procedurefrom a textbook .
Answer:
trainee done it previously

205. What colour bag for disposing waste in the waiting room of a clinic

Answer:
Black

206. Laws and regulations governing different scenarios e.g Dental


Nurse wanting to take radiograph. Options included

IRMER,
Health and Safety,
COSHH.

206 Which sealant is more likely to cause tissue damage if extruded


– AH plus, grossman's sealer,tubliseal.

207Which LA becomes neurotoxic at 4%.

Answer:
Articine

207Cellularity of exocrine glands

-Uni,
multi,
bi

etc as choices.

209. What is the kind of epithelium lining exocrine ducts?

210. What affects cardiac output

-Stroke vol and heart rate.

21 . what decreases when u exercise

Answer:
total peripheral resistance
212. Patients with hypothyroidism should not receive conscious sedation
with benzodiazepines, but i s p o s s i b l e t o u s e n i t r o u s o x i d e a n d
oxygen. The problem with benzodiazepines and opioid
analgesics is the risk of
myxoedema coma.

I t h i n k t h i s a p p l i e s f o r b o t h t r e a t e d a n d u n t r e a t e d patients,
because thyroxine is only given for symptomatic patients.

213. A 64 year old man with type 2 diabetes


and raised blood glucose and sodiumlevels
is already being treated with iv insulin
a n d s a l i n e w h a t d r u g s h o u l d b e added?

Answer: IS HEPARIN BECAUSE THEY ARE MORE LIKELY TO


DEVELOP BLOOD CLOTS(THROMBUS ) IN THE BLOOD .SO TO
AVOID THAT HEPARIN IS GIVENREF: EMQ IN DENTISTRY

214. cells in herpetic stomatitis?


options lymphos ,monos,neutrophils,macros,plasmacells

215.. percantage of patients indicated to hospitals by infection?


options5%,10%,15%,20%

216. muscle involved in snoring?

Answer;
genioglossus, palatoglossus

217. How many cpd hrs do the nurses need from 1st
August 2008 (started compulsory)

Answer:
150 in 5 year cycle
50 verifiable.

218.. How many cpd hrs does the dentist need?

Answer:
250 in 5 year cycle, 75 verifiable, 50 approximately per year

219. Core subject for CPD


Answer:
Medical emergencies - 10 hrs
Disinfection/decontamination – 5hrs
Radigraphy/radiation protection – 5 hrs
For dentists working in clinical environment, CPD in legal/ethical issues
and handling complaints should be carried out (verifiable and non
verifiable)

220. When an individual is exposed to radiation a certain amount of


radiation is needed before clinical signs of damage to somatic cells
appear. For these effects to occur a minimum radiation dose has to be
exceeded and this is known as:

A. Background radiation dose


B. Threshold dose
C. Equivalent dose
D. Absorbed dose
E. Effective dose

Answer: D
221. A panoramic cassette was opened in the darkroom to remove and
process the exposed film. On opening the cassette, a piece of paper
was discovered on the surface of the intensifying screen. What kind of
artefact would the presence of paper in the cassette most likely
produce?
A. A black artefact
B. No artefact
C. A white artifact
D. Reticulation
E. Dichroic fog

Answer: A

222. You arrive at a new practice and notice that almost every
radiograph in the patient's notes has turned brown. Your nurse confirms
that this is a widespread problem that no-one has ever remedied and
she also remarks that the films tend to get browner with age. What
corrective action will you take to remedy the problem?
A. Develop the films for the correct period of time
B. Heat the developer to a higher temperature
C. Ensure films are fixed for the correct period of time
D. Wash the films properly after fixing
E. Change the solutions more regularly

Answer: D

223.
The size of the wire used for splinting an avulsed tooth and why we do
bucally.

Answer:
It is 0.6 mm (or basically a semi-soft wire). Wire placed buccally because
it is easier during treating trauma and better moisture control for
bonding the wire to the teeth.

224. The gutta percha becomes soft at 65c and melts at 100c

225. The management of herpes labialis is by topical antivirals NOT


systemic (tablets) as follows:

Aciclovir cream 5% 4 hourly ( is used during the prodromal symptoms


stage, before ulcers appear,otherwise it won't be effective)

Penciclovir cream 1% 2 hourly ( it can be used after the appearance of


the ulcers and it acceleratesthe healing process)

Systemic aciclovir (tablets) are only used in immunocompromised


patients with hepres labialis andin such case the lesions are
more severe and appear intra orally as well...

226. basically there are three types of hand hygiene proc:

1,social for 10 to 15 sec using liquid soap for non clinical activities like
decontamination

2.hygienic for 15 - 30 sec using antimicrobial disinfectants before and


after using gloves(clinical proc)

3.surgical scrub 2-3 mins using 4% chlorhexidine gluconate or 7.5%


povidone iodine after oral, periodontal or implant surgery
ref: BDA may 09
227. MAX.RECOMMENDED ALCOHOL UNITS FOR MEN PER WEEK .
MAX.RECOMEMMENDED ALCOHOL UNITS FOR WOMAN PER
WEEK

Men should drink no more than 21 units of alcohol per week (and no
more than four units in anyone day).

Women should drink no more than 14 units of alcohol per week (and no
more than three units in any one day). Man should not regularly drink
more than 3-4 alcohol units a day (equivalent to a pint and a half of 4%
beer) and a woman should not regularly exceed 2-3 units a day
(equivalent to a 175ml glass of 13% wine).1unit of alcohol = 1/2 pint or
10 ml or 8gm

228. 15 YEAR BOY LOST HIS CENTRAL TOOTH IN SKIING-WHAT IS


THE BEST OPT ?

Answer:
15 yrs is still growing, so offer only Dt or resin bonded and you should
inform him about the space that going to happen below the pontic.

229. What part of the nervous system causes increased salivary


secretion?

Answer:
parasympathetic

230. What part of the nervous system increases heart rate?

Answer:
Sympathetic

231. What hormone increases blood glucose?

Answer:
Glucagon

232. What hormone reduces blood glucose?


Answer:
insulin
233. What hormone increases blood glucose and decrease S.
Potassium?
Answer:
Cortisol.

234. 10% of UK population receives fluoridated water.

235. Is Myasthenia Gravis and Grave's disease type 2 or type 5


hypersensitivity reaction?

Answer:
Both, mediated by IgM and IgG

236. Which immunoglobulin increases in gingival inflammation?

Answer:
IgG

237. What type of cell junctions are seen in oral mucosa?

Answer:
desmosomes, gap junctions and zona occludens

238. Percentage of caries in children in the UK

Answer:
43% 5 years old and 57% 8 years old by children heath survey 2003

239. Most abundant extracellular cation?

Answer:
Na is main extracellular cation while K is main intracellular cation.

240. ig present in periodontitis?

Answer:
IgG especially IgG2

241. Which immunoglobulin is dimeric?

Answer:
secrotory IgA

242. Antibiotic contraindicated in lactating mothers?

Answer:
QUINOLONES, TETRACYLINES, METRANIDAZOLE,
SULFONAMIDES

243. Which nerve makes a pt. look upwards n downwards?

Answer:
OCULOMOTOR NERVE (3RD NERVE)

244. Which has softer walls? veins, artery or arterioles?

Answer:
Veins

245. Most common failure of maryland bridge?

Answer:
Retention failure is the most common prob associated with Maryland
bridges.

246. % of alcoholics in uk?

Answer:
5% ADULTS ARE ALCHOLICS

247. %children with caries less than 5years old?

Answer:
43% OF 5 YEARS OLD

248. Cement used to temporary cement a crown?

Answer:
Cementation of temporary crowns is with the help of ZOE.(tempbond)

249. Which cement is more susceptible to fail if contaminated with


saliva?
Answer:
Cement most susceptible to failure if contaminated with saliva is GIC

250. % of ppl in uk who drink too much?

Answer:
50% OF MEN AND 30% OF WOMEN EXCEEDS THE
RECOMMENDED WEEKLYGUIDELINES

251. Measles caused by rna or dna virus?

Answer:
RNA virus

252. cjd/ prions most infectious? n that dnt get sterilised.

Answer:
PRIONS RESISTANT TO INACTIVATION BY CONVENTIONAL
STRLIZATIONMETHODS

253. Which is elastic artery or vein?

Answer:
Artery

254. The layers of the skin are as follows:from in to outwards:


stratum basale
stratum spinosum
stratum granulosum
stratum lucidum
stratum corneum

The stratum lucidum is a layer of the epidermis found throughout the


body, but is thickest on the palms of the hands and the soles of the feet.

255. What is the main function of EDTA?

Answer:

Is a chelating agent,

Negotiate sclerosed canals,


dissolve inorganic component of smear layer which sodium hypochlorite
can’t do.
Sodium hypochlorite: dissolve organic debris and it’s bactericidal, 2.5%
conc available chlorine.

256. A 22 year old woman has acute gingival hypertrophy, spontaneous


bleeding from the gingiva and complains of weakness and anorexia. Her
blood analysis was as follows: HB=12gm, Neutrophils=90%,
Monocytes=1%, Platelets=250000, WBC=100000,
Lymphocytes=9%,Eosinophils=0%The most likely diagnosis is:

a. Myelogenous leukaemia
b. Infectious mononucleosis /glandular fever/
c. Thrombocytopenic purpura
d. Gingivitis of local aetiological origin
e. Pernicious anaemia /Vitamin B12 deficiency/

Answer: B
(Option A Could be the answer too if constitutional symptoms like weight
loss, fever, night sweats are there)

257. Which of the following organisms are pathognomonic of acute


necrotic ulcerative gingivitis?

a. Spirochaetes and fusobacterium SP


b. Spirochaetes and eikenella corrodes
c. Polymorphs and lymphocytes
d. Actinobacillus actinomycetes comitans oral capnocytophaga
e. Porphyromonas gingivalis and prevotella intermedia

Answer: A

258.
Which of the following is true regarding gingivosis (Desquamative
gingivitis)

a. It is caused by hormononal imbalance


b. Is seen only at or after menopause
c. Is frequently caused by lichen planus
d. Is a variant pregnancy gingivitis
e. Is related to nutritional disturbance
Answer: C

259.
The treatment of Localised Juvenile Periodontitis is frequently
supplemented with tetracycline because flora involved is predominant:
A. Aerobic
B. Strictly anaerobic
C. Facultative or microaerophilic
D. Resistant to other antibiotic

Answer: C
Prevotella intermedia - Localised Juvenile Periodontitis, necrotizing
periodontal disease.
Prophyromonas gingivalis – chronic periodontitis and aggressive
periodontitis
Actinomycetem comitans – microaerophilic, capnophilic gram –ve
rod, pathogen in aggressive periodontitis

260.
The most accurate way to evaluate the effectiveness of root planning is
by:

a. Inspect the root surface with an instrument for root smoothness


b. Use air for visual inspection
c. Activate a curette against root surface and listen for a high pitched
sound which indicates as mooth, hard surface.
d. Evaluate the soft tissue at the end of the appointment for a
decrease oedema and bleeding
e. Evaluate the soft tissues 10 to 14 days later.

Answer: A

261. Probe pressure at the sulculus of pocket should be enough to:

a. Feel the top of the crestal bone


b. Balance the pressure between fulcrum and grasp
c. Define the location of the calculus deposit
d. Feel the coronal end of the attached tissues
e. Limit the lateral pressure

Answer: D

262) A curette may be inserted to the level of the attached gingiva with
minimal trauma to thetissues because of:

a. Has a round base


b. Is easy to sharpen
c. Has rounded cutting edges
d. Provides good tactile sensitivity
e. Has two cutting edges

Answer: C

Explnation:
A periodontal curette is a dental instrument used primarily in the
prophylactic and periodontal care of human teeth. The working tips are
fashioned in a variety of shapes and sizes, but they are always rounded
at the tip in order to make subgingival cleansing less traumatic
to thegingiva.

263) Tetracycline hydrochloride conditioning of root surface in


periodontal surgery is to:

a. Sterilise the root surface


b. May enhance binding of fibronectin and fibroblast
c. Aids in re-mineralising the root surface
d. Assist the biding of lamina dura
e. Prevents post operative infections

Answer: B

264. A cold stimulus applied to a tooth will produce a hypersensitive


response if the tooth

A. is non vital
.B. has a periodontal pocket.
C. has a hyperemic pulp.
D. has chronic proliferative pulpitis.
Answer: C
265. The location and extent of subgingival calculus is most accurately
determined clinically by

A. radiopaque solution used in conjunction with radiographs.


B. disclosing solution.
C. probing with a fine instrument.
D. visual inspection.

Answer: C

266. A characteristic sign of aggressive periodontitis in an


adolescent (juvenile periodontitis) is

A. marginal gingivitis.
B. painful, burning gingivae.
C. hyperplastic gingivitis.
D. drifting of the teeth.

Answer: A

267. Which treatment procedure is indicated for a patient with


asymptomatic age related gingival recession?

A. Connective tissue graft.


B. Gingivoplasty.
C. Lateral sliding flap.
D. Gingival graft.
E. No treatment.

Answer: E

268. The absence of adequate drainage in a periodontal pocket may


result in
A. cyst formation.
B. abscess formation.
C. epithelial hyperplasia.
D. increased calculus formation.

Answer: B
269. For an otherwise healthy patient, with an acute localized
periodontal abscess, initial treatmentmust include

A. scaling and root planing.


B. occlusal adjustment.
C. prescription of an antibiotic.
D. prescription of an analgesic.

Answer: A

270. The most likely cause of tooth loss following a tunneling procedure
to provide complete accessfor a mandibular Class III furcation
involvement is

A. root caries.
B. root sensitivity.
C. pulpal involvement.
D. recurrent pocketing.

Answer: A

271. Maximum shrinkage after gingival curettage can be expected from


tissue that is

A. fibroedematous.
B. edematous.
C. fibrotic.
D. formed within an infrabony pocket.
E. associated with exudate formation.

Answer: B

272.

Apart from Dentigerous Cyst, which other lesions are associated with
absence of teeth?
Answer:
Adenomatous odontogenic tumor (canines) Keratocyst
Ameloblastoma
Calcified Odont tumor
273. Regarding Hand washing

Answer: Chlorhexidine gel apart from alcohol and povidone iodine as a


surgical hand washing gel. Apart from MRSA, it prevents the spread of
clostridium difficle.

274. What is the normal resting flow rate for saliva?

Answer:

vary b/w 0.1-0.6ml per min


Stimulated saliva 1-2ml per min

275 .Which nerve is affected if a patient is unable to gaze laterally to


the left?

-rt abducent
-rt trochlear
-lft abducent
-lft trochlear

Answer: A

Left abducent as it supply lateral rectus of the eye which is responsible


for lateral movements

276. development of the parotid gland in utero in which month?

Answer

development of parotid gland in utero (at what months)---40-44 days in


utero which would be approx 1.5 months

277. Muscles used in swallowing?

278. Bone formed totally intramembranously?

-frontal - parietal -occipital -mandible -sphenoid -temporal

Answer:

All Intramembranous ossificationis:


1. parietal
2. maxilla
3. frontal
4. nasal bone
5.vomer
6. lacrimal
Mixed is: (TOMS)
1. temporal
2. occipital
3. mandible
4. sphenoid
Endocondrial OSSIFICATION:
1. HYOID
2. INFERIOR NASAL CONCHAE
3. Ethmoid boneExplanation:
Intramembranous ossification

Intramembranous ossification mainly occurs during formation of the flat


bones of the skull but also the mandible, maxilla, and clavicles; the
bone is formed from connective tissue such as mesenchyme tissue
rather than from cartilage. The steps in intramembranous ossification
are:

1.Development of ossification center


2.Calcification
3.Formation of trabeculae
4.Development of periosteum

Endochondrial ossification

Endochondral ossification, on the other hand, occurs in long bones and


most of the rest of the bones in the body; it involves an initial hyaline
cartilage that continues to grow. The steps in endochondral ossification
are:

1.Development of cartilage model


2.Growth of cartilage model
3.Development of the primary ossification center
4.Development of the secondary ossification center
5 . F o r m a t i o n o f a r t i c u l a r c a r t i l a g e a n d epiphyseal plate

Endochondral ossification begins with points in the cartilage called


"primary ossification centers."They mostly appear during fetal
development, though a few short bones begin their primary ossification
after birth. They are responsible for the formation of the diaphyses of
long bones, short bones and certain parts of irregular bones. Secondary
ossification occurs after birth, and forms the epiphyses of long bones
and the extremities of irregular and flat bones. The diaphysis and both
epiphyses of a long bone are separated by a growing zone of cartilage
(the epiphyseal plate).
When the child reaches skeletal maturity (18 to 25 years of age), all of
the cartilage is replaced by bone, fusing the diaphysis and both
epiphyses together (epiphyseal closure).

279. 5.what condition is caused by failure of closure of the vertebral


arches?

Answer:
Spina bifida

280.If the gland on 1 side are stimulated, what will the response be like?

Answer:

Ipsilateral

281. Muscles attached to the auditory tube?

Answer:
Tensor tympani

282. 1. Bitewing radiography is the main special text used to help


in diagnosis of proximal caries. The performance (accuracy) of
a diagnostic test like bitewing radiography can be expressed in terms of
sensitivity and specificity. Which of the following is a reasonable
summary of the diagnostic accuracy of bitewing radiography for proximal
caries diagnosis?

A. Moderate sensitivity and low specificity


B. Moderate sensitivity and moderate specificity
C. Moderate sensitivity and high specificity
D. High sensitivity and moderate specificity
E. High sensitivity and high specificity

Answer: E

283. You are interested in finding out what the risk indicators are for a
rare form of oral cancer and decide to undertake a study to examine this.
What type of study would be the most appropriate for addressing this
issue?

A. Cohort
B. Prevalence study
C. Clinical trial
D. Case-control study
E. Case-series

Answer: A

284.
Sex ratio for oral cancer is 2:3 or 1:2?

Answer:

In England and Wales Male: female ratio of oral cancer is 2:1 and in
Scotland is 3:1

285. What is mean mode n median

Mean: average

median is described as the numeric value separating the higher half of a


sample, a population, or a probability distribution, from the lower half.

Mode:
the mode is the value that occurs most frequently in a data set or
a probability distribution

286. What is least important for success of post?

A. diameter,
B. material,
C. luting agent,
D. length

Answer: C

287. Which type of cells is seen in the early stage of chronic


periodontitis?

Basophils, eosinophils, lymphocytes, neutrophils, macrophages

4.Which type of cells is seen in late stage of chronic


periodontitis?

5.Which type of cells is seen after early stage of chronic


periodontitis?

Answer:

the term chronic indicates presence of chronic inflammatory cells like


macrophages,monocytes n lymphocytes.

288. Cellularity of exocrine gland?

multi,
bi,
unicellular

Answer:
almost all the exocrine in human body are multicelluar except goblet
cells are unicellular

289. Type of epithelium lining exocrine gland?

Answer:
columnar or cuboidal secretory cells.
290. Which is NOT TRUE in relation to the prescription of 5mg or 10mg
of diazepam for*sedation*

A. Patient commonly complain of post operative headache


B. An acceptable level of anxiolytic action is obtained when the drug
is given one hour preoperatively
C. There is a profound amnesic action and no side affects
D. Active metabolites can give a level of sedation up to 8 hours
postoperatively
E. As Benzodiazepine the action can be reversed with Flumazepil

Answer:
C

Explanation: Diazepam has a range of side-effects that are common to


most benzodiazepines. Most common side-effects include:
• Suppression of REM sleep
• Impaired motor function
o Impaired coordination
o Impaired balance

o Dizzinessand nausea
• Depression [54]
• Reflex tachycardia[55]

Less commonly paradoxical side-effects can occur and include


nervousness, irritability, excitement, worsening of seizures, insomnia,
muscle cramps, changes in libido(increased or decreased libido)and in
some cases, rage, and violence. These adverse reactions are more
likely to occur in children, the elderly, individuals with a history of drug or
alcohol abuse and people with a history of aggression.
[4][56][57][58]
Diazepam may increase, in some people, the propensity toward self-
harming behaviours and, in extreme cases, may provoke suicidal
tendencies or acts.
[59]
Very rarely dystonia can occur.
[60]
Diazepam may impair the ability to drive vehicles or operate machinery.
The impairment is worsened by consumption of alcohol, because both
act as central nervous system depressants.
[33]
During the course of therapy, tolerance to the sedative effects usually
develops, but not to the anxiolytic and myorelaxant effects.
[61]
Patients with severe attacks of apnea during sleep may
suffer respiratory depression (hypoventilation) leading to respiratory
arrest and death. Diazepam in doses of 5 mg or more causes significant
deterioration in alertness performance combined with increased feelings
of sleepiness.
[62

291.

Which is the danger zone in dental surgery


Answer:

the danger zone of the face is the region of the face where
infections could pass to the cavernous sinus and to the brain
possibly via the
pterygoid plexus and deep facial vein
. This could occur when giving LA to
the posterior superior alveolar nerves.

This area is triangular and extends from the corners of the mouth to the
bridge of the nose, including the nose and maxilla.292. %ge of children
suffering from dental trauma

Answer:

8 year olds- 5%12 year olds- 11%15 year olds- 13%this is for trauma to
permanent teeth.

293. what is BMI( body mass index) for? How do we measure it?

Answer:

Its to check if someone is overweight/obese/ underweight.

It is calculated with height in meters and weight in kgs.

weight (kgs)/(height in metres) 2

20-25= normal weight for height,


26-30= overweight,

31-35= obese,

36-40 morbidly obese

BMI= 76 kgs/ 1.76*1.76 = 25

294. Antibiotic for chronic sinusitis

Answer:

.doxicycline

295. Depth of topical anesthesia

Answer:
2mm

296. Where bone is lost most in perio disease

Answer:
inter dental (approximal)

298. Suture material for lip trauma,oroantral fistula

Answer: black silk

299. Pulp tester, its readings and what they signify

Answer:

Exaggerated response to pulp testing is seen in reversible pulpitis and


no response or are duced response is seen in irreversible pulpitis.

300. Nerve fibers in pulpal pain

Answer:

Delta nerve fiber is responsible for sharp, stabbing pain of tooth


ache.Polymodal nerve fibers give rise to dull, aching and poorly
Localised.
301. what does the translucent/ sclerotic zone in dentin represent?

Hisopathological zone in the advancing edge of the carious lesion. The


four zones of dentin in order from the advancing edge are

1.zone of sclerosis
2.zone of demineralization
3.zone of bacterial invasion
4.zone of destruction

The four zones of enamel in order from the advancing edge are

1.zone of translucency
2 . d a r k z o n e
3.body lesion
4.surface zone

302. Medical condition in which patient starts using 3-4 pillows at night?

Answer:
left sided heart failure because they come down with breathlesness with
pulmonary edema.

303. A pat complaints of loss of taste sensation on the lateral border of


tongue after extraction of 3rd molar. which nerves would be damaged?

Answer:

lingual nerve

304. a nurse injures her finger with a needle prick. what is the first step
she needs to do?

Answer:

Encourage bleeding, wash hands under running water, do not scrub.

305. An old man with loose lower denture comes to u, making every yr a
new one.
will u make a new denture,
suggest implants,
wait for 3 months,
refer to specialist,
alveoloplasty?

Answer:
refer to specialist

306. Gingival pocket lining is formed from;

reduced E E,
hertwigs,
dental papilla

Answer:

REE

307. Where to discard a face mask?

Answer:
clinical waste

308. What’s the color of bin for special waste?

Answer:
309.

Do we give antibiotic prophylaxis before dental procedure if pat has


prosthetic heart valve?

Answer:
No antibiotics required anymore for routine dental procedures.

310. Membranes of expanded polytetraflourethylene have been


designed for periodontal regenerative techniques. Which of the following
defects will respond most predictably to regenerative therapy?

a shallow, wide 1-walled defects


b shallow 2- wall defect
c deep narrow 3- walled defect
d deep narrow 1- walled defect
e shallow narrow 2- walled defect

Answer: C

311. You examine and a patient and find bpe score of 4 in all sextants.
radiographs show generalised bone loss with minimum of 50% of bone
support remaining on all teeth. which of the following factors is important
in considering the prognosis of teeth
a age of pt
b oral hygiene status
c bleeding on probing score

d mobilty

e gingival recession.

Answer: B

312 you decide to refer an 8 yr old pt to your local oral surgery


department for extraction under General anaesthesia. what key reason
for asking for GA would you put in the referral letter

a Parents requests GA
b failed to complete treatment under inhalation sedation
c child wouldn’t accept la
d parent thinks hospital is more convenient.

Answer: D

313. pt presents with clinking in their TMJ joint, the clink is present
midway the opening cycle and is consistent. There is some pre auricular
pain and the lateral pterygoid mucscle is tender to resisted movement
test. There is no trismus and the click is absent when the pt closes
edege to edge instead of her normal occlusion the pt would like Rx. The
most appriopate occlusal splint is

a stabilization splint
b Localised occlusal interferences splint
c bite raiser
d soft bite guard
e anterior repositioned splint

Answer: D

314. The subodontoblastic plexus of Raschkow occurs:

a. Below the cell bodies of odontoblasts


b. In the root region of the pulp
c. Within the central pulp core
d. Within the cell-rich zone of Weil

Answer: A

315. Which of the following is NOT a major mechanism of action for


fluoride in caries inhibition?

a. Increases remineralization of enamel.


b. Inhibits carbohydrate metabolism
c. Reduces enamel solubility.
D. ability to prevent reduction of the pH of plaque

Answer:B

316. The best approach for diagnosis of odontogenic pain is which of the
following?

a. Radiographic examination
b. Percussion
c. Visual examination
d. A step-by-step, sequenced examination and testing approach

Answer: D

317. A patient with no positive history came along for scaling. The
moment you pick up the scaler you punch your finger, what should you
do?

A. Complete the procedure as nothing has happened


B. Check patient's blood for Hepatitis B antibody HBsAb
C. Check patient's blood for Hepatitis B antigen HBsAg
D. Check dentist's blood for Hepatitis B antibody HBsAb and HIV antigen
HIVAg
E. Check dentist's blood for Hepatitis B antigen HBsAg and HIV antibody
HIVAb
F. Dentist should go and take a HBsAb vaccine

Answer: A

318. What is least important for success of post?


A. diameter,
B. material,
C. luting agent,
D. length

Answer: C

319. which antibody is found in bacterial and viral infection?


Answer: IgG
level of IgA?
A) Saliva>serum>breastmilk>tears
b) serum>saliva>breastmilk>tears
c) saliva>breastmilk>tears>serum
Answer: B

Alternate answer :

SERUM>TEARS>SALIVA>BREAST MILK OR if to give in figures

SERUM--40-400mg/dl,
TEARS--60-85mg/dl,
SALIVA--6.2-14.5 mg/dl,
BREASTMILK--approx 0.708 mg/dl

320. Tumor that spread along the neural sheath of the submandibular or
parotid duct?

Answer:

Adenocystic carcinoma has tendency to spread along nerve pathways.


Appear on palate from minor salivary glands. H/F – cribiform or Swiss
cheese pattern.
321. What solution is used for alginate impression disinfection?

Answer: 2% gluteraldehyde with various trade names.

322. How often should you review the smoking pattern

a)6months
b)12months
c)24months
d)3months

Answer:A

323. What is the concentration of sodium hypochloride to clean a blood


splatter in the dental surgery?

Answer:
10,000ppm Sodium hypochlorite, cover it with a disposable cloth and
dispose it as clinical waste.

324. Which antibody is used for dental caries vaccine?is it IgA or IgG?

Answer: IgA

INHIBITS THE ADHERENCE of microbes and neutralizes toxins and


viruses.

325. what are the bones which grow by endochondral ossification?

Answer:

Mandibular condyle, Coronoid process, angle of the mandible all


undergoes endochondrial ossification. (long bones of the body)

326. Which is the most adhesive dental cement?

Answer:
glass ionomer cement is adhesive to both dentine and enamel.

327. Is it mercury that causes black staining in amalgam or silver?

Answer:
silver for the blackness
328. Impression taking in an apprehensive adult with gag reflex?

Answer:

For patients having gag reflex...inhalation anesthesia is the best option.


We can also use a lower tray for upper impression to reduce the contact
of impression material with the palate.

329. What we use instead of formcresol now in one visit pulpotomy..

Answer:

Ferric sulphate

330. Which one is more aggressive between Okc and ameloblastoma?

Answer:

Ameloblastoma is more aggressive in terms of tendency to


metastsize and how locally invasive the tumour is. I haven't read
anytext book where ameloblastoma and okc were compared.You
just have to look at the clinical features and decide for yourself.
It also depends on what you mean by aggressive. do you mean
recurrence rate, rate of local invasion or ability to metastasize. The
recurrence rate for okc is higher than that of ameloblastoma (11-
60%; soames and southam) but ameloblastoma has a higher
tendency to metastasize and I would say its local invasion of
bone is more than that of okc ( ameloblastoma causes
buccolingual expansion while okc spreads anterio- posteriorly but
can cause mesio buccal expansion as well; soames and southam,
cawson and odell).

331. What is the BULL rule??


Answer:
BULL rule as you said is for Buccal Upper and Lower
Lingual cusps, which
ares u p p o r t i n g c u s p s . T h e U p p e r p a l a t a l
a n d l o w e r b u c c a l c u s p s a r e f u n c t i o n a l cus
ps which bear most of the masticatory stresses.

332. patient with grade 2 mobility you did root planning? When should
we again recall patient?
1 week,
1 month
or
3 months?

Answer:
3 months

333. Writing 5 faults/defects in a try-in denture/wax- up......what could be


the possible causes of that?

Answer:
mid line shift

2.the molar relation not in class 1 on both sides


3.on one side the teeth may not be in arch, for example the lateral may
be palatally inclined
4.instaed of a molar a premolar is put
5.the central incisors are put other way like the right and left side
wrongly put.
6.there could be open bite on either side.

334. In a patient with chronic periodontitis with BPE score of 4 or *


how many times per year do we have to take radiographs?

Answer.

• Patients with perio disease not under good control - - iopa and/or
vertical bitewings of problem areas every 12-24 mnths n full mouth
every 3-5 yrs.

• Patients with perio disease under good control - - iopa and/or vertical
bitewings of problemareas every 24- 36 mnths n full mouth every 5 yrs.
• dental implants-- 6, 12 n 36 mnths after prosthetic replacement, then
every 36 mnths if thereis no clinical problem

• Maintenance pt -- full mouth if not avilable, if taken within 24 mnths


then only problematic areas....

335. An apical radiolucency (2mm) is noticed as an incidental


radiographic finding associated with the apex of the Mesiobuccal root of
the lower right first molar. The tooth has been root filled but is2mm short
of the radiographic apex. There are no other clinical or radiographic
findings and the patient is fit and well.

What is the most appropriate course of action?

A. Extract the tooth.


B. Redo the root filling
C. Perform periapical surgery.
D. Advise the patient of the situation and monitor clinically and
radiographically
E. Prescribe antibiotics then review

Answer.

D336. Which condition is relieved by sleeping on 3-4 pillows?

Answer: orthopnoea

Explnation:
Orthopnea or orthopnoea
(Greek from
ortho
, straight +
pnoia
, breath)

is shortness of breath(dyspnea) which occurs when lying flat,


[1] causing the person to have to sleep propped up in bed or sitting in a
chair. It is the opposite of platypnea.

It is commonly measured according to the number of pillows needed to


prop the patient up to enable breathing (Example: "3 pillow orthopnea").

Causes

Orthopnea is due to increased distribution of blood to the pulmonary


circulationwhile recumbent, but usually can be attributed to a more
fundamental cause. Orthopnea is often a symptom of left
ventricular heart failure and/or pulmonary edema.
[3][4]. It can also occur in those with asthma and chronic bronchitis, as
well as those with sleep apnea or panic disorder .
337. What is the minimal preparation cantilever and how is it different
from conventional cantilever.

Answer:
A cantilever is a bridge suspended from one end only. For eg, minimal
preparation usually i n v o l v e s n o o r m i n o r p a l a t a l t o o t h
reduction and use of adhesive cements (resembles a tooth
suspended off a wing). A conventional cantilever preparation
involves full crown prep and the tooth suspended off it.

338. What is the radiation exposure when taking an OPG

a 0.001mSv

B 0.01 mSv

c.0.01mSv

d. 1.0mSv

e. 10 mSv

Answer:
B and C

339. The use of rectangular collimator reduces the dose of radiation by


50% or 65%??

Answer: 65%

340. what is the optimal shape for collimator

a. square
b.rectangular
c. round
d. hexagonal
e. circular

Answer: b
341. what is the radiation dose in milli-Sieverts for a patient associated
with having an OPG??

A. 0.034
B 0.34
C 34
D 3.4
E 340

Answer: B

342.

Which cells are found in hepatic stomatitis

A, lymphcytes
B, monocytes
C,neutrophils
D, macrophages
E, plasma cells

Answer:

There are multinucleate giant cells present in herpes labialis


which closely relates to macrophages in the given options.

343. Which is the most common site for Mandible Fracture - Is it


the Condyle or Angle?

Answer:
condyle is the commonest site followed by angle...30% vs 25%

344. Radiograph for RCT follow up.

Answer:
Further follow-up radiographs s h o u l d b e t a k e n a t o n e
year and four years after completion of treatment – for

asymptomatic teeth.

Large periapical radiolucencies should be monitored more


frequently.
345. Dental survey 2009 results

The survey consisted of a questionnaire interview with all adults aged


over 16 years at all sampled households, and an oral examination of the
mouth and teeth of all those adults who had at least one natural tooth.
The sample size for the survey was
13,400 households (1,150 in each English Strategic Health Authority and
Wales, and 750 households in Northern Ireland)
. Data collection for the survey took place between October 2009 and
April 2010, and the final household interview response rate was
60 per cent.

A total of 11,380 individuals were interviewed, and 6,469 dentate adults


were examined
, making this the largest ever epidemiological survey of adult
dental health in the United Kingdom.
This report covers the following topics:

•Loss of all natural teeth

•Functional dentition - adults with 21 or more teeth

•Summary of tooth condition

Sound and untreated teeth

Restored, otherwise sound teeth

Decayed or unsound teeth

Decay on the crowns of the teeth

•Regular dental attendance

•Levels of dental anxiety.

Key facts

•Over the last 30 years the proportion of adults in


England who had no natural teeth
("edentate") has fallen by

22 percentage (points, from 28 per cent in 1978


to 6 per cent in2009.)

•In Wales
, the proportion of adults who were
edentate
has fallen 27 percentage points from37 per cent in 1978 to
10% in 2009.

• In
Northern Ireland,
the proportion of adults who were
edentate
has fallen 26 percentage points from 33 per cent in 1979 to
7% in 2009•

• Eighty-six per cent


of dentate adults had 21 or more natural teeth.

•The average number of teeth


among all dentate adults was 25.6.

•The average number of sound and untreated teeth was 17.8.


•Over three-fifths
(61 per cent) of dentate adults
said they attended the dentist for regular check-ups;

• Twelve per cent of all adults (who had ever been to the dentist) were
classified as having extreme dental anxiety.

The proportion of dentate adults with 21 or more natural teeth varied


by country.

In
England 86 per
cent of dentate adults had 21 or more natural teeth compared with
80 per cent of dentate adults in Wales.
Eighty-four per
cent of dentate adults in Northern Ireland had 21 or more natural teeth.

There was a clear socio-economic gradient in the proportion of adults


who had 21 or more natural teeth ranging from

91 per cent of adults from managerial and professional occupation


households to 79 per cent of adults from routine
and manual occupation households.

• The majority of dentate adults (71 per cent) had no visible coronal caries.

Over three-fifths of dentate adults said they attended the dentist for
regular check-ups.

• The proportion of adults attending the dentist for regular check-


ups increased in all three countries between 1978 and 2009 (1988 to
2009 in Northern Ireland5). The greatest increase was observed in
Wales, up 30 percentage points from 39 per cent in 1978 to 69
per cent in2009.Twelve per cent of all adults (who had ever been
to the dentist) were classified as having extreme dental anxiety.

• Extreme dental anxiety was more prevalent among women than men,
17 per cent compared with 8 per cent respectively.

• There was a clear pattern of higher levels of dental anxiety among


younger adults. Levels of e x t r e m e d e n t a l a n x i e t y r a n g e d f r o m
1 5 p e r c e n t o f a d u l t s a g e d 1 6 t o 2 4 t o 9 p e r c e n t o f adults
over 85.

346. Cancer- referral in how many days should be seen in secondary


care?

Answer:

within 2 weeks

347. Which of the following is the most important factor in the disease
progression in smokers?

A. Smokers have drier mouths than non-smokers


B. Smokers have poorer oral hygiene than non-smokers.
C. Nicotine will impair the chemotactic and phagocytic properties of
PMNs.
D. The gingival blood flow is reduced in smokers.
E. Smokers alter the oral environment encouraging the growth of
anaerobic bacteria

Answer: D

348.
What are notifiable disease and examples pls.

Answer:

notifiable diseases in England and Wales under public health act 1984,
public health regulations 1988

- acute encephalitis,
leptosprosis,
malaria,
measles,
meningitis,
meningococcal septicemia,
mumps,
ophthalmic neonatorum,
paratyphoid,
plague
acute poliomyelitis,
rabies,
relapsing fever,
rubella,

scarlet fever,
smallpox,
tetanus,
TB,
typhoid,
typhus,
viral hemorrhagic fever,
viral hepatitis,
anthrax,
whoophing cough.

Cholera
diphtheria
dysentery
food poisoning
leprosy
others like SARS, Avian flu.

349. Salivary gland secretions?

Answer:
Parotid - serous
Submandibular - mixed
Sublingual – mucous

350. The percentage of possible damage to the lingual and dental


inferior nerve in surgical extractions of wisdom tooth

Answer:

inferior alveolar nerve

- temporary 4-5 %
- permanent
0.2%

Lingual nerve

- temporary 0.2%
- permanent 2%

Check the Odell page 116, the blue chart there. They give all the
information about nerve damage and accidents post surgery

351. Abundant immunoglobulin?

Answer:
IgG is the most abundant in body followed by IgA
Immunoglobulins, quantitative

serum IgA 68 - 378 mg/dL


IgG 768 - 1632 mg/dL
IgM 60 - 263 mg/dL
IgE 10 - 180 IU/L
351. HUMAN TISSUE THAT HEALS WITHOUT SCARRING ?

Answer:
Gingiva

352. WHAT IS REQUIRED TO INCREASE THE EFFICIACY AND


DECREASE THE FATIGUE OF UNIVERSAL CURETTE?

Answer:
efficacy is increased by sharpness

353. what the maximum period imposed on a dentist on grounds of


fitness to practice?

Answer:
12 months

354. The 'SPLINT TIMES' for avulsion, intrusion, extrusion, lateral


luxation, root fractures alveolar bone fracture?

Concussion – Ressuarance and soft diet


Avulsion – 7- 14 days
Intrusion, extrusion – 1-2 weeks
Luxation – 2-3 weeks
alveolar bone fracture – 3-4 weeks
subluxation – 1-2 weeks
Middle 3rd root fracture – 4 weeks
cervical 1/3 root fracture – 4 months

355. WHICH SALIVARY GLAND TUMOUR OCCURRS BILATERALLY?

Answer:
Warthin's tumor is bilateral, but the two masses usually are at different
times. Warthin's tumor is highly unlikely to become malignant.

http://en.wikipedia.org/wiki/Warthin%27s_tumor

356. An apical radiolucency (2mm) is noticed as an incidental


radiographic finding associated with the apex of the mesiobuccal root of
the lower right first molar. The tooth has been root filled but is 2mm short
of the radiographic apex. There are no other clinical or radiographic
findings and the patient is fit and well. What is the most appropriate
course of action?

A. Extract the tooth.


B. Redo the root filling
C. Perform periapical surgery.
D. Advise the patient of the situation and monitor clinically and
radiographically
E. Prescribe antibiotics then review

Answer: D

357. What is the normal bleeding and clotting time?

Answer:
Normal value of clotting time is 5-8 minutes
normal bleeding - 2 - 9 minutes depending on the method used.

358. how many years are we supposed to tell the patient that
the following treatments last in average?

Answer:
Amalgam = class 1,2,& 4 .............. 5-25 years
[ median/ average 11 years ].

Anterior composite........................... 3-10 years


[ median/average 6 years ].

Direct posterior composite=...............3-9 years


[ median/average 5 years ].

Glass ionomer= class1,3 &5..............3-5 years


[ median/average 4 years]

Cast metal- precious/gold/noble metal=10-25 years


[ median/average 17 years].

PFM = .............................................10-25 years


[ median/average 18 years ]
359.
What Local anesthetic is:

Answer:
- used in children- Lignocaine
-used in cardiac arrythmias: Lignocaine without adrenaline
-neurotoxic at 4%., articaine

360. .During swallowing,

a) suprahyoid muscles relax


b) masseter contract
c)tongue touches the palate

A. a and b
B. a and c
C. b and c
D. none of the above
E. all of the above

Answer: D
Explanation:
http://en.wikipedia.org/wiki/Swallowing

361.
.Endogenous morphine like substance which can control pain is known as

A. Bradykinins
B. Endorphins
C.Prostaglandins
D.Serotonins
E.Enkephalins

Answer: B
Explanation: Endorphins
("
endo
genous mo
rphin
e") are endogenous opioid peptidesthat function as neurotransmitters. They
are produced by the pituitary glandand thehypothalamus
in vertebrates during
exercise,
excitement,
pain,
consumption of spicy food,
love
and
orgasm,

and they resemble the opiates in their abilities to produce analgesia and a
feeling of well being.

T h e t e r m " e n d o r p h i n " i m p l i e s a pharmacological


a c t i v i t y ( a n a l o g o u s t o t h e a c t i v i t y o f t h e corticosteroid category
of biochemicals) as opposed to a specific chemical formulation. It consistsof
two parts:
endo-
and
-orphin
; these are short forms of the words
endogenous
and
morphine,
intended to mean "a morphine-like substance originating from within the body

362.
What is the golden time for the tooth to be out of the socket.
Is it 30 min or 45 min.

Answer:
30 minutes

363. What is the taper of a standard GP cone ?

A) 0%
B) 1%
C) 2%
D) 5%
E) 10%
Answer: C

364. A 73 year old man taking fluoexetine for depression and now taking
misoprostol and diclofenac for arthritis

Answer:
Diahorrea

365. A 55 year old woman taking inhaled salbutamol for asthma, now
prescribed timolol eye dropsfor glaucoma

Answer:
dyspnoea and wheeze

366. A 70 year old man taking bendrofluazide and enalapril started on


doxazosin

Answer:
postural hypotension

367. A 23 year old woman with Hodgkins lymphoma, being treated with
vincristine and cyclophosphamide

Answer:
neutropenia

368. A 50 year old treated with oxybutynin for bladder over-activity and
dihydrocodeine for pain.

Answer:
constipation

369 .Which of the following is true regarding lathe cut silver alloy?

A. Has low creep.


B. Requires least amount of mercury.
C. Has tensile strength, both at 15 mins & 7 days comparable to high copper
Unicompositional alloys.
D. Achieves lowest compressive strength at 1 hour.

Answer:
A
370. Which antibiotic is safe to prescribe to patient on warfarin,?

Answer:
Warfarin's anticoagulant effect was
potentiated by 6 antibiotics
(cotrimoxazole, erythromycin,fluconazole, isoniazid, metronidazole, and
miconazole)Warfarin's anticoagulant effect was
inhibited by 3 antibiotics
(griseofulvin, rifampin, and nafcillin)

Safe to prescribe penicillin v (
penicillin or amoxicillin)

371. .Bilateral swellings not associated with food smell associated with
lymphadenopathy?

Answer:

372. How is acetyl choline released?

Answer:

373. You have recently read a paper in the Guardian about the best treatment for
oral candidosis. This you have decided to use, as the new colleagues have been
using. Which of the following methods will you choose to evaluate the results.

a. Mean
b. Median
c. Standard deviation
d. Standard reduction

Answer: c

374. In which of the following case you will require post-op antibiotics?

a. Radiotherapy for head and neck region


b. Valvular heart disease
c. Pacemaker

Answer: None
375. Parts of the audit.Which is not one of them?Options:
a. Collect control group
b. Implement change
c. Compare set standards with clinic standards
d. Re audit

Answer: A

376. Which of the following is the best method of clinical evidence?

a. Case control
b. Cohort
c. Systematic review
d. RCT (Randomised control trial)

Answer: C

377.
Reason for tingling of hands and feet during treatment of long standing
procedures ?

Answer:
shortage of blood to the feet or any nerve damage, lyme disease or tarsal tunnel
syndrome causes tingling and numbness in the feet

65Person with swollen salivary glands+swollen lips+ swollen


gingiva…diagnosis

Answer: sarcoidosis

378. Photo of two canines completely abraded/attrited with gingival recession


and cerv. abrasion
- Give 1 advantage of having these two teeth alone

- Give 2 disadvantages to having these two teeth alone

- T/t options for this patient other than RPD

- If overdentures are given the how would you obtain stability for this
denture- 2 ways

- 4 advantages of overdentures for this patient.


Answer:

379. Fracture (mid-palatal) of denture

- 4 patient related factors leading to it


- 4 dentist related factors leading to it
- Give 2 ways of overcoming this fracture

Answer:

380. Picture of 14 yr old girl with erosion of Buccal surfaces of molar

- 3 types of non carious tooth loss


- What do you think this is
- What disorder would you expect in this
- 4 ways in which you will monitor tooth wear
- What is the most important advice that you will give in this?

Answer:

A.3 non carious lesions:abrasion,attrition,erosion


b. it is bullimia or anorexia nervosa-since they have clearly mentioned
that it is a disorder
c.4 ways to monitor tooth wear:study models, photos, clinical picture and
history, tooth wear indices
d.Management:

Referral to medical care, psychosocial interventions, nutritional


counseling

381.Which fibers are most resistant to periodontitis?

a. Transeptal fibers
b. Intergingival
c. Circular
d. Dentogingival

Answer: A

382. If the root filled tooth has no symptoms. When do you take the first
xray for the follow up. Is it6,12 or 18monts?

Answer:
12 months
383. A dentist is carrying out electro surgery on a patient and patient
sneezes with his head moving forward. Which is most likely to occur?
a GINGIVAL TRAUMA
b mucosal burn
c mucosal trauma
d. trauma to adjacent tooth

e. gingival laceration

Answer: c

384.
Bilateral swellings not associated with food smell associated with
lymphadenopathy?

Answer:
Sarcoidosis

385. How is acetyl choline released?

Answer:
exocytosis

386. What age the sphenoid and mastoid fontanels are closed?

Answer:

• The posterior fontanel usually closes about two months after birth;

•s p h e n o i d f o n t a n e l c l o s e s a t a b o u t t h r e e m o n t h s , t h e
m a s t o i d f o n t a n e l c l o s e s n e a r the end of the first year,

•A n t e r i o r o n e m a y n o t
c l o s e u n t i l t h e m i d
d l e or end of the second year.
387. .A dentist is using soflex disc on an upper molar and lacks finger
support. What is most likelyto occur?

a GINGIVAL TRAUMA
b mucosal burn
c mucosal trauma
d taruma to adjacent toothe gingival laceration

Answer:

388. What hormone increases blood glucose and decreases potassium?

Answer:

Cortisol

389. What antibiotic is used to manage a super infected herpetic lip


lesion?
a.Ciprofloxacin
b.Metronidazole
c.Cefuroxime

Answer:
c

390. Most abundant immunoglobulin in saliva during infections?

Answer:
IgA

391. Consent is needed from a patient to share information, which of the


following needs explicit consent?

a.To share information with an insurance company


b.To share information with other doctors in the practice
c.To share information with the therapist treating the patient
d.To share information with the patients GMP
Answer:
A

392. Which immunoglobin will increase in gingival inflammation?

Answer:

IgG

394. Which immunoglobulin increase in periodontitis?

Answer:

IgG

395. These phonetics are in relation to complete denture phonetics.

Answer:


Mechanism of speech

The voice is produced in the larynx: the muscles of the thorax and
abdomen control theflow of the air with nasal cavity act as resonant
chamber

The air from the larynx divided into 2 streams by the velum:a) Upper
stream: the air expelled entirely through the nose to produce the nasal
sounds: N-M-Ng. b) Lower stream: the air expelled through the oral
cavity and altered by the palate, tongueand position of the teeth and lips
to produce all other sounds.

Types of sound:I. Vowels:


Produced by vibration of the vocal cords and not affected by oral
structures. The tongue is positioned in the floor of the mouth and contact
lingual surface of anterior teeth.

Types: Vowels are: a-e-i-o-u.
II. Consonants:

Produced by constriction, obstruction and direction of the air stream
when the air passthrough the mouth

Types:

a) According to the manner of production:


1. Nasal sounds: N-M-Ng

Produced through the nose. When the nasal cavity is blocked (adenoid
hypertrophy-deviated nasal septum), hypernasality occurs.2. Plosives
sounds: P-B-T-D-K

Produced by complete stop of air stream, build up of pressure in the oral
cavity thensudden release and explosion of air 3. Sibilant (fricative)
sounds: S-Ch-H-X-Z

Produced by friction of the air stream when forced through narrow path
way
b) According to the site of production:
1. Bilabial sounds: B-P-M

Formed by lip only. The air from the lung builds up pressure behind the
closed lip,


Too high occlusal plane: tongue spread on the lower teeth→ lisping (S
pronounced Ch), and F pronounced V

Too low occlusal plane: difficulty in correct positioning of the lower lip
and tongue contact occlusal surfaceduring the speech → V pronounced
F2- Vertical dimension

Increased vertical dimension: denture teeth make contact during
speech→ clicking, defect in Ch-C-J sounds,whistling, Th pronounced T
due to failure of the tongue to be placed between anterior teeth

Decreased vertical dimension: leading to lisping (S pronounced Ch)

M sound: used as an aid to obtain correct vertical dimension. When the
patient say M, if the lips arestraightened and unable to make contact, the
record blocks are occluded prematurely and the VD is high

S sound: also used as an aid to obtain correct vertical dimension. When
the patient say S (sixty-six), theupper and lower teeth should be
separated 2mm from each other (closest speaking space method)3-
Teeth arrangements:1- Width of the dental arch:

Too narrow dental arches→ the tongue cramped and the size of air
channel decreased → faulty articulationof consonants (T-D-N-K-C),
therefore, the teeth should be placed in the position previously occupied
by naturalteeth2- Antro-posterior position of the anterior teeth

Upper anterior teeth

Too far palatally:- Upper incisors difficult to contact the upper lip → affect
labiodentals sounds (F-V)- Tongue make contact with the teeth
prematurely → affect lingupalatal sounds→ lisping (S pronounced
Ch), Tpronounced D

Too far labially: whistling and D pronounced T

Lower anterior teeth:

Too far lingually: Th pronounced T and the tongue rested in the floor
of the mouth behind lower anterior teethin pronunciation of vowels

Too far labially: affect pronunciation of vowels.3- The relationship of
upper and lower anterior teeth

Abnormal protrusive or retrusive Jaw relations (class II, class III angle
classification) associated with increaseor decrease the overjet leading to
difficulty in pronunciation of S sound (increase overjet→ whistling)

396.which of the following gypsum products is commonly used for


pouring up the primary dental impressions......
a.TYPE I
b.TYPE II
c.TYPE III
d.TYPE IV
e.TYPE V
Answer: B

397.
Most reliable test to indicate the presence of Active Hepatitis -
a. HBs antigen
b. HBc antigen
c. HBe antigen
d. Antibody to HBs antigen

Answer: C

398
-What vessels are the capacitance vessels and hold most of the blood
volume?

a.Arteries
b.Arterioles
c.Capillaries
d.Veinules
e.Veins

Answer: E

399.
What enzyme is found in dentinal caries?
Lactoferrin,
collagenase,
or enolase?

Answer:
enolase

400. Blood supply interferes in salivation. How?


Sympathetic,
parasymphatetic, etc
(Type of stimulation on blood vessels in salivary glands)

Answer:

increase in salivary glands secretion is a result of parasympathetic


and sympathetic stimuli. Sympathetic nervous system affects gland
secretions inversely by innervating blood vessels that supply gland.
401. Lots of neurophysiology (even alpha, beta, gamma, delta
something), do you know what they mean?

Answer:
alpha, beta, Gamma, DELTA,B,C are nerve fibres which are myelinated
except C which is non myelinated.

402. What’s increased in elderly? Systole, diastole, stroke volume, etc

Answer: Diastole

403. Function of intermediate zone of adrenal gland

Answer:
Glucocorticoids (Cortisol) secretion

404. How you describe saliva in relation to plasma?


-options, hypotonic, isotonic, hypermolar?

Answer: Hypotonic

405. physiology in cardiac arrest, angina??/

Answer:

angina and MI are common causes of IHD


Angina pectoris is caused by an imbalance b/w myocardial
oxygen requirements and myocardial oxygen
supply. pain of angina is due to metabolic changes in ischemic myocardi
um. chemicals like adenosine, histamine release 4rm ischemic cells
which act on intra cardiac sympthateic nerves which goes 2 cardiac
plexus and symp ganglia. impulses then transmitted through spinal
chord to thalamus and cerebral cortex. myocardial oxygen req.
increases as workload of heart inc. if coronary arteries
are unable 2 deliver required O2, myocardial ischemia happens inc.
leading 2 MI.

406. Irreversible shock

407. what increases pulse rate during sex?


Answer: Adrenaline
408. What neurotransmitter is responsible for closing pre capillary
sphincters?

a.Acetylcholine
b.Serotonin
c.Substance P
d.Nor-adrenaline

Answer:
D

409. What hormone increases blood glucose and reduce potassium?

Answer: Cortisol

410. .best restoration in a missing incisor in a healthy caries free mouth?

Answer: Implant

411. Radiation burns in cheek, what will u find intraorally?

Answer:
Mucositis

412. Antibiotic prophylaxis required in which case?

Answer: renal dialysis, immune compromised case.

413. antibiotic given in chronic sinusitis .

Answer:
Amoxicillin/
erythromycin/
cephalosporin plus metronidazole

414. which drug u give to a Patient coming 2 days after extraction of 3rd
molar with 2 extraoraldraining sinuses?

Answer: Metronidazole 400mg 3 time for 5 days

415. What s the nerve block for upper 4?

Answer:
Infra-orbital nerve block

416. What cement for cementing temporary bridge?

Answer:
Temp bond...ie zinc oxide eugenol

417. What s the best restoration for a peg shaped lateral?

Answer: composite

418. What should be added to beta lactams to be more efficient against


anaerobes?

419. what does fracture of the 2 condyles cause?

Answer: Anterior open bite

420. What s the 2ry impression material for resorbed ridge?

Answer:

Zinc oxide eugenol paste

421. What material should be used in a lingual cavity in a patient with


Parkinson's disease?

Answer: GIC

422. What anaesthesia should be given in a flap for upper lateral


Apicectomy?

Answer: infraorbital nerve

423.
.You arrive at a new practice and notice that
a l m o s t e v e r y r a d i o g r a p h i n t h e patient’s notes has turn
ed brown. Your nurse confirms that this is a widespread problem that no-
one has ever remedied and she also remarks that the films tend to get
browner with age. What corrective action will you take to remedy the
problem?
A. Develop the films for the correct period of time
B. Heat the developer to a higher temperature
C. Ensure films are fixed for the correct period of time
D. Wash the films properly after fixing
E. Change the solutions more regularly

answer:
D

424. A 10½ year old boy with an uncrowded sound dentition attends
your practice.His permanent canines are unerupted and not palpable
and primary canines are retained. On radiographic examination, you find
the canines are mesially inclined and in the line of the arch. What is the
most appropriate management option?

A. Arrange surgical removal of permanent canines.


B. Extract primary canines.
C. Extract upper first premolars.
D. Grind primary canines.
E. Monitor occlusal development.

answer: E

425.
What gives the black stein to amalgam tattoo? Silver or mercury?

Answer: Silver

426.
A patient says that he does not like the appearance of his previously root
filled upper central incisor tooth. His dentition is otherwise well
maintained and his periodontal health is good. The tooth appears to be
darker than the adjacent teeth. What is the most appropriate approach
to treatment?
A. Provision of a post crown
B. Provision of an all ceramic crown
C. Provision of a metal bonded to ceramic crown
D. Carry out a non vital bleaching procedure

E. Provision of a porcelain veneer


Answer: D

427. A 35 year old male patient who admits to grinding his teeth at night
has a number of wedge-shaped cervical (Class V) lesions on his upper
premolar teeth. These are causing some sensitivity and are
approximately 3mm deep. What is the correct management option?

A. Provide tooth brushing instruction and fluoride


B. Restore the lesions with compomer
C. Restore the lesions with micro-filled composite
D. Restore the lesions with a hybrid composite
E. Restore the lesions with conventional glass-ionomer

Answer: D

428. You are trying in a partial chrome denture framework which fails to
seat properly. It fits the master cast. What is the most likely cause of this
problem?

A. Insufficient expansion of the investment material


B. Distortion of the impression
C. Contraction of the metal framework during casting
D. Failure to block out unwanted undercuts
E. Complex denture design

Answer: b

429. Bitewing radiography is the main special text used to help in


diagnosis of proximal caries. The performance (accuracy) of a
diagnostic test like bite wing radiography can be expressed in terms of
sensitivity and specificity. Which of the following is a reasonable
summary of the diagnostic accuracy of bitewing radiography for
proximal caries diagnosis?

A. Moderate sensitivity and low specificity


B. Moderate sensitivity and moderate specificity
C. Moderate sensitivity and high specificity
D. High sensitivity and moderate specificity
E. High sensitivity and high specificity

Answer: E
430.
Types of cells that proliferate in later stages of life
a Odontoblast
b Cementoblast
c Rest of Malassez
d Undifferentiaied Mesenchymal Cells

Answer: D

431.

Dentist did RCT for child patient, now he does Crown for the same
patient. What band the dentist should claim?

a Band 1
b Band 2
c Band 3
d No Claim

Answer: C

432.
A patient with a removable partial denture is dissatisfied with the false
appearance of the mandibular anterior teeth. The dentist could
CORRECT this appearance by

I.Moving the teeth farther lingually so that they are not so obvious
II.Varying the inclinations of the incisors so that alternate teeth appear
tilted
III.Moving the teeth farther facially so that their appearance is enhanced
IV.Separating the teeth slightly to make each one look distinct

Choose one answer.


a. III only
b. Any of these options
c. II, III
d. II, IV
e. I, III

Answer: E
433. A PRIMARY consideration for full coverage of abutment teeth to be
used in the construction of a fixed partial denture is the:

Choose one answer.

a. Health of the mucous membrane


b. Caries susceptibility of the patient
c. Amount of supporting bone remaining
d. Health of the gingival tissues
e. None of these options

Answer: E

434. What is the % of de oxygenated blood coming from lungs?

Answer: 75%

Explanation: Oxygen transport

Basic hemoglobin saturation curve. It is moved to the right in higher


acidity (more dissolved carbondioxide) and to the left in lower acidity
(less dissolved carbon dioxide)

About 98.5% of the oxygen in a sample of arterial blood in a


healthy human breathing air at sea-level pressure is chemically
combined with the Hgb. About 1.5% is physically dissolved in the
other blood liquids and not connected to Hgb. The hemoglobin
molecule is the primary transporter of o x y g e n i n mammals a n d
many other species (for exceptions, see below).

H e m o g l o b i n h a s a n oxygen binding capacity of between 1.36 and


1.37 ml O2 per gram Hemoglobin,
which increases the total blood oxygen capacity seventy fold,
[15] compared to if oxygen solely was carried by its solubility of
0.03 mL O2 per liter blood per mmHg partial pressureof oxygen
(approximately 100mmHg in arteries).
[15]
With the exception of pulmonary and umbilical arteries and their
corresponding veins, arteries carry oxygenated blood away from the
heart and deliver it to the body via arterioles and capillaries, where the
oxygen is consumed; afterwards, venules, and veins carry
deoxygenated blood back to the heart. Under normal conditions in
adult humans at rest; hemoglobin in blood leaving the lungs is
about98–99% saturated with oxygen, achieving an
oxygen delivery
of between 950 - 1150 mL/min
[16]
tothe body.

In a healthy adult at rest, oxygen consumption is approximately


200 - 250 mL/min and deoxygenated blood returning to the lungs is still
approximately 75% (70 to 78%) saturated. Increased
oxygen consumption during sustained exercise reduces the
oxygen saturation
of venous blood, which can reach less than 15% in a trained athlete; alt
hough breathing rate and blood flowincrease to compensate, oxygen
saturation in arterial blood can drop to 95% or less under these
conditions.
[19]
Oxygen saturation this low is considered dangerous in an
i n d i v i d u a l a t r e s t ( f o r instance, during surgery under
anaesthesia. Sustained hypoxia (oxygenation of less than 90%),
is dangerous to health, and severe hypoxia (saturations of less than
30%) may be rapidly fatal.
[20]
A fetus, receiving oxygen via the placenta, is exposed to much lower
oxygen pressures (about 21%of the level found in an adult's lungs), and,
so, fetuses produce another form of haemoglobin with a much higher
affinity for oxygen (hemoglobin F) in order to function under these
conditions.

[21]

Carbon dioxide transport


When blood flows through capillaries, carbon dioxide diffuses from the
tissues into the blood. Some carbon dioxide is dissolved in the blood. A
part of CO2
reacts with hemoglobin and other proteins to form carbamino
compounds. The remaining carbon dioxide is converted to
bicarbonate and hydrogen ions through the action of RBC carbonic
anhydrase. Most carbon dioxide is transported through the blood in the
form of bicarbonate ions. Carbon dioxide(CO2) , t h e m a i n cellular
waste product i s c a r r i e d i n b l o o d m a i n l y d i s s o l v e d i n plasma,
in equilibrium with bicarbonate(HCO3-) and carbonic acid (H2CO3).
86–90% of CO2 in the body is converted into carbonic acid, w h i c h
c a n q u i c k l y t u r n i n t o b i c a r b o n a t e , t h e c h e m i c a l equilibrium
being important in the pH buffering of plasma.
[22]
Blood pH is kept in a narrow range(pH between 7.35 and 7.45).
[9]
435. How would you treat a person with TB, HIV, hepatitis , MRSA

•send them to the hospital

•use single use instruments

•treat with sterile instruments

•treat using standard condition for infection control

Answer:
treat using standard condition for infection control

436. .Type of Hepatitis that doesn’t become chronic – Hep A?

Answer: Hep A and E

437. Which fluoride varnish aggravates effects of ANUG

Answer:

438. Which Antifungal should not be given with Diazepam

Answer:

fluconazole

439. Upper missing Laterals and Pointed Canines, gaps between


centrals and canines. What would be the best Rx options

a Orthodontically move the Canines


b Give bridgesc Crowns on Canines

Answer:

440. Upper denture loose. A root beneath the lower CD. Which X-ray will
you take?

Answer:

Periapical
441. Condition most commonly affected by Dry Socket

a. Smoker
b. Pt. On warfarin
c. Elderly

Answer: A

442.

QUESTION 1

A. COPPER
B. PALLADIUM
C. PLATINUM
D. SILVER
E. ZINC

CHOOSE FROM THE ABOVE FOR THE FOLLOWING:

1. INCREASED TARNISH RESISTANCE

2. SCAVENGER, PREVENTS OXODATION OF OTHER METALS

3. INCREASED STRENGTH AND HARDNESS

4. DECREASED DENSITY AND MELTING POINT

5. INCREASED DENSITY AND MELTING POINT

6. INCREASED CORROSION7. INCREASED POROSITY

Answer: Copper increases strength and hardness. Palladium increased


density and melting point. Silver increase tarnish resistance/ Corrosion
resistance platinum is added as scavenger Zinc is used at times for the
same purpose of scavenging

443. The most important feature to differentiate between an upper


neuron motor lesion and a lower neuron motor lesion is:
a- eye involvement
b- ear involvement
c- forehead involvement
d- anaesthesia of the facial nerve

Answer: C

Explanation:

it is because in upper motor neuron lesions, there’s no affect on


voluntary control of muscles of forehead due 2 bilateral innervations
of portion of motor nucleus of cranial nerve vii that innervates upper
muscle of facial expression.

444.

What are D1 D2 D3 lesions?

Answer:

That’s the WHO classification System:


The caries lesion shape and depth is scored in a 4 pointsscale.

D1: Clinically detectable enamel lesions with intact (non-cavitated)


surfaces.
D2: Clinically detectable cavity limited to enamel
D3: Clinically detectable cavity involving dentin
D4: Lesions extending into pulp

445. A PARKINSON DISEASE PATIENT COMES TO YOUR SURGERY


AND WITH WHATMATERIAL WILL YOU RESTORE A LINGUAL
CAVITY IN HIM?

Answer:
Parkinson dis : glass ionomer as they suffer from dry mouth and flouride
releases from GIC

446. PARAESTHESIA OF LINGUAL N. LASTS FOR HOW


MANY MONTHS?

Answer: 2-3 months


447.

Which part of the cranium is considered as the most stable area,

A. Frankfort plane
B. Occlusal plane
C. Anterior cranial plane
D.Anterior nasal to gnathion

Answer:

448. .A 9 yr old child requires extraction of their upper right first


permanent molar under local anaesthesia. Her medical history is
unremarkable except that she has had rheumatic fever at 3 yrs of age
and a chest infection 4 months ago, which was treated with penicillin.
What is the correct precaution for this child?

Answer: Routine extraction with no antibiotics required

449. Appointment for asthmatic patient should be given in:

1. Morning.
2. Afternoon.
3. Evening.
4. Any time.

Answer:
Afternoon.

450. Chi square test is:

1. Measures qualitative data.


2. Measures both qualitative and quantitative data.
3. Measures the qualitative data between two proportions.
4. Measures the quantitative data between two proportions

Answer: 4

Chi-square is a statistical test commonly used to compare observed data


with data we would expect t o o b t a i n a c c o r d i n g t o a s p e c i f i c
hypothesis. For example, if, according to Mendel's laws,
y o u expected 10 of 20 offspring from a cross to be male and the
actual observed number was 8 males, then you might want to know
about the "goodness to fit" between the observed and expected. Were
the deviations (differences between observed and expected) the result
of chance, or were they due to o t h e r f a c t o r s . H o w m u c h
deviation can occur before you, the investigator, must
c o n c l u d e t h a t something other than chance is at work, causing the
observed to differ from the expected. The chi-square test is always
testing what scientists call the
null hypothesis,
which states that there is no significant difference between the
expected and observed result.

451. Radio graphically lingual developmental groove is seen as:

1. Blurring of root canal.


2. Small pulp chamber size and constriction of root canal compare
to contralateral tooth.
3. Constriction of pulp canal only.
4. Large pulp chamber size and constriction compare to ipsilateral tooth.

Answer:

452.

1. A 15 year old girl in boarding school comes to your surgery for an


extraction which of the following cannot give consent on her behalf?

a. Her grandfather with legal guardianship

b. Her biological father who has separated from her mother

c. The girl herself

d. Her older sister

Answer: C
453. Consent is needed from a patient to share information, which of the
following needs explicit consent?

a. To share information with an insurance company


b. To share information with other doctors in the practice
c. To share information with the therapist treating the patient
d. To share information with the patients GMP

Answer: A

454.

EMQ about muscle fibres’

-a. what is the A band made of - Myosin


b. what combines with calcium- Troponin
c. what protein has I,T and C subcomponents.- Troponin

455. What enzyme assists microorganisms when they are causing


dentine caries?

a.Collagenase

b.Enolasec.

c. Lactoferrin

Answer: C

456.
What neurotransmitter is responsible for closing pre capillary sphincters?

a. Acetylcholine
b.Serotonin
c.Nor-adrenaline
d.Substance p

Answer: C
457.

What antibiotic is used to manage a super infected herpetic lip lesion?

a.Ciprofloxacin
b.Metronidazole
c.Cefuroxime

Answer:C

458. What component of amalgam gives strength


-a. Copper
b. Silver

Answer:
A

459.

Picture of the tongue with an ulcer on the lateral border

- what drug causes it

a. ACE- inhibitors
b. Calcium channel blockers
c. Potassium sparing drugs

Answer:
A

460. 1. Fluoride treatment for a child with high caries rate

a. 2,800 ppm Tooth paste


b. 5,000 ppm toothpaste
c. Application of 2.2% duraphat 2 time yearly
d.Application of 2.2% duraphat 3-4 times yearly

Answer:
D

461. Average number of 15 year olds who have lost 6-7 teeth

a. 1.1
b. 3.1
c. 4.7

(Look at current statistics)

Answer:

462. A man who smokes 2 packets of cigarettes a day and drinks.


Would have an increase risk of developing oral cancer of

a. 11 times
b. 13 times
c. 47 times

Answer:
C

463.

A picture of the gingival of a black person with melanin pigmentation


and a white line across the central incisors alone

1. What is the cause of the white line

a. Amelogenesis imperfecta
b.Dentinogenesis imperfecta
c.Hypo mineralization
d.Flourosis

Answer:
C

464. 64-year-old patient with a past medical history of metastatic breast


cancer presents to your surgery complaining of loose teeth, dental pain
and swollen gums.
A. What appearance is shown in the above X-ray?
B.What is the most likely causative agent?
C.How would you manage this patient?

Answer:

A. Osteonecrosis of the left mandible.


B. Long-term use of bisphosphonates to treat hypercalcaemia
secondary to metastastic disease.
C.Polite introduction. Take a history. Ask about trauma. Take detailed
personal past medicalhistory (PMH) regarding breast cancer diagnosis
and subsequent management and length of bisphosphonate use.

Diagnosis history:

• examination further investigations – CT scan, inflammatory markers

• treatment iv antibiotics liaise with oncology team concerning


bisphosphonate cessation.
465.
which of the following is derived entirely from meckel cartilage ..?

incus , malleus. stapes,mandible.Answer: Malleus

Explnation:
Meckel's cartilage
Meckel's cartilage forms in themesoderm of the mandibular process
and eventually regresses to f o r m t h e incus and malleus o f
t h e middle ear ; the anterior ligament of the malleus and
t h e sphenomandibular ligament. T h e mandible or lower jaw forms by
perichondral ossification using M e c k e l ' s c a r t i l a g e a s a
'template', but the mandible does
not
arise from direct ossification of Meckel's cartilage.

466. Which antibiotic causes abdominal cramps in therapeutic doses?

Answer: clindamycin

467. Which antibiotic causes abdominal cramps in toxic doses?

Answer: cephalosporins

468.
1-carious half way through enamel on proximal surface - bitewing
2-carious half way through enamel on occlusal surface – visual exam
3-carious half way through dentine not cavitated - wet surface,
4- stained fissure – bite wing and visual exam
5-hidden/occlut caries – bitewing and beam aiming devices

choice (traslumination, bitewing, dry surface, wet surface, briuat prop


wilames prob, stright sharp prob

469.

1. the cells in cementum that line its boundary with PDL with cytoplasmic
processes directed towards cementum?

2. cells that line the cemental boader with cytoplasmic processes


directed towards PDL ?
3) pt. came from holiday and have diarrhea and vomiting and claim that
he has eaten somethingfrom street? Hep A
4 )which condition that phagocyte kill bacteria is more difficult(tbc etc.
)6)severly disabled pt sedation – GA

471.

1. % of edentulous pts in England and Scotland

2. %of people with decay b/w 10-12 yrs old in 2003

4.Antibiotic used to manage superinfected herpetic lip cefuroxime or


metronidazole?

5.%of bacterial load that will decrease by cleaning

6.condition that prevents to maintain oral hygiene


Answer: down syndrome

7. % of children getting cavities after pit nd fissure sealant application

8.what to do when a pt comes with acute pain the very next day he had
been discharged after myocardial infarction

9. Three weeks ulcer present after denture insertion what to do? Trim
and observe or refer

10. Maternity leave allowed – 6 weeks

472. suture material used on lips?

Answer: prolene, nylon....

2.suture material for use in oroantral fistula?

Answer: vicryl

473. Which ion acts as a second messenger?

Asnwer; Calcium.
474. most common reason of failure of a proximal

amalgam

C A U S
E E F F
E C T
CAUSE
Too shallow cavity
Too thick cavity liner
Too thick cement base

EFFECT
Amalgam thin
--------------------------------------------------------
CAUSE

Inadequate cuspal reduction


Giving cavosurface bevel

EFFECT
Amalgam thin (over the cusp)
---------------------------------------------------------
CAUSE
Sharp axio-pulpal line angle
Sharp angles in occlusal outline form of Class II

EFFECT
Stress concentration (fracture thro'isthmus)
----------------------------------------------------------
CAUSE
Sloping gingival step
Too narrow gingival step
EFFECT
No resistance form (fracture thro' isthmus)
--------------------------------------------------------------
CAUSE
Insufficient Hg
Excess Hg
Under trituration

EFFECT
Amalgam weak
--------------------------------------------------------------
CAUSE
Moisture contamination (Zn havingalloys)

EFFECT
Delayed expansion
--------------------------------------------------------------
CAUSE
insufficient condensation pressure
Not squeezing out excess Hg

EFFECT
Increased residual Hg
--------------------------------------------------------------
CAUSE
Mix squeezed too dry
Condensation of partially crystallized amalgam

EFFECT
Lack of cohesion (amalgam weak)
--------------------------------------------------------------
CAUSE
Overfilling

EFFECT
Thin amalgam over margins
--------------------------------------------------------------
CAUSE
Failure to warn patient not to chew on the restoration for first few hours
Failure to support proximal part of the restoration while removing matrix
band
EFFECT
Early strength not high enough
----------------------------------------------------

----------------------------------------------------
TOOTH FRACTURE
----------------------------------------------------
CAUSE
Sharp angles in occlusal outline of ClassII

EFFECT
Stress concentration
----------------------------------------------------
CAUSE
Excess removal of tooth structure

EFFECT
Enamel undermined and tooth weakened
---------------------------
EFFEC
T

TOOTH DISCOLOURATION
Sometimes excess Hg within the restoration may seep through the
dentinal tubules, discolour dentin and result in blackish or greyish
staining of teeth. Since enamel is semi-translucent, this discolouration is
not inconspicuous.

475. Who checks if there is written protocol put up in the clinic for
radiation protection

Answer:

written protocol set by legal person monitored by radiation protection


supervisor

476. Whom to contact if there is 20s of extra radiation dose

Answer

. Radiation protection advisor

477. Most common reason of failure of approximal amalgam filling?


Answer

Isthmus fracture

478. what is more likely to happen to an interproximal composite filling


rather than amalgam it's feature or under contouring?

Answer

Polymerization shrinkage

479. Largest possible cavosurface angle?

Answer

110 degree

480. What cement you use to glue alumina core ceramic crown?

it's panavia or resin modified gic?

Answer
. Resin modified gic

481. Pigmented fissure, which is the best way of caries diagnosis?


Answer

. Investigate the area with round bur

483. What is the sensory nerve supply to temporalis muscle?

Answer

The Sensory and motor are both supply by the V3 , (Sensory by


Auriculotemporal branchof V3 and motor by ant. and post. deep
temporal branch of V3).

484. What’s the percentage of >25 years with periodontitis in UK

Answer
64%
485. What’s the percentage of tooth erosion in 6 years-old and 15 years-
old in UK?

Answer

5-52% and 15--27%

486.
Do you know what enzyme assists micro-organisms in dentin caries
process?
I'm in doubt between Enolase and Collagenase
.
Answer
. Enolase

Enolase
, a l s o k n o w n a s p h o s p h o p y r u v a t e h y d r a t a s e , i s a metall
oenzyme

r e s p o n s i b l e f o r t h e catalysis of the conversion of

2-phosphoglycerate

(2-PG) to phosphoenolpyruvate

(PEP), the ninth and penultimate step of

glycolysis

Enolase belongs to the class Lyase

Enolase can also catalyze thereverse reaction, depending on


environmental concentrations of substrates.
[3]

The optimum pH for t h i s e n z y m e i s 6 . 5 .


[4]

Enolase is present in all tissues and organisms capa


b l e o f g l y c o l y s i s or fermentation. The enzyme was discovered
by Lohmann andMeyerhof in 1934, and has since been isolated
from a variety of sources including human muscle and erythrocytes

Fluorideis a known competitor of enolase’s substrate 2-PG. The fluoride


is part of a complex with
magnesium and phosphate, which binds in the ac
tive site instead of 2-PG.
[4]
A s s u c h , drinking fluoridated
water provides fluoride at a level that inhibits oral bacteria enolase activ
itywithout harming humans. Disruption of the bacteria’s glycolytic
pathway - and, thus, its normal metabolic functioning - prevents
dental caries from forming.

http://en.wikipedia.org/wiki/Enolase

487. % of leukocytes required for extraction to be carried out?

Answer

50X10^9

489. Which kind of prevention are these?


1. Scaling and tooth polishing - secondary prevention
2.curettage – tertiary
3.deep curettage - tertiary
4.correction of tooth malalignment - secondaryoptions
(primary,secondary,tertiary?Levels of Prevention (Dentistry)

• Primary – Prenatal counseling, infant oral health program, pre-disease


tooth brushing/flossing,routine fluoride use, etc.

• Secondary – Remineralization of enamel caries, restoration of carious


teeth, treatment of gingivitis, correction of malocclusions, etc.

• Tertiary – Dental prosthesis, implants, surgical reconstruction following


trauma, periodontitis,cancer, etc.

PRIMARY

1.health education
2.plaque control advice
3.diet counseling
4.all forms of flouride
5.caries activity test
6.pit and fissure sealants
7.prophylactic odontomy
8.daily brushing and flossing
9.mouthguards in contact sports
10.supervised brushing

SECONDARY
1.periodic screening of oral disese
2.PRR
3.S&P
4.simple restorative procedures- eg amalgam, pulp capping

TERTIARY
1.pulpotomy
2.rct
3.Ext
4.rpd fpd implants
5. Minor tooth mvmts
6.deep curettage root planing
7.splinting
8.perio surgery

490. First line of treatment of a pt with lower lateral extracted while the
alveolus heals to cover thegap?
1. Conventional cantilever,
2. Partial denture.

Answer
Partial denture

491. Which is the most common jaww cyst in UK?

Answer
Radicular cyst (75%)

492.
1. What time do you give BCG vaccine

The BCG vaccination is recommended for all babies up to one year old who:
Babies who should have the
BCG vaccine
 are born in areas where the rates of TB are high
 have one or more parents or grandparents who were born in countries
with a high incidence of TB

Older children and adults who


should have the BCG vaccine
The BCG vaccination is recommended for all older children and adults at risk
of TB including:
 older children with an increased risk of TB who were not vaccinated
against TB when they were babies
 anyone under 16 who has come from an area where TB is widespread
 anyone under 16 who has been in close contact with someone who has
pulmonary TB (TB infection of the lung)

2. What time do you give MMR vaccine -

- At 1 & 4 years

493.
Patient 16 years old with bleeding gums and bad smell which antibiotic?
- metronidazol

2.Patient with osteitis + pus after 3rd molar extraction which antibiotic?. -
metronidazol

3. pt with pain after extraction:which analgesic – NSAID

494. Which nerve affected if tongue deviates to right when protruded?


A. Right hypoglossal
B. Left hypoglossal
C. Glossopharyngeal

Answer
A

495.Which nerve is affected if patient is unable to gaze laterally to left?


A. Right abducens
B. Left abducens
Answer. B

496. What's the most frequent site of endocarditis?


In other patients without a history of intravenous exposure,
endocarditis is more frequently left-sided.

Patients who inject narcotics or other drugs intravenously may introduce


infection which will travel to the right side of the heart classically
affecting the tricuspid valve, and most often caused by
S.aureus.

Aortic valve – 5-36%


Tricuspid – less than 10%
Bicuspid
Mitral – 28-45% (mitral and aortic both 0-35%)

497. Diet advice for a child. What's the most effective method?
A. tell the parents not to give sweets for the child,
B. dentist tells the child which are good and bad foods,
C. nurse talk to the child,
D. send the child to a dietician, etc

498. What protein has I, T and C subcomponents in muscle fibres?

Answer
. Troponin

499. Cells found in herpetic stomatitis and lichen planus?

500. Patient who had epilepsy 10 years ago but now in control. Which
tech will u use?
A. Inhalation sedation
B. IV sedation
C. Carry n with treatment

Answer: C

501. What enzyme assists the micro organism in causing dentine


caries?
1. collagenase
2. enolase
3. lactoferrin

Answer: Enolase
502. Is rheumatoid arthritis type 3 hypersensitivity ???
Yes
503. Which of the following is the most critical step in root canal
therapy?
A.Cleaning and shaping.
B.Obturation.
C.A hermetic apical seal.
D. A hermetic coronal seal.
E. C and D

Answer. (E)

504. At which angle to the external surface of proximal cavity walls in


a class II preparation for amalgam should be finished
A. An acute angle
B. An obtuse angle (Answer)
C. A right angle
D. An angle of 45°

505. If the sealant of bonding agent is not placed on part of enamel that
has been etched by an acid solution; you would expect:

A. Arrest of enamel carries by organic sulphides


B. The enamel is to return to normal within 7 days (Correct answer)
C. Continued enamel declassification in the etched areaD. Slight attrition
of the opposing tooth

506. In the inferior alveolar block the needle goes through or close to
which muscles:

A. Buccinator and superior constrictor (Correct)


B. Medial and lateral pterygoid
C. Medial pterygoid and superior constrictor
D. Temporal and lateral pterygoid
E. Temporal and medial pterygoid

507. Which of the following may be caused by newly placed restoration


which interferes with the occlusion
A. Apical abscess
B. Pulpal necrosis
C. Apical periodontitis (Correction)

508.
Rank the following impressions materials according to their flexibility

A. Alginate> Polysulphide> Silicone> Zinc Oxide Eugenol


B. Silicone> Alginate> Polysulphide> Zinc Oxide Eugenol
C. Alginate> Polysulphide> Zinc Oxide Eugenol>Silicone
D. Alginate> Silicone> Polysulfide> Zinc Oxide Eugenol (Correct)
E. Alginate> Zinc Oxide Eugenol> Silicone> Polysulphide

509. Mercury is dangerous when it turns into vapour form because of

A. It is accumulative and cause liver poison


B. It is accumulative and cause kidney poison
C. It induces neoplasia in the liver
D. It is accumulative and cause brain poison
E. It induces neoplasia in the brain

510. implant

sometime back i posted this doubt, as to re implant tooth in cardiac


patients and IF yes, do u prescribe antibiotics.

luckily i got this question answered by. Dr. James, who was one of
teachers for us on the DTP session day, he was also a previous ORE
examiner. He said u should implant the tooth back or ask it to be put
back even if its a cardiac patient . and there is no need to give antibiotics
now according to the recent guidelines, coz in cardiac patients it was
noted that even With antibiotics they carried the risk of infection, so there
was no point in prescribing antibiotics to them, it was not helping in
anyways. so, though it is given in Odell as we read that DO NOT re
implant avulsed tooth in cardiac patient ,that is outdated , the above is
the answer now.

511. WHAT FRACTURE OF THE JAW IN 8 YAER OLD WOULD


AFFECT GROWTH?

Ans. the condyle fracture would affect the growth.

512. Do we see tight junctions in oral mucosa?

Ans. less frequently observed

513. FLUORIDE SUPPLEMENT FOR A 4 YEAR CHILD,WHEN WATER


FLUORIDE LEVELIS LESS THAN 0.1?

Ans. Toothpaste 1350-1500ppm pea-size (Alternative - 0.5 mg)


Fluoride varnish 2.2%F twice yearly

514. A MAN HAS DIET CONTROLLED TYPE 2 DIABETES.HE


CONSUMES 30 GLASSES O F
ALCOHOL AND SMOKES 40 CIGARETTES
PER DAY.HE IS OTHERWISEH E A L T H Y
. W H A T M A Y B E T H E R E A S O
N O F D R Y M O U T H ?

Ans. Dehydration

515. Alginate after impression if you put soaked gauzes on it what


happens (contraction, expansion, shrinkage)

516. Ideal rake angle required for the universal curette in subgingival
curettage?

Ans. 70degrees

517. Which immunoglobin will increase in gingival inflammation?


Ans. IgG

518. Mercury is dangerous when it turns into vapour form because of,

A. It is accumulative and cause liver poison


B. It is accumulative and cause kidney poison
C. It induces neoplasia in the liver
D. It is accumulative and cause brain poison
E. It induces neoplasia in the brain

Ans.
B

Summary of toxicology
Effects on Humans:

Mercury vapor can cause effects in the central and peripheral nervous
systems, l u n g s , k i d n e y s , s k i n a n d e y e s i n h u m a n s . I t i s a l s o
mutagenic and affects the immune
system[ H a t h a w a y e t a l . 1 9 9 1 ; C l a y t o n a n d C l a y t o n 1 9
8 1 ; R o m 1 9 9 2 ] . A c u t e e x p o s u r e t o h i g h concentrations of
mercury vapor causes severe respiratory damage, while chronic
exposure to lower levels is primarily associated with central nervous
system damage [Hathaway et al. 1991]. Chronic exposure to mercury is
also associated with behavioural changes and alterations in peripheral
nervous system [ACGIH 1991]. Pulmonary effects
of mercury vapor
inhalation include diffuse interstitial pneumonitis with profuse fibrino
us exudation [Gosselin 1984].
Glomerular dysfunction and proteinuria have been observed mercury
exposed workers
[ A C G I H 1 9 9 1 ] . C h r o n i c m e r c u r y exposure can cause
discoloration of the cornea and lens, eyelid tremor and, rarely,
disturbances of vision and extra ocular muscles [Grant 1986]. Delayed
hypersensitivity reactions have been reported in individuals exposed to
mercury vapour [Clayton and Clayton 1981]. Mercury vapour is reported
to be mutagenic in humans, causing aneuploidy in lymphocytes of
exposed workers [Hathaway et al.1991].

519.
what is the percentage of plaque in the uk? - plaque...66%
what is the percentage of calculus in the uk? - calculus...69%
What % of population are allergic –
Statistics:
20-30% of people claim to have a food allergy, 5-8% of children and 1-
2% of adults have a food allergy. Peanut allergy - 0.4-0.6%,3% or 1.5
million of the British population who are at risk – latex allergy

What % of population are allergic in UK?

What % of bacterial load will decrease by cleaning

520. What bone lines alveolus?

What happens to alveolus after extraction?

Ans. cortical bone (lamina dura)

521. Nerves that may be damaged during lower molar extraction? How
to avoid?

Ans. lingual nerve and inf alveolar nerve

522. Extraction of lower first molar- nerves to be anaesthetised, after


anaesthesia , pt can't close eyes .. Reason?

Ans. injury to facial nerve

523. 3 year-old child, baby teeth to be extracted. You want to check


permanent
germs. Which radiograph?
Panoramic,
periapical,
bimolar, (Answer)
occlusal,
lateral skull view

DPT or Bimolar:
Identification of the devlopeing dentition
Confirmation of the presence/absence of teeth
Preliminary assessment of caries, apical condition. periodontal state

Standard occlusalview:
Identification of abnormal pathology
To show presence of unerupted teeth
Vertical parallax localisation either with DPT or periapical film
To supplement bimolar film

Mandibular occlusal:
Localisation of unerupted teeth

Periapicals:
To assess root morphology
To assess root resporption
To assess apical pathology
In combination with a standard occlusal or second periapical to
localiseunerupted teeth by horizontal parallax

Bitewings:
To assesss teeth of doubtful prognosis
Caries identification and periodontal bone levels

Lateral Cephalometric view:


To assess skeltal pattern and labial segment angulation
To aid assessment of unerupted teeth

524.Which Xray is needed for implants? – CBCT2.

Diagnosis of interproximal caries, what would you use? - bitewing

3.What do you find in the caries tissue?

4.Bacteria that most commonly cause infection in digestive system in the


UK? Helicobacter pylori common bacteria in stomach lining and
duodenum

5.Percentage of edentulous patients in England and Scotland.? -


England 6% Scotland 12

6.Percentage of tooth loss (wear) in 2003? - 36% in 16 to 24, 89% in 65


and over

7.Percentage of people without decay between 10 to 12 years old in


2003? - 66.6%

8.% of 12 year old with caries? - 38%


9.%of 16 year old with non carious tooth loss?
-10.%of edentulous in the uk? – 13%

525. Which cells r seen in TB and RA? - T lymphocytes

2 A growing plaque like pigmented patch on thigh. What investigation is


this? - purpura not verysure of this ans

3 Which hepatitis doesnt become chronic? - Hepatitis A

4 Saliva drooling, swelling in the floor of mouth something wrong with


finger. Wt diagnosis? -Ranula but no clue abt this question related to
fingers

5 Sharp shooting pain radiating to shoulders? -


6 Angioneurotic edema is characterized by which esterase inhibitor? -C1
levels low
C1 c2 c3 c4?

http://en.wikipedia.org/wiki/Angioedema

7. multiple lumps on neck, they are sensitive, and their skin is


pigmented.
Daignosis? - Chronicmyeloid leukaemia

526. someone who drinks 14 units in a week and smokes 30 cig a day
has diagnosis of precancrous lesion after biopsy of palate...wats da best
action to reduce chances of malignancy?
(excise lesion ,evaporate by laser ,avoid to stop smoking, avoide to stop
drinking)

Ans. stop smocking

527. In regards to carbide burs; the more number of cutting blades


and low speed will result in:
A. Less efficient cutting and a smoother surface
B. Less efficient cutting and a rougher surface
C. More efficient cutting and a smoother surface
D.More efficient cutting and a rougher surface (correct)

528. 1. What part of the nervous system is responsible for fight or flight-
Sympathetic Nervous System
2. What part of the nervous system causes increased salivary secretion?
Autonomic Nervous System

3. What part of the nervous system increases heart rate? Autonomic


Nervous System (Sympathetic Nervous System)
Maybe you will find this article useful: T h e n e r v o u s s y s t e m i s
divided into the somatic nervous system which controls
o r g a n s u n d e r voluntary control (mainly muscles) and the
Autonomic Nervous System (ANS) which regulates individual
organ function and homeostasis, and for the most part is not subject to
voluntary control. It is also known as the visceral or automatic system.
The ANS is predominantly an efferent system transmitting
impulses from the Central Nervous System (CNS) to peripheral
organ systems. Its effects include control of heart rate and force
of contraction, constriction and dilatation of blood vessels,
contraction and relaxation of smooth muscle in various organs,
visual accommodation, pupillary size and secretions from
exocrine and endocrine glands. Autonomic nerves constitute all
of the efferent fibres which leave the CNS, except for those
which innervate skeletal muscle. There are some afferent
autonomic fibres (i.e.transmit information from the periphery to
the CNS) which are concerned with the mediation
of v i s c e r a l s e n s a t i o n a n d t h e r e g u l a t i o n o f v a s o m o t o r a n d r
espiratory reflexes, for example the
baroreceptors and chemoreceptors in the carotid sinus and aortic arch w
hich are important in thecontrol of heart rate, blood pressure and
respiratory activity. These afferent fibres are usually carried to
the CNS by major autonomic nerves such as the vagus,
splanchnic or pelvic nerves, although afferent pain fibres from blood
vessels may be carried by somatic nerves. The ANS is primarily involved
in reflex arcs, involving an autonomic or somatic afferent limb, and then
autonomic and somatic efferent limbs. For instance, afferent
fibres may convey stimuli
from pain receptors, or mechanoreceptors and chemoreceptors in the h
eart, lungs, gastrointestinal tractetc.There may then be a reflex response
to this involving autonomic efferent fibres causing contraction of smooth
muscle in certain organs (e.g. blood vessels, eyes, lungs, bladder,
gastrointestinal tract)and influencing the function of the heart and
glands. The efferent limbs of these reflexes may also involve the
somatic nervous system (e.g. coughing and vomiting). Simple
reflexes are completed entirely within the organ concerned,
whereas more complex reflexes are controlled by the
higher autonomic centres in the CNS, principally the hypothalamus.
The ANS is divided into two separate divisions called the

Parasympathetic
and
Sympathetic

Systems, on the basis of anatomical and functional differences.


Both of these systems consist
of m y e l i n a t e d p r e g a n g l i o n i c f i b r e s w h i c h m a k e s y
n a p t i c c o n n e c t i o n s w i t h u n m y e l i n a t e d postganglionic fi
bres, and it is these which then innervate the effector organ. These syna
psesusually occur in clusters called ganglia. Most organs are innervated
by fibres from both divisions
of t h e A N S , a n d t h e i n f l u e n c e i s u s u a l l y o p p o s i n g ( e . g
. t h e v a g u s s l o w s t h e h e a r t , w h i l s t t h e sympathetic nerves
increase its rate and contractility), although it may be parallel (e.g. the
salivary glands). The responses of major effector organs to
autonomic nerve impulses are summarised in
Table 1
.
Parasympathetic Nervous System
The preganglionic outflow of the parasympathetic nervous system
arises from the cell bodies of the motor nuclei of the cranial nerves
III, VII, IX and X in the brain stem and from the second, third
and fourth sacral segments of the spinal cord. It is
t h e r e f o r e a l s o k n o w n a s t h e c r a n i o - s a c r a l outflow.
Preganglionic fibres run almost to the organ which is innervated, and
synapse in ganglia close to or within that organ, giving rise to
postganglionic fibres which then innervate the relevant tissue.
Theganglion cells may be either well organised (e.g. myenteric plexus of
the intestine) or diffuse (e.g. bladder, blood vessels).The cranial
nerves III, VII and IX affect the pupil and salivary gland
secretion, whilst the vagus nerve (X) carries fibres to the heart,
lungs, stomach, upper intestine and ureter. The sacral fibres form
pelvic plexuses which innervate the distal colon, rectum, bladder and
reproductive organs. In physiological terms, the parasympathetic system
is concerned with conservation and restoration of energy, as it causes
a reduction in heart rate and blood pressure, and facilitates
digestion and absorption of nutrients, and consequently the
excretion of waste products. The chemical transmitter at both pre and
postganglionic synapses in the parasympathetic system is Acetylcholine
(Ach). Ach is also the neurotransmitter at sympathetic preganglionic
synapses, some sympathetic postganglionic synapses, the
neuromuscular junction (somatic nervous system), and at some sites in
the CNS. Nerve fibres that release Ach from their endings are described
as cholinergic fibres. T h e s y n t h e s i s o f A c h o c c u r s i n t h e
cytoplasm of nerve endings and is stored in vesicles in
t h e presynaptic terminal. The arrival of a presynaptic action potential
causes an influx of calcium ions and the release of the contents of
several hundred vesicles into the synaptic cleft. The Ach then
binds to specific receptors on the postsynaptic membrane
and increases the membrane permeability to sodium, potassium and
calcium ions, which results in an excitatory post-synaptic potential. The
action of Ach is terminated by hydrolysis with the enzyme Acetyl
Cholinesterase. The specific Ach receptors have been subdivided
pharmacologically by the actions of the
alkaloidsm u s c a r i n e a n d n i c o t i n e . T h e a c t i o n s o f A c h
a t t h e p r e g a n g l i o n i c s y n a p s e s i n b o t h t h e parasymp
athetic and sympathetic systems is mimicked by nicotine, and all autono
mic ganglia aretherefore termed nicotinic. Nicotinic transmission also
occurs at the neuromuscular junction, in the CNS, the adrenal
medulla and at some sympathetic postganglionic sites (see
later). However, the actions of Ach at the parasympathetic
postganglionic nerve ending is mimicked by muscarine.
Muscarinic transmission also occurs at certain sites in the CNS. If you
are interested, it is continued on
http://www.nda.ox.ac.uk/wfsa/html/u05/u05_011.htm

4. What hormone increases blood glucose? Glucagon (But also Cortisol,


Growth Hormone & Adrenaline)

5. What hormone reduces blood glucose? Insulin

6. What hormone increases blood glucose and reduces potassium?


Cortisol

529. A 30 year old man with unknown allergy to latex goes into
anaphylactic shock whilst being treated in the dental surgery. Which
drug and route of administration is of most benefit in this situation?

A. Hydrocortisone - orally
B. Chlorphenamine – intramuscularly
C. Chlorphenamine - orally
D. Epinephrine - intravenously
E. Epinephrine – intramuscularly
530.75. What material is LEAST UNUSABLE for impression of PFM? -
Alginate

72. What makes that the post does not til - Ferrule effect

Statistics: % of fissure sealants that fail and get carious

53. How far from the papilla would you place teeth in a denture? - 10mm

51. Which x-ray to use for implant? in eric whaites it mentions cone
beam ct scan is it that or another one?

46. Periapical with approximal caries in a molar the question was: what
is the best access to thecavity? Direct access. Oclussal access

39. Sarcoidosis is: hereditary, autoimmune

34. Cement used for temporary crowns

36. Cement used for cantilever with little preparation

57. Immunoglobulin e present in mumps?is it igm

3. Minimum percentage of leucocytes for an extraction?

531. Goldish crust in the corner of the mouth of a complete denture


wearer patient (candida) not included

Angular chelitis

532. patient with recent complete denture, has ulcer in buccal sulcus? (
is it overextended flange? -

Overextended

533. Matrix band poor technique. what happen?


a. marginal over construction
b .marginal under construction
c. overhang
d. open contact point
ans: improper placement of both wedge and matrix band cause
overhang.

"improper placement of wedge will cause?" answer is overhang.

•if it says "not placing the wedge at all will cause?" - answer is both
overhang and open contact

•if quest is "improper matrix band placement will cause?"answer could


be a, b or c.. as the function of the matrix band is to restore anatomical
contours and establish good contact. but a marginal over construction
can be corrected easily and a marginal under construction is mostly due
to inadequate filling but if it is due to improper band placement, it would
mean that the band has been placed too gingivally(which usually does
not happen) and would also lead to a gingival overhang. hence i opted
for c.

534. How to decrease polymerization shrinkage of a composite?

Major ways of reducing polymerization shrinkage are:

1.Incremental placing of composite by two techniques: herring bone and


lateral filling methods. These two minimize the wall to wall effects of
shrinkage.
2. Directional curing technique: VLC materials start to cure at surface
closest to curing light and then shrink towards light. At the base of the
box, curing can be initiated by using light transmitting wedge to cure the
increment closest to cervical margins first. Remainder of increment
cured from occlusal aspect and follows again herring bone method of
increment placing.3. Increasing the effective filler loading

535. MOST POTENT VASOCOSTRICTOR OF SKIN – ADRENALINE/


NOR ADRENALINE/

2.OTHERS-PRODUCT WHICH IS FIRST FORMED-


CO2/ UREA/ OTHERS

3.DEVELOPMENT OF PERMANENT TOOTH GERM BY WHICH WEEK


/ 20 weeks- 5months in utero

4- WHICH MUSCLE IS PIERCED IN INFIRIOR ALVEOLAR NERVE


BLOCK /
temporalis & medial pteregiod

- buccinator and superior constrictor,

(if it pierces medial pterygoid it causes trismus)

5.BEST POST OPERATIVE ANALGESIC -


paracetamol 500mg 4 hrly upto 4gm maximum dose

6-FIBRIN STABILIZING FACTOR -


factor xiii is called fibrin stabilising factor

7-CLOTTING FACTOR DECRASED IN PATIENT WITH LIVER


DISEASE -

vit k is required for facor ii, vii, ix and x so it could be any of them becoz vit k
is synthesized in liver

536.
What happen if we use old diamond bur? - you have to apply excessive
pressure and more heat will be produced which is not good for pulp

2. What is best protection for pulp? -its dentine and if material ,then ZnO
eugenol (Â eugenol content of ZOE acts as obtundent and analgesic
and therefore used as sedative dressing over reversibly inflamed pulp.
calcium hydroxide can induce mineralization of adjacent pulp)

3. What happen to inter proximal composite filling?


(Over contouring,/under contouring, fracture)

537. Which lung volume doesn't change after exercise? - residual volume


What happens to your lung capacity when you smoke?Smoking doesn't
reduce lung capacity, it does increase the dead space as it reduces the
surface areaavailable for the exchange of gases.

What happens to stroke volumeduringexercise ?
it increases with exercise

What happens to muscleduring exercise ?
The muscle cells burn off ATP (Really fast) and they get much tighter to
deal with the strain. Makesure that you always stretch and warm up
before running as the muscles could strain!

What happens to your residual volumeduring exercise?
In contrast to Inspiratory Reserve volume,Tidal volume and Expiratory
reserve volume, residualvolume does not change with exercise

What is the difference between lung volume and lung capacity ?
lung capacity/volume: the volume of the air in the lung after maximal
inspiration there is nodifference

538. Immediate denture stages are:

1- assessment
2- primary imp.
3- sec. imp.
4- record occlusion
5- try-in
6- extraction7
- finish

539. The correct order of steps of partial denture design..??


1.- surveying
2.- outline saddles
3.- plan support
4.-obtain retention
5.-assess bracing required
6.- choose connector...

540. Which of the following ahs the highest sucrose content:

A. Ice cream
B. Canned juice
C. Cough syrups
D. Breakfast cereal
E. Sweet potato

541. Â penultimate tooth to erupting permanent dentition? - second


molar
542. DOSAGE OF 8 YEAR OLD CHILD WITH ANAPHYLACTIC
REACTION?

- 0.3 ml of 1:1000 adrenalin and if less than 6 than .15ml

543. DEVELOPMENT OF PAROTID GLAND BEGINS IN UTERO AT


WHAT MONTHS?

- 6weeks

544. After the age of 6 years, the greatest increase in the size of
the mandible occurs:

A. At the symphysis
B. Between canines
C. Distal to the first molar

545. Which of the following does state BEST the morphology of


periodontal ligament fibres:

A. Elastic
B. Striated
C. Non striated
D. Levity
E. Wavy

546.
Ten years old amalgam filling raised on tooth surface. Reason why its
raised?

– creep

547. Patient whose hands fell warm and moist is MOST likely to be
suffering from:**

A. Anxiety
B. Congestive cardiac failure
C. Thyrotoxicosis

548.
What kind of neurons supply mastocatory muscles ?
- Slow conducting or
fast conducting.

549.
1 what is most likely to happen to an interproximal composite filling
rather than amalgam?

Overcontouring,
undercontouring
,overhang.
fracture)

2.Amalgam MOD on molar.you did rct and palatal cusps are


missing..how do you restore?

Full gold crown ,


¾ gold crown.
550. 1. radiograph of choice for new edentulous patients in surgery?
- periapical for edentulous patient in problem area
2. method of diagnosing inter proximal caries in anterior teeth?
- its periapical by parallel technique

551. X-RAY TO DETECT CARIES IN PRIMARY MOLARS OF A THREE


YEAR OLD CHILDOPTIONS:

BIMOLAR TECHNIQUE
PA
BITEWING ETC

552. WHICH ANAESTHETIC PENETRATES THE BONE BETTER –

Articane

553. WHICH FRACTURE WILL LEAD TO MENINGITIS

- Le Fort 3 leads to meningitis

554. WHICH FRACTURE WILL LEAD TO EMPHYSEMA


- Orbital emphysema is typically a benign condition that occurs following
forceful injection of air into the orbital soft tissue spaces. In many cases
there is a history of trauma and fracture of an orbital bone, which permits
air entry. However, other mechanisms of orbital emphysema have been
reported including infection, pulmonary barotrauma, injury from
compressed-air hoses, and complications from surgery including dental
procedures

555. A 58 year old male presents at your surgery complaining of a sharp


pain of no more than 30minutes duration arising from his upper left
molar region. The pain is brought on by cold stimuli but persists after the
stimulus is removed. It does not seem to occur spontaneously. He has
tried taking Paracetemol and this does temporarily stop the pain from
recurring. The upper left 6 reacted to a lower current on electronic pulp
testing than the upper right 6, upper left 7 or the lower left molars. What
is the most likely cause of the patient's pain?

A. Acute/reversible pulpitis
B. Dentine sensitivity
C. Chronic/irreversible pulpitis
D. Periapical periodontitis
E. Trigeminal neuralgia

Explanation: importantly, there is a reduced response to pulp testing.


Hence, the answer would be Chronic Irreversible Pulpitis. But reversible
pulpitis doesn't persist after removal of the stimulus. I think the answer is
chronic/irreversible pulpitis based on the history given.

556. Vaccination which doesn’t include deactivated micro organism?

- polio vaccine

557. Which immunoglobin is present after a bacterial and viral infection?

- IgM and IgG

558. Which immunoglobulin is pentameric?

– IgM

559. fluorides

Prescribing high concentration fluoride toothpaste


Sodium fluoride 2,800 ppm toothpaste
Indications: high caries risk patients aged 10 years and over.

Sodium fluoride 5,000 ppm toothpaste

Indications: patients aged 16 years and over with high caries risk,
present or potential for root caries, dry mouth, orthodontic appliances,
overdentures, those with highly cariogenic diet or medication.

560. In periodontal ligament where are fibers are thinner?

- middle third of root. (Tencate)

561. Jugular venous pressure is best described as

A. Pulse pressure
B. 10 mmHg more than ventricular pressure
C. 10mmHg less
D. 20 mm hg less than VP
E. 20mmhg moreAns:

Jugular Venous pressure is right atrium pressure and it is 20 mm Hg


less than ventricular pressure.

562. PH of venous blood - 7.35-7.45

563. 1. Woman with hiatus hernia with tooth surface loss. Best treatment
option( surgery wasn't an option)

A. Medicine
B. Dietary advices.
C. Occlusal splint

well in patient with hiatus hernia it depends how sever is the condition. if
mild can be controlled by medicine and dietary advice and if sever may
require surgery. usually in most cases, sufferers experience no
discomfort and no treatment is required. However, when the hiatal
hernia is large, or is of the paraesophageal type, it is likely to cause
esophageal stricture and discomfort. Symptomatic patients should
elevate the head of their beds and avoid lying down directly after meals
until treatment is rendered. If the condition has been brought on by
stress, stress reduction techniques may be prescribed, or if overweight,
weight loss may be indicated. Medications that reduce the lower
esophageal sphincter (or LES) pressure should be avoided. Anti
secretory drugs like proton pump inhibitors and H
2 receptor blockers can be used to reduce acid secretion. Where hernia
symptoms are severe and chronic acid reflux is involved, surgery is
sometimes recommended, as chronic reflux can severely injure the
esophagus and even lead to esophageal cancer .

2.young adult with tooth surface loss. Best treatment option

removal of iatrogenic cause. And taking photographs and study models


and comparing. usually it is known as smith index.

3. Patient with bulemia. Best treatment option


referring to gp and gp will decide to have a psychiatric counselling or not

4.patient with anorexia nervosa, high caries rate, high sugar intake

control of sugar intake, dietary advice. ohi which include tooth brushing
and use of mouth wash, and regular appointment with dentist. and
referring to GP for anorexia nervosa.

564.
What is TRUE

A. Boiling point of acrylic > boiling point of water


B. Boiling point of acrylic is similar to that of water
C. Boiling point of acrylic < boiling point of water

565. which radiograph for the following BPE scores

1..

2 3 2
2 2 2

2.
* 1 4
4 2 3
3..
* 3 4
4 2 *

4..
* 2 *
* 1 *

5..
4 2 4
4 2 3

Ans

1.horizontal bitewing

2.upper sextant=full mouth periapical long cone technique with vertical


or horizontal bitewing
Lower=vertical bitewing

3.same as upper in 2 for both upper and lower

4.same

5.vertical bitewing for both upper and lower

566. Hormone that control calcium metabolism?(

parathyroid hormone,
vit D
calcitonin

Ans. Answer is Parathyroid hormone. Vitamin D is essential for the


ACTION of parathyroid. Calcitonin is antagonist of parathyroid hormone.

567. What are the end products of co2 metabolism?

HCO3 and H+
568.
Teenager has swelling involving his upper lip, the corner of his nose and
a region under his left eye. The swollen area is soft, fluctuant and
pointed on the labial plate under his lips on the left side. His body
temperature is 39°. What is the first thing you would do after taking
history and temperature

A. Refer him to physician


B. Anaesthetise all of the maxillary left anterior teeth to provide instant
relief
C. Give him an ice pack to be placed on the area to control the swelling
D. Take radiograph and test vitality of his teeth
E. Write prescription for antibiotics and delay treatment until swelling is
Reduced

569.
progenitor cells of platelets...?
Ans. Megakaryocytes are progenitor cells of platelets ..

570. What are neurons of Muscles of mastication ?


alpha,
beta,
gamma

571. what’s the maximun time GDC leaves the dentist physically
impaired away from his profession?
- 12 months

572. Which is completely derived from Meckles cartilage


- incus,
malleus
stapeus

573. Regeneration of odontoblast after a pulp pathology is from

A. Regenerate from the left odontoblast


B.Regenerate from undifferentiated mesenchymel cells
C. Regenerate from ectodermal cells
D. Regenerate from the undelying nectrotic tissue
574. In children a disease with enzyme deficiency

A. hypohpospatesia
B. Cyclic neutropenia
C. Juvienile periodontitis

Answer: A.Hypophosphatasia

Hypophosphatasia is a rare, and sometimes fatal metabolic


bone disease. Clinical symptoms are heterogeneous ranging from the
rapidly fatal perinatal variant, with profound skeletal hypomineralization
and respiratory compromise to a milder, progressive osteomalacia later
in life. Tissue non-specific alkaline phosphatase (TNSALP) deficiency in
osteoblasts and chondrocytes impairs bone mineralization, leading to
rickets or osteomalacia. The pathognomonic finding is subnormal serum
activity of the TNSALP enzyme, which is caused by one of 200 genetic
mutations identified to date in the gene encoding TNSALP.

575. In a flouridated toothpaste with 0.304% monoflourophospate the


ampont of flouride ions

A.400ppm
B.1000ppm
c.1500ppm
D.4000ppm

Answer: A. 400ppmA low fluoride, sorbitol-based toothpaste designed


specifically for children is available (ColgateJunior Toothpaste) and
contains 0.304% MFP (400 ppm fluoride)

576. Compared to natural teeth the displacement occurring in the


denture supporting mucosa under functional load

A. Ten times greater


B. Ten times less greater
C. One half

577.
Cause of hypoplasia in lateral permanent incisor

?Ans. Caries or trauma to primary tooth


578.Which type of dentin is not formed immediately due to a pulpal
trauma

A. primary
B. Secondary
C. Reparative
D. Tertiary

Normal prothrombin time & increased partial thromboblastin time seen


in
A. Haemophilia A
B. Thrombocytic pupura
C. Leukemia
D. Won willibrand disease

579. Which drug causes abdominal cramps in high doses?

- clindamycin

which drug causes diarrhea in high doses?

- amoxicillin or ampicillin is better choice

580. According to the Electricity at Work Regulations 1989 electrical


equipment must be tested every?

a. three years
b. one year
c. two years
d. 6 months

581. According to the water regulations, all dental equipment must be


protected by back syphonage and between the water ingress and
drainage points there should be a

a. “Type A” gap of 20mm


b. “Type B” gap of 30mm
c. “Type C” gap of 40mm
d. “Type D” gap of 50mm.
582. Fire certificates are only required for buildings with 20 people
working in them or if more than ten people work on floors other
than ground floors (same applies to dental practices). Fire
risk assessment should be carried every
a. 3 months
b. 6 months
c. 9 months
d. one year

583.
Most predictable anesthetic technique for lower lateral incisor?

Ans. infiltration

584. Lingual cavity in post teeth in Parkinson pt which mat is used

Ans. RMGIC

585. Condition in which phagocytic killing of bacteria is difficult?

Ans. Granuloma

586. child had 5mg/kg flouride its potential lethal dose what the first line
treatment?

Give child salty drink


Give child sugary drink
Give child injection
Give child glucose

587. Best xray for:

1.Showing successor teeth in young child - OPG


2.caries in 3 years old - Bimolar
3. Bilateral fracture of condyles - OPG
4.proximal caries in dentine - bitewings
5.dentine caries in anterior teeth - direct vision
6.impacted canine not palpable palataly - upper occlusal
7.showing amount of bone on a molar - bitewings
8. Showing relation between impacted 3rd molar and ID nerve - IOPA
9. Salivary calculi in submandibular duct - lower occlussal

Options: bimolars, bitewings, OPG, lateral oblique, IOPA, upper


occlusal, lower occlussal, directvision, probing, CT scan, occipitomental,
lateral cephAlometrics

588.
1) BP in 70 year old 140/90 mm of Hg

2) Consent-children

http://www.dentalprotection.org/adx/aspx/adxGetMedia.aspx?DocID=32
58,3257,158,1,Documents&MediaID=1907&Filename=Consent+P%26A.
pdf

3) Root canal sealer causing tissue damage - Paraformaldehyde-


containing endodontic filling materials or sealers (frequently known as
Sargenti pastes, N-2, N-2 Universal, RC-2B or RC-2B White) should not
be used for endodontic treatment because those materials are unsafe.
Extensive scientific research has proven unequivocally that
paraformaldehyde-containing filling materials and sealers can cause
irreversible damage to tissues near the root canal system including the
following:
destruction of connective tissue and bone;
intractable pain;
paresthesia and
dysthesia of the mandibular and maxillary nerves; and
chronic infections of the maxillary sinus.

4)SELECTED BACTERIAL SPECIES FOUND IN DENTAL PLAQUE

Facultative Anaerobic
Gram-Positive Streptococcus mutans
Streptococcus sanguis
Actinomyces viscosus
Gram-negative
Actinobacillus
actinomycetem comitans
Capnocytophypa species
Eikenella corrodens
Porphyromonas gingivalis
Fusobacterium nucleatum
Prevotella intermedia
Bacteroides forsythus
Campylobacter rectus
Spirochetes
Treponema denticola
(Other Treponema species)

5) Best Dental Health


- Wales
Scotland
NorthLondon
West Midlands
worst
South
589. % of UK population with type 1 hypersensitivity?
Ans. 10% scully
590. impression for gaging reflex is polyether
Ans. t is polyether as it is fast setting
591. What type of virus is hepatitis
A.a. Dna
b. Rna

592. Features of epithelial dysplasia. The changes that occur in


epithelial dysplasia include:
 Drop-shaped rete processes
Basal cellhyperplasia
Irregular epithelial stratification
Nuclear hyperchromatism
Increasednuclear-cytoplasmic ratio
Increased normal and abnormal mitosis
Enlarged nucleoli
Individual cell keratinization
Loss or reduction of cellular cohesion
Cellular pleomorphism
Loss of basal cell polarity
Anisocytosis
koilocytosis

Epithelial cell dysplasia is divided into three categories of severity


: mild, moderate, and severe.

Epithelial dysplasia becomes microinvasive squamous cell carcinoma


once the tumor begins to invade nearby tissue.

593. 1. Partial pressure of oxygen in the alveoli

Ans. 105

594. Picture of a fractured central incisor with the pink pulp showing and
a few drops of blood, how would you manage it?

a. Non setting calcium hydroxide and composite


b. Non setting calcium hydroxide and gic
c. Setting calcium hydroxide and gic
d. Setting calcium hydroxide and composit

595. A 30 year old man with unknown allergy to latex goes into
anaphylactic shock whilst being treated in the dental surgery. Which
drug and route of administration is of most benefit in this situation?

A. Hydrocortisone - orally
B. Chlorphenamine – intramuscularly
C. Chlorphenamine - orally
D. Epinephrine - intravenously
E. Epinephrine – intramuscularly
596. 1. What is the cause of the white line

a. Amelogenesis imperfecta
b. Dentinogenesis imperfecta
c. Hypomineralization
d. Flourosis

2. How would you manage this white line

a. Composite veneers
b. Porcelain veneers
c. Porcelain crowns

597. Picture of red coloured post in central incisors of cervical level


shown. Identify the post?

What is the ideal length required for post?

What is needed to prevent rotation of post?

Why is post used?

Ans. it is burnout indirect wrought metal post, length one third of tooth
length for CI it is 9 mm parallel side twist drill to remove dentine and
create post space, to distribute stress and provide retention for the tooth

598. Root filled tooth with no symptoms. When do you take the first x-ray
for the follow-up?

6months,
12 months,
18 months, etc.

2. Best inter dental cleaning for perio patients:

interdental brushes,
dental floss,
tooth picks, etc

599. What blood vessels have the most amount of smooth muscles?
a.Arteries
b. Arterioles
c. Capillaries
d. Veinules
e. Veins

Arteries have a great deal more smooth muscle within their walls
than veins, thus their greater wall thickness. This is because they
have to carry pumped blood away from the heart to all the
organsa n d t i s s u e s t h a t n e e d t h e o x y g e n a t e d b l o o d .
T h e endothelial l i n i n g o f e a c h i s s i m i l a r .

600. What cells do you see in parasitic infection?

eosinophils

8. What is the best protection for the pulp?

Dentine

601. 1.WHICH ONE IS POLISHED THE BEST?

MICROFILM,
GLASS IONOMER,
RESINMODIFIED GLASS IONOMER ...SO MANY OPTIONS)

2.WHICH OPTION IS MANDATORY DURING INHALATION


SEDATION?

OPT:
EGG,
PULSOXYMETRY..

3.HOW LONG DEPTH OF NAYARR CORE SHOULD GO THROUGH


THEROOT.

OPT:
3MM
,6MM,
HALF OF THE ROOT..,

601. A 13 year old girl presents with an unerupted permanent


canine and a retained primary canine. You cannot palpate the unerupted
canine in the buccal sulcus and you are uncertain as to whether it is
displaced palatally or in the line of the arch. Which single radiographic
view would be most helpful in locating the unerupted tooth?

A. Bitewing
B. DPT
C. Lateral oblique
D. Single periapical
E. Upper anterior occlusal

602. Which vessels involved in thermoregulation and vasodilatation out


of:

capillaries arteries
arterioles

4)cells present in rheumatoid arthritis?

- lymphocytes and plasma cells increase in RA

603. An african man with microcytic anaemia, normal ferritin levels,


what other investigations would you do?

Ans. CBC, sickelde , Hb electrophoresis

604. Most important factor for efficacy of hand instruments?

Ans. instrument should be sharp

605. picture of buccal mucosa with a white line at the level of teeth what
causes it?

lichen planus
lichenoid reaction
traumatic keratosis

2 first stage in management of rampant caries?

excavation and fluoridation


prevention and stabilisation.

3. %of 5yrs olds with dental caries?


4 picture of man with whitish growth on upper lip?
a sq cell papilloma
b sq cell carcinoma
c bcc

5 man with multiple myeloma comes for extraction and comes back after
6 weeks later and hissocket has not healed yet ?

a multiple myeloma of mandible


bdrug induced osteoradionecrosis
c dry socket
d drug induced osteo sclerosis

6 a man who smokes 2 packs of cigarettes a day and drinks would have
increased risk of oral cancer of?

a 11times
b 13 times
c 47 times

7 a black person with gingiva with melanotic pigmentation and white line
across the central incisors alone how wud u manage this?

a composite veneers
b porcelain veneers
c porcelain crowns

8 a picture of man with inability to open mouth properly for treatment


what is the cause?

a sarcoidosis
b scleroderma

606.
1. The ideal rake angle requred for the universal curette in sub gingival
currettage?
(-20,0,10,20)

2. Rake angle achievable by universal curette in subgingival curette?


(options same as above)
Ans. rake angle is not always equal to the cutting angle. by definition, if
the leading edge of the blade is ahead of the perpendicular,( hence
acute angled) the rake angle is negative. if it is behind the perpendicular
(obtuse angle) rake angle is positive. and if it is in line with the
perpendicular, rake angle is 0.in the first ques it asks the ideal rake
angle which is -20. (ideal cutting angle is 45)2nd quest asks upto a max
of which angle. ans is 0. (cutting angle can go to a max of 90)

607. Lower anterior missing in a 71 years old lady, has no pathology of


significance and good amount of bone. What kind if prosthesis?

- minimum preparation bridge

Upper anterior missing in 22 years old sports man who wears a mouth
gard?

- conventional bridge

Upper anterior lost in 13 years old - minimum preparation resin bonded


bridge

608. Anaemia's most commonly inherited disorder?

- Sickle cell anemia

609. Bone with endochondrial ossification only?

- squamous part of temporal bone and parts of mandible

610. Bone with intramembranous ossifi only

? Maxilla not given in option:-((

611. Saliva in relation to plasma?

Hypo, iso , hrpertonic –

hypotonic

612. Cells in granulomatous inflammation?

- macrophages, MNG's AND epithelioid cells


613. Maximum occlusal force beared by which pdl fibers?

– oblique

614. A patient complains of pain to his dentist. He has caries with


premolars and molars. He is visiting a dentist after 2 years. What part of
the history would you ask first.

Social history,
history of presenting complaint,
medical history,
dental history,
family history.

615. Which drug overdose causes hepatotoxicity? –

PARACETAMOL

616. What cells do you see in parasitic infection?

Basophils,
eosinophils,

617. Class III very small. What's the best treatment?

Lining
composite filling (no lining), etc

618. What's the best protection for the pulp?

Dentine,
lining,

619. Analgesic for children?

Dose - paracetamol.6-12 yrs 250-500mg,


1to 5 years, 120-250mg(every 4-6 hours)

620. drug for 8 yrs old child with anaphylaxis and dose ?

- epinephrine injection 250ug,0.5ml


621. Which lesion is more malignant?

Lichen planus
submucous fibrosis,
pemphigoid?

622. First line treatment pt 5years old with an abscess on buccal of


lower first molar

I and D.

623. Most common failure of Maryland bridge?

– debonding (retention failure)

624. Pulp stones seen in which syndrome? -

Ehlers–Danlos syndrome

625. SUCCESFUL MOLAR ENDODONTIC DEPEND ON EITHER

– a.straight line access


- b. patency

626. Which of the following drug is mast cell stabilizer?

a. salbutamol
b. adrenalinE
c. ipratropium bromide

627. Patient with iatrogenic overproduction of ACTH?

- Cushing syndrome

628. Pt with endogenous production of ACTH?

- Cushing disease
Exogen ous vs. endogenous

Hormones that come from outside the body are called exogenous;
hormones that come from withinthe body are called endogenous. The
most common cause of Cushing's syndrome is
exogenous
administration of glucocorticoids prescribed by a health care
practitioner to treat other diseases (called
iatrogenic Cushing's syndrome). This can be an effect of steroid
treatment of a variety of disorders such as asthmaand rheumatoid
arthritis, or in immunosuppression after an organ transplant.
Administration of synthetic ACTH is also possible, but ACTH is less often
prescribed due to cost and lesser utility. Although rare, Cushing's
syndrome can also be due to the use of medroxyprogesterone.

Endogenous
Cushing's syndrome results from some derangement of the body's
own system of secreting cortisol. Normally,ACTH is released from
the pituitary gland when necessary to stimulate the release of cortisol
from the adrenal glands.

• In pituitary Cushing's, a benign pituitary adenoma secretes ACTH. This


is also known asCushing's disease and is responsible for 70% of
endogenous Cushing's syndrome.
[10]

• In adrenal Cushing's, excess cortisol is produced by adrenal gland


tumors, hyperplastic adrenal glands, or adrenal glands with nodular
adrenal hyperplasia.

• Finally, tumors outside the normal pituitary-adrenal system can


produce ACTH that affects the adrenal glands. This final etiology is
called ectopic or paraneoplastic Cushing's syndrome
and is seen in diseases like small cell lung cancer .

Pseudo-cushing's syndrome
Elevated levels of total cortisol can also be due to estrogen found in oral
contraceptive pills that contain a mixture of estrogen and progesterone.
Estrogen can cause an increase of cortisol-binding globulin and thereby
cause the total cortisol level to be elevated. However, the total free
cortisol, which is the active hormone in the body, as measured by a 24
hour urine collection for urinary free cortisol, is normal.
[11]
629. Where the bone is lost the most in periodontal disease?

Buccal,
Lingual,
Mesial,
Distal,
Approximal

630. LOSS OF TASTE SENSATION IS ATTRIBUTED TO WHICH


NERVE

- Chorda Tympani(facial nerve)

631. WHICH GLAND PRODUCE CORTISOL - Adrenal Cortex ( Adrenal


Gland )

632. ACTH IS PRODUCED BY WHICH GLAND - Anterior Pituitary


Gland

633. What combines with O2?

- Hb

634. What is the arterial pressure for 02 and C02?

- Po2 is 100 Pco2 is 40

635. What cement u use to glue a alumina core ceramic crown?

- GIC
- RMGIC
- COMPOSITERESIN CEMENT

636. In which condition will direct immunoflourescence reveal binding of


autoantibodies to the intercellular substance of epithelial cells?

- Pemhigus ( IgG )
- Pemphigus/ benign mucous memb pemphigoid.
- erythema multiforme

637. WHAT IS THE FLAVOURED TOPICAL – benzocaine


.638. What’s increased in elderly?

Systole,
diastole,
stroke volume, etc

639. How many new cases of oral cancer are diagnosed in the UK
yearly?

– 6700-6800 cases in 2011 (almost 18 new cases every day)

How many deaths in UK each year caused by oral cancer?

- in 2012, 2119 patients died of oral cancer in the UK

•In 2007, 5,410 people in the UK were diagnosed with oral cancer .
i.e around 3% of allcancers.

•Oral cancer caused around 1,822 deaths in the UK in 2008.

640. Pulpotomy – which sulphate, etc (calcium hidroxide was not an


option!!!)is the best material to use, especially because of its low
toxicity?

Formocresol,
ferric

Ans. ferric sulphate

641. Root filled tooth with no symptoms. When do you take the first x-ray
for the follow-up?

6months,
12 months,
18 months, etc.

642. Best interdental cleaning for perio patients:

interdental brushes,
dental floss,
tooth picks,etc
643. Most benign:
Kaposi sar /
adenolymphoma
/ burkitts lymphoma

644. Immunoglobulin in mucosa – IgA

645. Site of oral cancer in a farmer exposed to sun light – lower lip

646. Adenoid cystic carcinoma is the most


common type of cancer in the minor salivary

647. In which condition will direct immunoflourescence reveal binding of


autoantibodies to the intercellular substance of epithelial cells?

Pemphigus
/ benign mucous memb pemphigoid. /
erythema multiforme

648. Everything about intraligamentary anaesthesia:


can it affect permanent tooth germ?
Does the needle need to be smaller than periodontal ligament?
Is it painful?

- YES TO ALL 3

649.

650. Cement for temporary crown:

GIC,
zinc phosphate,
zinc policarboxylate,
resin,
ZOE

651. Which root filling material comes in 2 pastes?

Tubli Seal, etc

652. Which cell release histamine?


Mast cells

653. Tumour in salivary gland which spreads along nerve sheaths:

Adenoid cystic ca

654. What's the percentage of >25 years with periodontitis in UK? - 64%

655. which oral tissue heals without scar?

- Palatal tissue

656. Which salivary gland tumor mostly occurs bilaterally?

- wartin tumor (benign) ,actinic cell carcinoma (malignant)

657. Which has softer walls?

Veins,
artery
Arteriole, etc..

658. Cell not present in healthy periodontal

- Cementoclasts they r present only in pathological conditions

659.
Which LA in patient with cardiac arrythmias?

– Lidocaine

660. Which hormone is secreted by the middle lobe of the pituitary


gland?

MSH

661. In which blood cell. Is the nucleus lost in the early stage?

RBC

662. What is the dose of Amoxicillin oral suspension

- 125mg/5ml-kid dose
663. What is the year of when RIDDOR regulation came out?

– 1995

664. What is the normal level of Glucose on a BM stick

- 4-7mmol

665. What is the altered cast technique?

- special technique for mucosa and tooth-born pd (keneddy's class1&2)?


Mucocompressive/mucostatic impression.

666. Most imp function of saliva?

- lubrication

667. Disposal of rotary file?

- sharps box

668. When do we use a nayyar core?

- nayyar core also called amalcore or coronal radicular amalgam core is


used in non vital root filled premolars and molars as a core to give tooth
extra strength before placing a crown

669. Analgesic used in geographic tongue

- benzydamine hydrochloride mouthwash

670. Best way to assess caries activity in a root

- bitewing radiographs

671.How would you treat a patient with MRSA,TB,Hepatitis?> options:

single use instruments,


treat using standard conditions
, send them to hospital

672. Pt with epitaxis and desqamative gingivitis....Which condition?


- mucous membrane pemphigoid

673. An upper deciduous molar has a caries exposure and on X ray the
corresponding 2nd permanent premolar is absent. What treatment would
you do to the deciduous tooth:

A. Pulpotomy
B. Endodontic treatment

C. Pulp capping

674. Epithelium lining the free gingiva is

keratinized
or non keratinized?

675. Neutrophils present most in-

Bacterial./ viral. / fungal infections

676. Very small class 3 cavity...- Whats the best treatment?

-> Lining / composite /no lining

677. Child with ulcers on soft palate, fever, malaise?

– Herpangina

678. Child with fever, blisters on trunk n mouth...what diagnosis?

– Chickenpox

679. Who is responsible for radiation protection?

– RPA

680. The periodontal ligament in a teeth without use appear to be:

A. Narrow
B. Wide
681. In class II restoration, all of the following considered to occur as
probable causes of periodontal problems except:

A. Flat ridge
B. Faulty or not proper contour
C. Not properly polished restoration
D. Cervical wall is too deeply apical
E.Overextension of lining in cavity

682. How many pulp horns are presented in a typical mandibular


deciduous second molar:

A. 2
B. 3
C. 4
D. 5

683. Which root filling tech use a hand piece?

- Thermo mechanical compaction

684. 1st line of treatment for a patient with lower lateral extracted while
alveolus heals to cover the gap?

Conventional cantilever,
partial denture
, etc

685. Gutta percha softens at what temperature

- 80-82 degrees

686. Anaesthesia for upper lateral incisor apicectomy .


- Anesthesia for extraction of canine
-Infraobital n block nd infiltrations along wid depending on options
687. Features of Bilateral fracture body of mandible.
- Disturbed occlusion and open bite
- Sublingual hematoma

688. which cement is placed after treating tooth with polyacrylic acid?
– Gic

689. What is needed to prevent rotation of post? - Ferrule effect

670. What does a single motor neuron to a lingual musculature suppy

1. Muscle spindle
2. Single muscle fiber
3. Multiple muscle fibers

671. Compensated blood loss.

- 20%/
40%/
60%

672. A pt presents with pain in the tongue and on swallowing. What is


your diagnosis and howwill be treated?

- giant cell arthritis

673. A pt present with sharp pain and wind and cold make it worse.What
is your diagnosis andhow to treat it?

- Trigeminal neuralgia

674. Who is more appropriate to give consent?

- Stepfather of a child
- Grandmother of a child- 16 year old sister of an 8 yearold child-
Mother of 4 year old child
675. Crown for a post retained tooth
- Metalceramic

676. Impression material for resorbed lower ridge

- .Zoe paste

677.cement used for temporary crown

- .Zoe cement

678. Medication for chronic sinusitis

- amoxicillin / clindamycin+metronidazole

679. To minimize the load on free end saddle partial denture

1.Use teeth with narrow buccal-lingual dimension


2. Use mucco-compressive impression
680. Whats the antagonist for heparin?
- protamine sulfate
681. in which condition are all factors deficient ?
- disseminated intravascular coagulation
682. What harmone increases blood glucose and reduced potassium?
- Cortisol increases blood glucose and reduced potassium
683. What harmone increases blood glucose?
- Glucagon.
684. Just want to share Diet Analysis Marking diet sheet
- 03 Day diary record which includes weekdays and weekend
1. Ring the main meals
2. Underline all sugar intakes in red.
3. Identify between meal snacks and note any associations.
Dietary Advice –
· After looking at your diet sheet, I’ve noticed that you are taking a lot of
added sugar everyday in tea, candies, fizzy drinks etc.

· To improve the health of your teeth, you will have to make some chang
es to your diet.
· You have to avoid inbetween meal sugary snacks and fizzy drinks. If yo
u have to snack,eat nuts or crisps instead of sugary foods.
· Eat sugars with the meals instead of dividing them through the day.
· You can limit sweets to once in the week.
· Don’t finish your meal with sugars.
· If you eat a sugary meal eat cheese or chew gums
(suger free) afterwards.
· Eat a lot of fresh fruits and vegetables in the day.
· If you take fizzy drinks, try to use a straw whenever possible.
· Do you have any questions
? Theory questions-
I. Which study proves relationship between caries and sugar?
1. Vipeholm Study
2. Turku sugar study
II. What were the findings in the study?
1. Vipeholm study was done to investigate the association between suga
r consumption anddental caries. It concluded that-
i. Consumption of sugary food is associated with a higher caries inciden
ce.
ii. Increasing the frequency of sugar intake increases caries incidence
iii. Lowering sugar intake reduces caries incidence.
iv. Consumption of sticky sugar-containing between meal products can
be associated with highcaries risk.
v. Several dietary factors are associated with caries incidence:
§ Amount of fermentable carbohydrate consumed
§ Sugar concentration of food
§ Physical form of carbohydrate
§ Oral retentiveness
§ Frequency of eating meals and snacks
§ Length of interval between eating
§ Sequence of food consumption
2. Turku Study-
A longitudinal study to evaluate the caries incidence as affected by parti
alsubstitution of dietary sucrose (S) with xylitol (X), the effects of S- or X-
containing chewinggums being compared during one year. The results
show a profound difference in the caries rate between the two groups.
The findings clearly indicate a therapeutic, caries inhibitory effect
of xylitol.
III. Names of 4 commonest substances to look for and avoid on labels
- Refined sugar, Sucrose, Fructose, Glucose
685. What channels does lignocaine block?
- sodium channels

686. Anion in bone?

– phosphate

687. which orthodontic appliance requires doesnot requires pt


compliance?

– Fixed

688. GIC bonds better to

dentine or
enamel

689. Leison of nerve in cavernous sinus that supplies Musles of eye and
divides into two brancheswhat nerve?
- The oculo motor nerve III divides into superior and inferior branches
and also is inthe cavernous sinus.

690. Ulcers not common in which type of viral infection?

– measles

691. Which salivary gland tumour occur bilaterally?

- Yes, it is Warthin’s but only in 10%

692. WHAT IS THE GOLDEN TIME FOR A TOOTH TO BE OUT OF


THE SOCKET?

- 30min

693. What is the equivalent dose of background radiation in hours for


2 bitewing?

- 8hrs

694. What is required to increase the efficiency and decrease the fatigue
of Universal curette?

Options:
sharp,
parallel to long axis of tooth surface,
lower shank should be parallel to toothsurface,
light weight, small ribbed surface, use a modified pen grasp.)

695. What's the equivalent dose of background radiation in hours for


2 bitewing?

See table in churchill:

If F speed film: 6.4 hours


If E speed: 8 hrs
Panoramic(rare earth screen) 28hrs"
(calcium tungstate) 56 hrs

696. Which condition their is intracellular killing of bacteria?

Inflammatory bowel disease......monocytes aid in phagocytes


697. Analgesic causing ringing sound?

- Aspirin

698. Analgesic given to patient with bleeding disorder?

Aspirin

699. which type of hypersensitivity reaction is seen in tuberculous


lesion?

a) type1
b)type2
c)type3
d)type4

700. Which of the following is the most important in the development


of dental caries?

a) time of sugar intake


b) frequency of sugar intake
c) amount of sugar intake

701. A 25-year-old patient presents with pain in a non-restored, lower


posterior tooth. Whatdiagnostic tests would be MOST EFFECTIVE in
determining cracked tooth syndrome?
1). EPT
2). Fiber optic light
3). Tooth sleuth
4). Radiographs
5). Staining

A. (1) and (5)


B. (1), (2) and (5)
C. (2), (3), and (5)

D. (1), (3), (4) and (5)

E. (4) and (5)

702. Pt with fatigue, night sweats, gingival enlargement and bleeding,

- Leukemia

703. Bur to prepare the rest for dentures

- round bur

704. Restoration for pt with xerostomia

- conventional GIC

705. Restoration for pt with Parkinson

– RMGIC

706. Drug causing fixed ulcer?

– nicorandil

707. LA for patients with cardiac arrhythmias?

- prilocine

708. Which of the clotting factors are associated with the following
conditions:

a)chirstmas disease - factor 9

b) von villebrant disease – von willbrand factor

c)hemophilia A - factor 8

d)vit K deficiency - factors: 2, 7, 9 and 10

709. AUTONOMIC EFFECTS ON SALIVATION, HOW?


- sypmathetic decrease salivation,and parasympathetic increase it
710. WHAT IS THE %OF TOOTH EROSION IN 6 YEAR OLD? - 46% ,

15 YEAR OLD- 34%

IN The UK

711. DISEASE WD PROGRESSIVE CARTILAGENOUS


DESTRUCTION?

– Osteoarthritis

712. MAN WITH PAIN IN PHALANGES?

RHEUMATOID ARTHRITIS

713. Development of permanent tooth germ by which week?

- 20th week

714. Best way to give dental health education?

1. Dentist advising patient on dental chair


2. Parent teaching their children
3. Through leaflets
4. Dental health education in school days
715. If there is lesion of right hypoglossal nerve then tongue deviates to
which side?
- TO THERIGHT SIDE
(Bells palsy due to facial n palsy angle of mouth is drawn to unaffected(
opposite side)
716. Most potent vasoconstrictor of skin?
- non epinephrine
717. 62 YEAR OLD IS DENIED A JOB BECAUSE OF HIS AGE-WHAT
ACT?
– equalityact
718. DENTAL NJURSE WANTS TO TAKE RADIOGRAPH
– IRMER
719. NURSE DENIED LEAVE
– EMPLOYMENT act
720. HOW WOULD YOU TREAT A SPILLAGE THAN 30 ML BLOOD? -
10 000 ppm sodiumhypochlorite

721. If amalgam gets contaminated with moisture, the most uncommon


result is:

**A. Blister formation


B. Post operative pain
C. Secondary caries
D. Lower compressive strength

722.Concerning local anaesthetics which one??

1. has the longest duration of action? bupivacaine

2.has shortest duration of action? .mepivacaine

3.is commonly used for topical anaesthetics? lignocaine 5%,


benzocaine, tetracaine

4.penetrates bone the most? .articaine

5.causes methaemoglobinaemia? prilocaine

723. BP OF A HEALTHY 70 years man?What increases in healthy old


person?

Systolic
or diastolic

724. Slowest voltage gated channel?

– calcium

725. A patient whose hands fell warm and moist is MOST likely to be
suffering from:
**A. Anxiety
B. Congestive cardiac failure
C. Thyrotoxicosis

726. What type of fracture ressults if patient elbowed on body of


mandible?

- opposite condyle

727. A MAN HAS DIET CONTROLLED TYPE 2 DIABETES.HE


CONSUMES 30 GLASSESOF ALCOHOL AND SMOKES 40
CIGARETTES PER DAY.HE IS OTHERWISEHEALTHY.WHAT MAY BE
THE REASON OF DRY MOUTH? –

dehydration

728.

729. Cells found in herpetic stomatitis and lichen planus?

- T lymphocytes

730. what protien has I,T and C subcomponents in muscle fibers?

– Troponin

731. Which cells r seen in TB and RA?

- In TB langhan's cells and epitheloid cells, in RA lymphocytes

732. Which hepatitis doesn’t become chronic?

- hep A

733. Dull aching chest pain radiating to shoulders/arm/neck?

- Angina
(If sharp pain & increased by inspiration - Pleurisy, which is seen in
chest infection & PE)

734. Angioneurotic edema is characterized by which esterase inhibitor?

- C1
735. Multiple lumps on neck, they are sensitive, and there skin is
pigmented. Daignosis?

- vonRecklinghausen’s i.e. neurofibromatosis type 1

736. At which angle to the external surface of proximal cavity walls in


a class II preparation for amalgam should be finished

A. An acute angle
B. An obtuse angle
C. A right angle
D. An angle of 45°
737. If the sealant of bonding agent is not placed on part of enamel that
has been etched by an acidsolution; you would expect:
A. Arrest of enamel carries by organic sulphides
B. The enamel is to return to normal within 7 days
C. Continued enamel declassification in the etched area

D. Slight attrition of the opposing tooth

738. A major difference between light cured and chemical cured


composite is that during setting or in function the light cures material
tends to:

A. Seal the margins better and completely


B. Exhibit less wear on time
C. Undergo greater colour change
D. Shrink rapidly
E. Posses greater fracture toughness

q739. Which component of RPD provides indirect retention?

- Occlusal and cingulum rest they are indirect retainers

740. Which of the following may be caused by newly placed restoration


which interferes with the occlusion

A. Apical abscess
B. Pulpal necrosis
C. Apical Periodontitis
741. Electrical pulp testing is least useful in /or does not detect in some
papers/ ,

A. Traumatised teeth
B. Just erupted teeth
C. Multi-rooted teeth
D. Capped teeth
E. Necrotic pulp

742. why is handpiece inclined lingually during cavity preparation of


lower first premolar

A.to avoid fracture of tooth


B.to provide better retention
C.to avoid cutting the marginal ridge
D.to avoid undermining the lingual cusp
E.for better access

743. after the age of 6 years, the greatest increase in the size of
the mandible occurs:

A. At the symphysis
B. Between canines
C. Distal to the first molar

744. The effects of tooth removal in healthy individuals can show as,
A. Loss of contacts
B. Slight tilting
C. Pocket formation
D. TMJ problem

E. All of the above

745. FLUORIDE SUPPLEMENT FOR A 4 YEAR CHILD,WHEN WATER


FLUORIDE LEVELIS LESS THAN 0.1?

Toothpaste 1350-1500ppm pea-size


Fluoride varnish 2.2%F twice yearly
746. Which immunoglobin will increase in gingival inflammation?

IgG

747. what is the percentage of plaque in the uk?

- plaque...66%

what is the percentage of calculus in the uk?

- calculus...69%

748. The use of physical barriers in Guided Tissue Regeneration is to do


what?

A.Retard apical migration of epithelium.


B.Exclude gingival connective tissue from the healing wound
C.Both
D. Neither

749. Which of the following inflammatory cells in periodontitis transform


into plasma cells uponsecondary exposure to an antigen?

a. Neutrophils
b. T-Lymphocytes
c. B-Lymphocytes
d. Macrophages
e.Mast Cells

750. Which of these muscles may affect the borders of mandibular


complete denture,

A. Mentalis
B. Lateral pterygoid
C. Orbicularis oris
D. Levator angulioris
E. Temporal

751. For dental caries to progress in dentine, ?

A. The dentine must contain soluble collagen


B. Enamel must contain glycoproteins
C. Diet must contain simple carbohydrate
D. Diet must contain polysaccharides
E. Pulp must contain complement

752. Which is more retentive form for anterior bridge

A. ¾ partial veneer crown


B. Class V inlay
C. Pinlay Veneer
D. Class III inlay with pins

753. When primary molars are prepared for stainless steel crowns
should the depth for reduction of the proximal surface be similar to the
depth of the buccal and lingual surfaces?
A. Yes; reduction of all wall is similar for best retention
B. No, proximal reduction is greater to allow the crown to pass the contact
area
C. No, the buccal surfaces has the greatest reduction to remove the
cervical bulge
D. Yes, all undercuts are uniformly removed so that the steel crown can
be seated
E. No, because of lateral constriction, the lingual surface needs greatest
reduction

754. Upper anterior absent in a 22 year old sports person who wears a
mouth guard, what kind of prosthesis would you provides?

- Although using mouth guard , dental implant is the first choice.

755. Upper anterior lost in a 13 year old, what kind of prosthesis would
you provide?

- A lost centalin childhood requires space maintainer as a removable


aplliance as mostly all upper teeth have beenerupted at 13 yrs old.

756. Von Willebrand's disease is the most common bleeding disorder.


pg-146 Scully.
757. Black stain of amalgam tattoo is caused by Mercury or silver?

758. 1 most common reason of failure of proximal amalgam filling?

- isthmus fracture

759. What is more likely to happen to an interproximal composite filling


rather than amalgam it's feature or undercontouring?

- polymerization shrinkage

760. Largest possible cavosurface angle?

- 110 degree

761. What cement you use to glue alumina core ceramic crown it's
panavia or resin modified GIC?

-Resin modified GIC

762. Pigmented fissure, which is the best way of caries diagnosis?

- investigate the area with round bur

763. Whom to contact if there is 20s of extra radiation dose?

- radiation protection advisor

764.Regeneration of odontoblast after a pulp pathology is from

A.Regenerate from the left odontoblast


B.Regenerate from undifferentiated mesenchymel cells
C.Regenerate from ectodermal cells
D.Regenerate from the undelying nectrotic tissue

765. In children a disease with enzyme deficiency


A.hypohpospatesia
B.Cyclic neutropenia
C.Juvienile periodontits
767. In a flouridated toothpaste with 0.304% monoflourophospate the
ampont of flouride ions

A.400ppm
B.1000ppm
c.1500ppm
D.4000ppm

768. Compared to natural teeth the displacement occurring in the


denture supporting mucosa under functional load

A.Ten times more greater


B.Ten times less greater
C..One half

769. Normal prothrombin time & increased partial thromboblastin time


seen in

A.Haemophilia A
B.Thrombocytic pupura
C.Leukemia
D.Von willibrand disease

770.
When treating a tooth with a non-vital pulp with a fistula presented;fistula
should be treated by

A. Surgical incision
B. Antibiotic coverage
C. The usual root canal procedures for non-vital teeth and no special
procedures for fistula

771. The major cause of mentalis muscle hyperactivity is

A. Class II Division I
B. Tongue thrust

772. What is TRUE

A. Boiling point of acrylic > boiling point of water


B. Boiling point of acrylic is similar to that of water

C. Boiling point of acrylic < boiling point of water

773. The presence of antibodies against which antigens of the following


indicates immunity againstHepatitis B

Hbs antigen
Hbe antigenHbc antigen

774. What component of the diet is the most important cause of erosion
of teeth-

cola,
alcohol, etc

775. if x ray beam not horizental to film,,which problem will arise?


- superimposition /overlappingof adjacent structures

776. Painful unilateral parotid sweeling which is acute and having no


fluctuency..which of thefolllowing is the cause
a.mumps
b.bacterial seladenitis
777. Anterior lower central incisers avulsed long time ago..what would
be the treatment

Ans. If extra alveolar period >24hrs,leave n consider for


space maintainence

778. There is assault and tooth is palataly displaced and pt come after
6 days what would be thetreatment.

- orthodontic repositioning

779. Pt on immunosuppresive drug develops lesions on the half of


palate...What virus is the cause?

- varicella zoster virus


780. Trauma to tooth with closed apex. In which one the tooth is more
likely to maintain vitality?

concussion
subluxation,
intrusive luxation,
avulsion, luxation

781. Sign of asthma:

wheezing

782. Name of the first formed dentine:

mantle

783. Where does Hertwig's epithelial root sheath come from?

- .cervical loop of enamel organ

784. Buffering in saliva:

bicarbonate ions

785. Pt with epitaxis and desqamative gingivitis.... Which condition?

- mucous membrane pemphigoid

786. Which answer is correct?

Approximately 75% of teeth have canal aberrations in the:

a.Apical 1 mm of the root


b.Apical 3 mm of the root
c.Middle 1/3 of the root
d.Furcation area
e.None of the above

Answer: b. Apical 3 mm of the root

787.
1)which cells proliferate in later stage of life:
odontoblast
cementoblastrest
cells of malaze
Undifferentiated mysenchymal cells

788. x-ray for trauma of chin... -

PA mandible.
OPG

789.
For retention of post which is imp?

•Diameter

•Size

•Luting agent

•Type of post

•Length

790.
.rct treated tooth ,post placed which is the best option to restore tooth .
Nayar tech used

1.Gold crown
2. Gold inlay
3. Pjc
Ans.

1) In post,the most important factor is the lenght


2) Nayar core usually done on molar with amalgam or composite,gold
crown is the best choice

791.
The effective dose depends on various parameters like voltage,
collimator used and speed of film used:

As you will notice below the effective dose varies with the type of film
and collimator used.
two bite-wings-70kv-rectangular collimation- F speed film@ effective
dose is 0.0016mSv

two bite-wings-70kv-rectangular collimation- E speed film@ effective


dose is 0.002mSv

two bite-wings-70kv-round collimation- D speed film@ effective dose is


0.008mSv.

792.
In the construction of a full veneer gold crown, future recession of
gingival tissue can beprevented or at least minimised by,

A. Extension of the crown 1 mm under the gingival crevice


B. Reproduction of normal tooth incline in the gingival one third of
the crown
C. Slight over contouring of the tooth in the gingival one fifth of the crown
D. Slight under contouring of the tooth in the gingival one fifth of the
crown

Ans.B

793. In planning and construction of a cast metal partial denture the study
cast

A. facilitates the construction of custom trays


B. minimizes the need for articulating
C. provides only limited information about inter ridge distance, which is
best assessed clinically
D. can be used as a working cast when duplicating facilities are
not available

Ans. D

794.
Picture - Patient complains about her lower canines (last teeth), she
has a nice partial denture, canines pocketing less then 2mm and
50% bone support, what treatment is the best

implantbased overdenture, extraction and wait for healing to make


a new denture,extraction and immediate dentures, RCT if Canines
treatable and overdenture
795 A 43year old patient has missing upper right first premolar and
molar.He hasgood oralhygiene and requests a fixed replacement for
these teeth. The other teeth on the same side are

all moderately restored with MOD amalgam restorations and are


vital, except the canine,hasa very large restoration and is root-filled. He
has group function.

Radiographs show a large sinus cavity and no peri-apical


pathology.What would be the restoration of choice for replacement of the
missing teeth?

A. Implant supported crowns


B. A conventional fixed bridge using the 7 and 5 as abutments
C. Two conventional cantilevered bridges, using the 7 and 3 as
abutments
D. A resin-bonded bridge, using the 7 and 5 as abutments
E. A conventional fixed-moveable bridge using the 7 and 5 as abutments

796. A woman has an ulcer for about 3 weeks and she says that it is
there since the new lowerdenture was placed. What would you do?

Adjust the lower denture and wait for 3 weeks for the ulcer to heal, do
incision biopsy immediately,do excision, biopsy, send her to oncologist.

Answer
If the ulcer is caused due to new denture, adjust the denture and wait.
Give ora base or similar for ulcer healing.

Or

If the ulcer is due to other cause Do incision biopsy if lesion is large


(Incision biopsy or excesional biopsy depends on the size of the lesion)

797. What 5 things would you check in a crown which comes back from the
lab before you calla patient?

Answer

1. Check the lab card prescription and compare the cast and crown if it
belongs to the same patient.
2. Check if its done according to your specification e.g. metal
crown, ceramic etc.
3. Check for any faults like fracture, fit.

798. Give two reasons why the crown would not fit the patient if it seats the
model perfectly

Answer

1. Undercuts
2. Migration of the abutments

799.
You try but you cannot manage to seat the crown. What
would you do-
3 t h i n g s before/when you are sending it to the lab

Answer

1. Correct the undercuts and occlusion


2. Re record the impression
3. Instructions to the lab technician

---------------------------------------------------------------------------------------------
posterior one-third of the tongue, branches creep anteriorly to the
terminal sulcus of tongue to carrytaste (SA) and general sensation from
the vallate papillae.

822.
Which is the most suitable examination for the diagnosis of crestal bone
loss?

a. DPT
b. periapicals
c. true occlusals
d. horizontal bitewings

Ans: D

823. In metal porcelain crown why is there a butt join?


a. porcelain is brittle
b. metal is brittle
c. porcelain is thin
d. metal is thin

Ans:A

824. The use of latex gloves does has the following effect when a
polyvinyl siloxaneimpression is taken

a. retards the set of the impression material


b. enhances the set of the impression material
c. results in porosities in the impression material
d. latex gloves stick to the polyvinyl siloxane impression material

Ans:A,

as sulphur present in latex inhibit polymerisation of pvs.so it retards

825. Attached gingiva is the thinnest in the mouth in the following region
of themouth?

a. Maxillary anterior buccal region


b. Maxillary molar buccal region
c. Mandibular anterior buccal region
d. Mandibular anterior lingual region
e. Mandibular molar lingual region

Ans: E,

attached gingiva is generally greatest in anteror region and narrower in


posterior with 1.8 mm in maxilla and 1.9 in mandible

826.

Which fracture affects facial nerve –


maxillary fractures (le fort 1,2,3) affect facial nerve causing paraesthesia
of upper lip and cheeks

827. man on NSAIDs has blood crusted lips – blood crusted lips are the
sign of erythemamultiformae triggered by a variety of drugs. NSAIDS is
one of them
828.
Basic Periodontal Examination (BPE) – if only one tooth is present in
one of thesextants score of that tooth is

a. included in the score of the adjacent sextant


b. not included anywhere
c. included in the score of the opposite arch same sextant
d. included in the scores of the remaining two sextants of the same arch

Ans: A

829. Relation of apex of 3rd molar to id canal,which radiograph will show


it?

Ans:

opg

830. Radiograph for mental nerve?

Ans.
periapical

831. If it is missing incisor-the most common a supernumerary tooth

MISSING : congenitally missing (usually accompanied by othe missing


teeth in mouth), avulsed,extracted

PRESENT BUT UNERUPTED : ectopic position of tooth


germ,dilaceration due to trauma, scar tissue, supernumerary teeth,
crowding, pathology eg cyst, odontogenic tumor etc.But the most
common cause is

SUPERNUMERARY TOOTH with an incidence of 1-3% in premaxilla

832.
characteristic for best retention of post

a. length
b. taper
c. superficial texture

Ans : A
833.

Radiograph for condylar fracture?


opt-

pa jaw,
opg

Ans: Reverse townes

834.

1. A patient complains of pain to his dentist. He has caries with


premolars and molars.He is visiting a dentist after 2 years. What part
of the history would you ask first.

Social history, history of presenting complaint, medical history, dental


history, family history.

2. Same scenario. No treatment should be undertaken without taking


which part of the case history

Ans;
dental history

835. Which antibiotic causes a rash if given in infectious


mononeucleosis?

Amoxicillin

Which antibiotic causes diarrhoea and abdominal cramps at a high


dose?

erthromycin

Which antibiotic is prescribed for gram positive streptococcal infection?

amoxicillin

Which antibiotic causes pseudomembranous colitis?

Clindamycin
Which antibiotic is used in case of MRSA?

according to scully its vancomycin in combination withother drugs


.according to BNF clindamycin alone is oneof the options. Check the
most recent BNF (& there are local variations too)

836. Patient with history of myocardial infarction who underwent


angioplasty 3 days ago and now has acute pulpitis. How would you
manage it?

> > Refer to the hospital


> > Prescribe antibiotics
> > Extirpate the pulp
.> > Do rct

> >> >

2) the percentage of oxygen in the blood of the veins.


> > 47mmhg> >

3)A patient comes to you with a 10 year old amalgam filling that seems
to be raised from theocclusal surface of the tooth. What could be the
reason for this? (Creep, Hygroscopic expansion)

> > Creep> >

4)Which bone is formed entirely intra-membraneously?

Ans: parietal, greater wing of sphenoid, lacerimal and Maxilla

837. RCT was carried out in a molar which had a MOD amalgam
restoration. What would be your choice of restoration.

Full gold crown, cast inlay, gold inlay, repeat the amalgam, leave it as it
is.

838. What would be the choice of restoration in a young bulimic female?

Metal ceramic crowns, ceramic onlay, metallic onlay.


839. which drug stabilises Mast cells?

cromolyn & nedocromyl,


Salbutamol (Salbutamol is beta 2 agonist)

840.
what percentage of people in uk have type 1 hypersensitivity?
i think its 10 % but not sure as its different in different books....but overall
sensitivity is 30-40 %

841.What would be the choice of restoration for a root filled canine?

Cast post and core, fiber reinforced post and ceramic core ?

842.

What is creep amalgam?

creep : slow plastic deformation occurs due to application of static or


dynamic force over time.

843. mast cell stablizers ( I t i s n o t S a l b u t a m o l )

Mast cell stabilizers


are cromone medications (e.g. Nedocromil, Na Cromoglycate) used to
prevent or control certain allergic disorders.They block a calcium
channel essential for mast cell degranulation, stabilizing the cell and
thereby preventing the release of histamine and related mediators. One
suspected pharmacodynamic
mechanism is the blocking of IgE-regulated calcium channels. Without
intracellular calcium, thehistamine vesicles cannot fuse to the cell
membrane and degranulate .As inhalers they are used to treat asthma,
as nasal sprays to treat hay fever (allergic rhinitis) and aseye drops
for allergic conjunctivitis. Finally in oral form they are used to treat the
rare conditionof mastocytosis.

844.
Which of the following needs explicit consent?

a. to share info with insurance company


b. to share info with other doctors in practice
c. to share info with therapist treating the patient
d. to share info with patient's GMP

845. When the enamel of a tooth is exposed to preparation containing a


high fluoride concentration;the major reaction is:

1- sodium flouride
2- calcium flouride
3- stannous fluoride
4- fluorapatite

846. PERIODENTAL LIGAMENT IS THINNER IN?

NEAR THE CROWN

INTHE MIDDLE

NEAR THE APEX

847. PRINSIBLE ACTION OF SALIVA IS:

1-LUBRICATION
2-ANTIBACTERIAL
3-BUFFERING
4-FOOD TASTING
5-FOOD DIGESTION

Excrection, saturation, dilution

8 4 8 . A n e m i a ’ s m o s t c o m m o n l y i n h e
r i t e d d i s o r d e r ? ? ?

Sickle cell anemia

849.Bone with intramembranous


ossification only?

M a x i l l a n o t g i v e n i n o p t i o n . Mandibular growth is
principally attributed to intramembranous ossification, augmented by
focal endochondral ossification at condylar head, angle and coronoid
process.

8 5 0 . B o n e w i t h e n d o c h o n d r a l o s s
i f i c a t i o n o n l y
ethmoid, body of sphenoid, basiocciput and the petrous temporal bones.

851. which LA can be given for pregnant ladies?

lidocaine

852. Unit of alcohol in 1 pint of beer?

Ans: 2.3 Units

Formulae
The number of UK units of alcohol in a drink can be determined by
multiplying the volume of thedrink (in millilitres) by its percentage ABV,
and dividing by 1000. Thus, one pint (568 ml) of beer at 4% ABV
contains:The formula uses the quantity in millilitres divided by 1000; this
has the result of there beingexactly
one unit per percentage point per litre
of any alcoholic
beverage.You can also think of 4% as being Hence .04 X 568 mL gives t
he amount of alcohol in mL,which you then divide by 10 to show the
number of units.When the volume of an alcoholic drinks is shown in
centilitres, determining the number of units in adrink is as simple as
multiplying volume by percentage (converted into a fraction of 1). Thus
75 centilitres of wine (the contents of a standard wine bottle) at 12
% ABVcontain:

568 * 4 / 100 = 2.3 units

853. bud stage of upper 1st molar begins at ?

Ans:
8th week i.u (primary molar)

854. Which dental tissue is 45 percent mineralised and resorts slower


than bone?

Options
-Enamel
-dentin
-cementum
856.
levels of IgA. Is more in saliva, serum, breast milk,tears... Can any one
help me to arrange them Which constitutes more Iga?

857. Iatrogenic overproduction of ACTH causes?

Cushing’s syndrome

858. Endogenous overproduction of ACTH causes?

Options ,cushing syndrome,


Cushing disease
Cushing’s syndrome

859.
which nerve is affected if the patient can not gaze laterally to the left

a-right abducens
b-left abducens
c-left trochlear
d-right trochlear.

2-which nerve is affected if the tongue deviates to the right when


protruded

a- right hypoglossal
b- left hypoglossal
c-right glossopharyngeal
d- left glossopharyngeal

3- Which side of the face do fractures occur most commonly?


a- right
b- left

4-Developmental of parotid gland begins in utro at what month?

4to 6 weeks

Parotid gland primordia (anlage) arises in weeks 5-6 from


ectoderm, followed by submandibular gland primordia in week 6 from
endoderm, then sublingual gland primordia in weeks 7-8 from endoderm
5. 30 yr man with a lip lesion he had come bach from a holiday and his
partner had the same lesion earlier on
a- herpes simplex
b-TB
c- erythema multiform

860.

which cement has the greatest wear resistance?

zinc phosphate

861. which is the toxic root canal sealant?

AH 26 -resin based sealer

TOXIC ROOT CANAL SEALANTS--AH26 TOXIC---

A REVIEW OF THE LITERATURER E V E A L S T H A T T H I S R


EPRESENTS THE LARGEST PUBLISHED CASE SERI
ES OFE N D O D O N T I C A L Y R E L A T E D I N J U R I
E S T O T H E I N F E R I O R A L V E O L A R NERVE.ST
UDIES HAVE SHOWN THAT ALL RC SEALANTS ARE NEUROTOXIC
TOS O M E D E G R E E . T H E M O S T N E U R O T O X I C A P P E A R
T O B E T H O S E C O N T A I N I N G PARAFORMALDEHYDE OR ONE
OF ITS ANALOGUE INCLIDING SARGENTI
PASTE(n2)O R E N D O M E T H A S O N E ( S E P T O D E N T -
AVAILABLE IN CANADA N EUROPE)OTHER SEALANTS C
ONTAIN ANALOGS OF
FORMALDEHYDE PARTICULARLY BEFORETHEY HAVE SET
(EXAMPLE-AH26-DENTSPLY MAILEFER,TULSA,OKLA)

862.
Man with candidiasis, generalised enlarged lymph nodes is seen in?

HIV infection

863. Which drug causes fixed ulcer

Nicodril

864. Atrial pressure of oxygen and carbon di oxide


Partial pressure of Oxygen is 75 - 100mm of Hgpartial pressure of CO2
is 40 mm of Hg (vander shrman)

Oxygen tension

Arterial blood oxygen tension (normal) PaO2


— Partial pressure of oxygen at sea level (765 mmHg) in arterial blood is
between 75 mmHg and 100mmHg

Venous blood oxygen tension (normal) PvO2


— Oxygen tension in venous blood at sea level is between 30 mmHg
and 40 mmHg

Carbon dioxide tension


Carbon dioxide is a by-product of food metabolism and in high amounts
has toxic effectsincluding: dyspnea, acidosis and altered consciousness

Arterial blood carbon dioxide tension PaCO2


— Partial pressure of carbon dioxide at sea level (765 mmHg) in arterial
blood is between 35 mmHgand 45 mmHg

Venous blood carbon dioxide tension PvCO2


— Partial pressure of carbon dioxide at sea level in venous blood is
between 40 mmHg and 50 mmHg

865.
Which immunoglobulin is present after a bacterial and viral infections

IgG is present after bacterial n viral infections


IgM in early infections

866.
Trauma to tooth with closed apex,in which one tooth is more likely to
maintain vitality-Concussion

And which one is more likely to loose vitality?

- Avulsion

Concussion

Intrusive luxation
LuxationAvulsion

Subluxation

867.
which of following conditions would warrant hospital admission for dental
surgery?

a.haemophillia

b. h/o pertussis in childhood

c.Hb-12gms

d. urine analysis showing acidic ph

e.all of above

2.when a CD wearer says s n sh tip of tongue touches

a. hard palate
b.soft palate
c.max.canines
d.max.premolars
e.max.molars.

3.why should the lingual embrassure b/w upper 3 n upper 4 be enlarged


during mouth preparationfor maxillary partial denture??

a.to prevent denture slip mesially


b.to prevent denture slip distally
c.to provide adequate retention
d.to provide adequate space for reciprocating arm
e.none

868.
1)cells present in acute periodontitis - neutrophils.
2)cells present in chronic periodontitis - lymphocytes.
3) What all can be seen during the process of healing

A clot forms and inflammatory cells debride injured tissue


d u r i n g t h e i n f l a m m a t o r y p h a s e . Epithelialization, fibroplasia,
and angiogenesis occur during the proliferative phase.
Meanwhile,granulation tissue forms and the wound begins to
contract. Finally, during the maturation phase,collagen forms tight
cross-links to other collagen and with protein molecules, increasing the
tensile strength of the scar

4) Malignant melanoma prevalance in oral cavity

Over 70 per cent of cases involve the posterior maxillary alveolar ridge
and hard palate

5) which is the most benign lymphoma

Burkitt's lymphoma

869. PEPPER POT SKULL APPEARANCE CAN BE SEEN IN WHICH


MEDICAL CONDITION?

In primary hyperparathyroidism, extensive resorption bone in the skull in


combination with cysticareas of osteopenia are termed pepper pot skull.

870.
What's the maximum time the GDC leaves the dentist physically
impaired away from his profession ?

36 months (decided by gdc health committee) reference: GDC website

871. Which one of the following drugs in hospital cannot be used in pt


with anaphylactic shock?

1-oxygen
2-Hartman's solution
3-adrenaline
4-predenisolone
5-chlorpheniramine maleate

Answer : hartmann's solution because its side effects include edema and
breathing difficulties. oxygen and adrenaline are the first line treatment
and prednisolone and chlorpheniramine are among the second line
treatment for anaphylaaxis

872. Which periodontal ligament is thinnest and thickest along the tooth
surfaces?
Periodontal ligament is hour glass shaped, meaning is thinner in the
middle third and wider in apicaland coronal thirds.

873.
Fluoridated Toothpaste used:

Toxicity
Safely tolerated dose (STD)
Dose below which symptoms of toxicity are unlikely =
1mg/kg bodyweight

Potentially lethal dose (PLD)

Lowest dose associated with a fatality. Patient should be hospitalized


= 5 mg/kg body weight

Certainly lethal dose (CLD) Survival unlikely = 32 - 64 mg/kg body


weight

Fluoride concentration in various products

Standard adult fluoride toothpaste 1000 ppm F (parts per million fluoride)
= 1 mg F/ml

Daily fluoride mouthrinse 0.05% NaF = 0.023% F=0.23mg F/mlAPF gel


1.23% F = 12.3 mg/ml

Fluoride varnish 5% NaF = 2.26% F = 22.6 mg/ml

To reach the 5 mg F/kg threshold (requiring hospitalization) a 5 yr-old


(about 19 kg) would have to ingest 95 (1 mg F) tablets, 95 ml of
toothpaste, or 7.6 ml of 1.23% of APF gel.

Antidotes : <5 mg F/kg body weight large volume of milk. >5 mg


F/kg body weight refer to hospitalquickly for gastric lavage. If any
delay give IV calcium gluconate and an emetic.

Water fluoridation
in a concentration of 1 ppm (1 mg F per litre) gives a caries reduction of
50%. Yet despite the proven benefits only 10% of the UK population
receive fluoridated water.
Topical fluoride
Overall, caries reductions of 20 - 40% are reported

Rinsing solutions

Mouthrinses are C/I in children <7 yrs. The concentration prescribed


dependsupon the frequency of use: 0.2% fortnightly/weekly or 0.05%
daily. Daily use is the most beneficial.Caries reductions of the order of
16 - 50% have been reported with rinsing alone. The most widely used
solution is sodium fluoride.

Toothpastes
aid tooth cleaning and polishing, but, most importantly, act as a vehicle
for fluoridedelivery

•Fluoride. Most toothpastes contain sodium monofluoro-phosphate &/or


sodium fluoride, in concentrations of 1000 - 1450 ppm (i.e. 1-1.45 mg
per 1 cm of paste). Caries reductions of 15% (in fluoridated areas) to
30% (in non-fluoridated areas) are reported. Low-dose formulations for
children <7 yrs containing <500 ppm are available, to reduce risk
of mottling.

•Anticalculus agents, e.g. sodium pyrophosphate, can reduce calculus


formation by 50%.

•De-sensitizing agents, e.g. 10% strontium or potassium chloride, or


1.4% formaldehyde.

•Antibacterial agents, e.g. triclosan

what`s the concentration of flouride in 2.26% NaF reparation.?

- 22600ppm or 22.6 mg/ml

2.26% NaF actually contains 5% NaF and 2.26% Fl ion.

The formula is %NaF X .45 = %F ion

%SnF2 x 0.25 = %F ion%

F ion X 10000= ppm


F ppm=mg/L

So 5%NaF multiplied by 0.45 is 2.25% which then multiplied by 10000


will give the result 22500 (which is close enough since Im sure figures
have been rounded off in the originalquestion)

Fissure sealants

The accepted figures for composite resin-based sealant retention are


> 85% after 1 yr and > 50%after 5 yrs.

Sugar
The term, sugar, is commonly used to refer to the mono-
a n d d i s a c c h a r i d e m e m b e r s o f t h e carbohydrate family.
Monosaccharides include glucose (dextrose or corn sugar),
fructose (fruitsugar), galactose, and mannose. Disaccharides include
lactose (in milk), maltose, and sucrose (caneor beet sugar).
Polysaccharides (starch) are composed of chains of glucose
molecules and are notreadily broken down by the oral flora.

In reducing cariogenicity:

•Sucrose, glucose, fructose, maltose (honey).

•Galactose, lactose.

•Complex carbohydrate (e.g. starch in rice, bread, potatoes).

874. Which following antifungal medication intercat with warfarin and


enhance its effect

a-miconazol
b-fluconazol
c-amphoticin B
d-ketokonazol

875.
Which test is the most reliable test to indicate the presence of active
hepatitis

Answer is HbsAg
Hbs(surface)antigen
- Hepatitis B surface antigen is a marker of infectivity. Its presence
indicateseither acute or chronic HBV infection.

Hbc(core)antigen

- It may be used in prevaccination testing to determine


previous exposure toHBV infection.

Hbe antigen

- Hepatitis B “e” antigen is a marker of a high degree of HBV infectivity,


and itcorrelates with a high level of HBV replication. It is primarily used
to help determine the clinicalmanagement of patients with chronic HBV
infection.

Antibody to HBs antigen

- Antibody to hepatitis B surface antigen


is a marker of immunity. Its presence indicates an immune response to
HBV infection, an immune response to vaccination, or the presence of
passively acquired antibody. (It is also known as

HBsAb,
but this abbreviation is best avoided since it is often confused with
abbreviations such as HBsAg.)

876. Edentulous population in UK :

It was 13% in 1998


(given in pink book, 5th edition)

BUT MOST RECENT STATISTICS SAYS:


Edentulous population: 6%Population with visible plaque:

66%Population with calculus : 68%Acc to ADULT DENTAL


HEALTH SURVEY,UK,2009,published in 2011

877. The ph when tooth starts deminarelising?

- pH 5.5
878. Etching

porcelain requires to b etched with


hydrofluoric acid & glass ceramics with ammonium bifluoride
---master dent vol 2 chapter3

879. Which has the most tendencies to become malignant?

Erythroplakia
Speckled /nodular leukoplakia
Erosive lichen planus
Oral sub mucous fibrosis

Ans: Erythroplakia>speckled leukoplakia>leukoplakia>oral sub mucous


fibrosis(10%)>lichen planus (1%).

880.
what is the fusion time for anterior , posterior and lateral fontanelle
(6,12,18,24months)

ans: Ans is anterior at 18-24 mths posterior at 2mths

881. Would you plz give me example of

Paracrine gland
Endocrine gland
Autocrine gland
Exocrine gland

Ans:

Exocrine gl.
- sweat gl., mammary gl, sebaceous gl., salivary gl.

Endocrine gl.
- thyroid (produce thyroxine), adreanl gland, ... actually most of the
hormones are produced this way. If I understood correctly, paracrine and
autocrine can be relevent to the type of signalling.
According to this:Paracrine - Somatostatin and histamine are paracrine
agents, hence pancreas is an example for this.
Autocrine - monocytes (for cytokine interleukin-1)
882.

The treatment for cracked tooth syndrom is:

1:Remove any associated restoration and investigate the extent of the


fracture line.

2:Adjust the occlusion  to reduce loading of offending cusp.

3:Placement of adhesive restorations,such as GIC,composit resin and


bonded amulgam may beeffective in preventive propagation of the
fracture in the short term. However,they are often ineffective, by
themselves,in the long term.

4:Placement of a full venner crown or adhesive metal onlay to splint the


remaining tooth structure ismore effective.

As an emergency measure,the tooth may be modestly reduced, to allow


placement of a temporary aluminium (or similar)crown form.Orthodontic
band or copper rings are (less satisfactory) alternatives.

5:Definitive restoration should be delayed untile the pulpal condition has


been determined. If RCT isrequired, this can be undertaken through the
temporary crown.
(all these applied to the vital tooth or the pulp not involved)

883. Which of the following decrease with age in the dental pulp?

A.Number of collagen fibers


B.Number of reticulin fibers
C.The size of the pulp
D.Calcifications within the pulp
E. A & B
F. B & C
G. A & C
H. C & D

Explanation:

as pulp ages there is decrease in reticulin fibres (pulp bcome more


fibrous and less cellular) size of pulp also decreases coz of continued
deposition of dentin.there is increase in number of collagen fibres and
calcification within pulp called pulp stones.
884.
Which of the following are true for porcelain Veneering alloys?

a) High noble metal is 45% noble


b) Gold (Au), Palladium (Pd) and Platinum (Pt) are noble metals
c) Pd-Ag is a base metal alloy
d) Cobalt-Cromium (Co-Cr) is a noble metal alloy
e) Noble metals alloys are 25% noble and have no gold requirement

1) a, c, d
2) c, d, e
3) a, b
4) b, e

885.
Which of the following medications has NOT been associated with
gingival enlargement?

A. Erythromycin.
B. Oral contraceptives.
C. Valproic acid.
D. Fluconazole.
E.Verapamil.

http://books.google.co.uk/books?id=VTeMNWAKqUcC&pg=PA330&lpg=
PA330&dq=erythromycin+and+gingival+enlargement&source=bl&ots=m
wccziX9UQ&sig=_Lr4QL34x1EwyqtbZpVyHLy6Ibg&hl=en&ei=lnK6ToGj
G4TH8gPov9nGBw&sa=X&oi=book_result&ct=result&resnum=10&sqi=2
&ved=0CG4Q6AEwCQ#v=onepage&q=erythromycin%20and%20gingiv
al%20enlargement&f=false

Gum hypertrophy may be caused by various drugs including:

•phenytoin
•nifedipine
•cyclosporin
•oral contraceptive with high oestrogen concentration - now uncommon

The drugs mainly associated with


GO are:
• Phenytoin
A drug used for the management of epilepsy,and other anti-convulsants
such as

sodium valproate,
phenobarbital
vigabatrin.
Ciclosporin an immuno-suppressant drug used to reduce organ
transplant rejection;

•Calcium-channel blockers
(nifedipine,verapamil ,diltiazem,oxodipine,amlodipine), a group of anti-
hypertensive drugs.

Other drugs, such as antibiotics (erythromycin) and hormones, have


been also associated with this side effect.

886. X-ray with defect x-ray placed on the wrong side (there were foil
impressions)

Ans: If u ever open an x ray film packet, u will see a lead


f o i l w h i c h h a s i t s t y p i c a l s u r f a c e irregularity. This foil is
present on the side from where u open the xray packet
i. e.Opposite to
the plain white part of the xray packet. Ideally, the white part of the pack
et is hit by the xray after hitting the tooth. If the film is place the other
way, the xrays after passing through the tooth, will hit the film with the
lead foil thereby giving the foil impression or design of the foil on the
developed film. This appearance is also called as a 'Tyre track'
appearance

887. Which of the following techniques could lead to the following


complications

: hematoma,

trismus,

temporary paralysis of cranial nerves III, IV, VI,

diplopia,

and
paralysis of the eye?

1. Posterior superior Alveolar (PSA)

2. Maxillary Nerve Block (V2)

3. Gow-Gates mandibular nerve block

4. Akinosi technique

5. Inferior alveolar nerve block -

888. what medication can we prescribed to a patient in a lichenoid


reaction to an amalgumrestoration who has a sore patch. -

benzydamine

889.

1. Which cement is more susceptible to fail if is contaminated with


saliva? – gic

2. Common reason for abscence of upper central incisor ?

3).According to…2000, there is color code separation.To which color


correspond the clinical areas and decontamination ?

a.Blue
b.red
c.orange
d.yellow

4) A patient is in status epilepticus,which medications will you give him?

a. Adrenaline
b.Diazepam
c.Midazolam (midazolam 10mg single dose )

890. Which of the following are disadvantages of using systemic


antibiotics?
a. Development of superinfections with opportunistic bacteria.
b. Potential drug interactions.
c. Increased cost of periodontal therapy.
d. Reliance on patient taking antibiotics as they are actually prescribed.
e. All the above.

891.
which one of the following radiograghic feachures would NOT suggest
that the patient would be at high risk of damage to thier inferior dental
nerve during the removal of mand 3th molar

1- loss of tramline of IDcanal


2-deviation of the tramlines of the IDcanal
3-narrowing of the tramline of the ID canal
4-radiolucint band across the tooth
5- Widening of the tramline of ID canal.

892. 1).How many colonies should contain a dental water unit?

a.>20 colonies/ml
b.>200 colonies/ml
c.>2000 colonies/ml
d.>20000 colonies/mlLess than <200 clonies/ml ( European standard)

2)A mum is concern by the oral hygien of her child who is 3 yrs old.They
live in area with appropriate fluoridation,which advice will you give her?

a. Do not swallow the tooth paste


b. Spit the tooth paste and do not rinse

3)What is the least important factor for a post?

a. The length
b. The diameter

4) you want to do a review on water fluoridation,which method will be


the most appropriate

a.Cohort study
b.randomized controlled study
c.systemic reviews
d.cross-sectonial surveys
893.

please , match the following files with their use

1. H files
2. Broaches
3. K files

a. engage and remove soft tissue from the canal

b. useful for penetrating and enlarging root canals

c. screwing-in forces due to sharp edges

Ans:

1c

2a

3b

894. diamond bur removes tooth hard tissue by grinding while carbide
do it by cutting for thatreason the latter is used in crown and bridges
preparation

895. Ore question child with red & gingival enlargement & fever &
lymphadenopathy. Which virus?

Herpes 1
Herpes 2
HIV
Cytomegalovirus
Coxsakievirus

896.
Ore question about antibiotic prophylaxis for patient allergic to penicillin
and can't swallow

Ans: Azithromycin if unable to swallow capsule( churchill page 72) or i.v


clindamycin

897.
3)Heart rate in a healthy young male individual during long period of
exercise -
90-100

4)Male suffered trauma and lost almost 1 lt of blood. What is the


reference blood volume per kilogram:
70ml,
120ml,
150ml,
220m –

Ans: 70

898.
Ore question about white line at the level of teeth ,what is the cause?

Lichen planus
Lichenoid reaction
Traumatic keratosis

899. how many ml of lignocaine 2% 1/80000 can injected to 125


kilogram person?

24 ,
26 ,
27 ,
30

Since the maximum dose allowed is 4.4mg/kg of lidocain 1:80 000

1. 7 x 125= 875 mg ( but remember you must never exceed the


maximum dose that is 500 mg, nomatter what the weight of the patient)
so you will use 500 mg in the second equation if the patient weight
was 70 then its 7mg x 70kg = 490 mg since its below 500 maximum
dose youwill us it in the second equation.

In a 20% Lidocaine means there is 1mL= 20 mg that`s because:

20% Lidocaine means 2gm per 100 ml of solution that`s equal to 2000
mg per 100 ml ( remove the zeros)

20 mg per 1 mL
2. then do a cross multiplication if:
1ml= 20mg then

x ml = 500 mg

that equals to= 25

from the available options 24 is the closest

you can`t give 26 ml or u will be exceeding themaximum dose.

so there are some points u need to memorize

2% plain lidocaine maximum dose is 4.4mg/kg maximum is 300 mg

2% lidocaine with 1:80 000 epinephrine 7mg/kg maximum is 500 mg

3% prilocaine 6 mg/kg maximum is 400 mg

4% prilocaine 6 mg/kg maximum is 400 mg

4% Articaine 7 mg/kg maximum is 440 mg

------------------------------------------------------------------------------------------------

900. Sudden unilateral onset of facial swelling and pain over parotid
area, which is not yet fluctuant

Options

a) Bacterial saliadenitis
b) Salivary duct infection
c) Saliorrhea
d) Viral sialadinitis

I think answer should be salivary duct infection which is unilateral.


Others like viral sialadinitis is most commonly caused by mumps
which is bilateral and similarly bacterial one is caused mostly by
S.aureus and is bilateral as well. Saliorrhea which is hypersalivation
is associated with oral lesions and heavy metal poisioning but
never read it to be unilateral.
901. the calculation is done in this way:
daily mouth wash contain neutral sodium fluoride which is in 4.5%
so for 0.5% (it will be like4.5 /1000 x 0.05 = 0.000225) - 225ppm on
the other hand if you want to know the ppm for 20% mouthwash
then you will do ( 4.5/1000 x0.20) = 900ppm

902.
1- what part of nervous system is responsible for fight or flight?
Sympathetic
2- What part of nervous system increase salivary secretion?
Parasympathetic
3- What part of nervous system increase heart rate?
Sympathetic

903. 1- What blood vessels have the most amount of smooth


muscles?
Arteries
2- what blood vessels are the capacitance and hold most of blood
volume?
Veins serve to return blood from organs to the heart . Veins are
also called "capacitance vessels" because most of the blood
volume (60%) is contained within veins.

904. In which condition is intra articular bleeding is minimal.


Vonwillebrand disease
905. osmf survival rate in 5 yrs & 10 yrs

osmf 10-15 yrs survival rate: 10%


5 yr : 8%

907. what does increase with age?


Systole
- In old age arteries get stiffer and systole increases.

so the bp increases
Diastole
Stroke volume

908 ELISA test used commonly for the diagnosis of which disease?

Because the ELISA can be performed to evaluate either the


presence of antigen or the presence of antibody in a sample, it is a
useful tool for determining serum antibody concentrations (such as
with theHIV test [8] or West Nile Virus ).

It has also found applications in thefood industry in detecting


potential food allergens such as milk , peanuts,walnuts, almonds,
and eggs.

ELISA can also be used in toxicology as a rapid presumptive


screen for certain classes of drugs.

ELISA (Enzyme-Linked Immunosorbent Assay) plate:

The ELISA was the first screening test widely used for HIV because
of its high sensitivity. In an ELISA, a person's serum is diluted 400-
fold and applied to a plate to which HIV antigens are attached. If
antibodies to HIV are present in the serum, they may bind to these
HIV antigens. The plate is then washed to remove all other
components of the serum.

A specially prepared "secondary antibody" — an antibody that


binds to other antibodies — is then applied to the plate, followed
byanother wash. This secondary antibody is chemically linked in
advance to an enzyme.
Thus, the plate will contain enzyme in proportion to the amount of
secondary antibody bound to the plate. A substrate for the enzyme
is applied, and catalysis by the enzyme leads to a change in color
or fluorescence.

ELISA results are reported as a number; the most controversial


aspect of this test is determining the "cut-off" point between a
positive and a negative result.
A cut-off point may be determined by comparing it with a known
standard. If an ELISA test is used for drug screening at workplace,
a cut-off concentration, 50 ng/mL, for example, is established, and
a sample that contains the standard concentration of analyte will be
prepared.
Unknowns thatgenerate a signal that is stronger than the known
sample are "positive."
Those that generate weaker signal are "negative."
Doctor Dennis E Bidwell and Alister Voller created the ELISA test
to detect various kind of diseases, such as Malaria, Chagas'
disease, and Johne disease.
ELISA tests also are used as in in vitro diagnostics in medical
laboratories. The other uses of ELISA include:
 detection of mycobacterial antibodies in tuberculosis.
 detection of rotavirus in feces.
 detection of hepatitis B markers in the serum.
 detection of enterotoxin of E. coli in feces.

909. What’s more likely to cause sickle cell crisis?


http://en.wikipedia.org/wiki/Sickle-cell_disease

2 Cement for temporary crown: GIC, zinc phosphate, zinc


policarboxylate, resin, ZOE
3 What are lymph node seperators and it's clinical significant?
4 what combines irreversible to Hb (haemoglobin)?

910. What’s the percentage of tooth erosion in 6 years-old and 15


years-old in uk?
6yrs-27%.
15yrs-54% erosion

911. Percentage of people indicated to hospital by infection?

5% 10% 15%20%

912. What’s the percentage of >25 years with periodontitis in UK?


- 64%

913. 1) Patient had trauma – Nasal – midface area and has watery
discharge and blood dischargefrom nose

a) Orbital fracture
b) Zygomatic fracture.
c) Lefort-1
d) Lefort-11

2) Patient with only mid-root fracture of front tooth, what is appropriate


splinting time?

a) 1 week
b) 2 weeks
c) 4 weeks
d) 3 months

3) Patient had trauma with minimal mobility of front teeth, no symptoms


or pain or anything, whatis the best approach

a) Splinting one week


b) Splinting two weeks
c) Splinting 4 weeks
d) Splinting 6-8 weeks
e) Soft diet and review

914.
A 90-year old gentleman presented to clinic who is edentulous and has
dentures upper and lower full 15-years old. Denture bit uncomfortable,
tooth structures little bit worn out, freewayspace 2-4mm, polished
surfaces satisfactory and occlusal wear minimal.

How would you proceed?

Copy dentures

Hard reline

Soft reline

Construction of new dentures

915. Treatment of basilar artery aneurysm ?

The basilar artery is the most important artery in the back


o f t h e b r a i n . T h e b a s i l a r a r t e r y c a n develop a bulge that pushes
outward, putting stress on the artery wall and causing it to burst. When
basilar artery aneurysms burst, they account for an estimated 27
percent of ischemic strokes, according to neurologist affiliated
with St Louis University.

Symptoms

•M o s t p e o p l e w i t h b a s i l a r a r t e r y a n e u r y s m r u p t u r e e x p e r i e
n c e m i n i s t r o k e s ( t r a n s i e n t ischemic attacks) in the days and
weeks before the aneurysm. The most common symptoms
of basilar artery aneurysm include:

weakness on one side of the body or face;

problems with speech and articulation

dizziness, nausea and vomiting;

headaches and visual problems;

l o s s o f consciousness.

Risk Factors
•People who develop basilar artery aneurysms often have the
following risk factors:

high blood pressure (hypertension),


diabetes,
heart disease,

vascular disease,

cigarette smoking

and

highcholesterol levels.

Diagnosis

A person may be diagnosed with basilar artery aneurysm after
undergoing blood tests to check for blood clotting problems and
heart disease. Doctors also use imaging tests, including
computed tomography (CT) scanning and magnetic resonance imaging
and angiography to check a patient's brain for bleeding, tumors and
aneurysm and rule out other conditions that may be causingthe
patient's symptoms. In addition, doctors may use transcranial
Doppler, echocardiography, and electrocardiography to make the
diagnosis of basilar artery aneurysm.

Medications

•If a basilar artery aneurysm is found before it ruptures, doctors may


prescribe medications tolower blood pressure and relax blood vessels
and prevent the aneurysm from breaking through the artert wall.

Basilar artery aneurysm is usually treated with intravenous (in


the vein) or intra-arterial (in the a r t e r y ) t h r o m b o l y s i s , w h i c h
uses drugs to break down clots in the body.

I n t h e U n i t e d S t a t e s , doctors administer medications called


tissue plasminogen activators (tPA) to people with basilar artery
aneurysm rupture. The success of these medications is
dependent on how quickly a personsought treatment. TPA
medicines may not be useful if given more than 12 hours after
symptoms appear.
Surgical Treatment

• The National Heart, Lung and Blood Institute also notes that
people with basilar artery a n e u r y s m m a y n e e d s u r g e r y t o
strengthen the artery wall and reduce the risk of basilar
artery aneurysmrupture.

People who have basilar artery aneurysm rupture are admitted to


a stroke unit for immediate and long-term care. Doctors' first
priority in treating patients with basilar artery aneurysm involves
maintaining blood flow to the brain and other areas of the body.
Patients may undergo a surgical procedure known as clipping, which
involves placing a small clip near the aneurysm.

This takes theaneurysm out of the brain's circulation and prevents


excessive bleeding.

Survival and Complications

•Basilar artery aneurysm rupture causes death in more than 70


percent of the people whoexperience it and occurs twice as often
in men as in women, (eMedicine.com reports).

People who survive basilar artery aneurysm rupture have a 20


percent chance of having an other stroke later. They often need
to make changes to their diet, exercise habits and lifestyle once
they are released f r o m t h e h o s p i t a l a n d p o s t -
stroke rehabilitation program. They may need neurological
a n d psychological therapy to cope with physical and emotional
changes they experience after aneurysm.

916.Which dental tissue is 45 percent mineralised an


d resorts slower than bone?

Options:
Enamel,dentin , cementum

917. A 3 YEAR OLD BABY YOU WANT TO CHECK PERMANENT


GERMS. WHICH RADIOGRAPH?
ANS- OPG

918. WHO IS RESPONSIBLE FOR JUSTIFICATION, QUALITY


ASSURANCE AND CONTROL X-RAY

ANS--IRMER PRACTITIONER OR
LEGAL PERSON

919. Mandibular Incisors - >

40% 2 canals but separate foramina in only 1%

Maxillary 4 – 74% >1 canal >1 foramina

Maxillary 5- 75% 1 canal and 1 foramina

Alcoholics in UK – Men – 72% Women – 57%

94 % OF COMBINED POPULATION OF ENGLAND, WALES AND


NORTHERN IRELAND WERE DENTATE

EDENTATE IS 6% England 12% Scotland

17 % HAS HEALTHY PERIODONTAL GUMS.

10% HAS EXCELLENT ORAL HEALTH

31%HAS TOOTH DECAY.

45% OF ADULTS HAS PERIODONTAL POCKETING EXCEEDING


4MM.

84%OF DENTATE ADULTS HAS ATLEAST ONE FILLED TOOTH.

ADULTS AGED UNDER 45 WERE LESS LIKELY TO HAVE ANY


FILLINGS.

37% OF DENTATE ADULTS HAD ARTIFICIAL CROWNS.

66%OF DENTATE ADULTS HAD VISIBLE PLAQUE.

CANCER survival in 65 year old:


Stage 1->85%

Stage 4 > 10%


stage1 males:

90% survival 1st year

65% survival 5 year

<55% survival 10 years

Submucosal fibrosis (due to betel quid chewing) 10% chances of


malignancy in 10 – 15 years

Tooth Wear (Attrition)

16-24yr old – 36%

65 yr and over – 89%

Percentage of population having type 1 hypersensitivity – 20 – 30%

Oral Cancer in UK – 2%

Squamous Cell Carcinoma – 90% of total oral cancers

CHILDREN DENTAL HEALTH SURVEY

Caries FREE 5 YEAR OLD IN ENGLAND AND WALES IS 55%

33% active caries – 2006/200738% OF 12 YEAR OLD IN UK AND


WALES HAD CARIES EXPERIENCE IN
PERMANENTDENTISTION.54%OF 9 YEAR OLD NEED
ORTHODONTIC EXPERIENCE.

33% boys and and 19% girls have experienced trauma at 12 years age

5 year old trauma 30-40% at toddler stage

More than 50% <4years trauma to primary tooth affects underlying


developing successor
Tooth erosion: 6 year old 52% , 15 year old 27%

Oral Cancers in UK: 2% which are 2000. Half die i.e. 1000

Percentage of fluoridated water in uk: 10%

Percentage of people with allergy in uk 10%

% of bacterial load decrease by cleaning – upto 80%

Gingivitis : 17-24yr olds - 77% , 25+ - 87%

Periodontitis: 17-24 yr olds – 3% 25+ - 64%

Percentage of plaque – 72%

Percentage of calculus – 73%

%age of fissure sealants that fail and get carious

>85% - after one year


>50% - after 5years

920. As for the hereditary enamel defects:

AMELOGENSIS IMPERFECTA, it’s either 1. Hypoplasia: according to


pink book 9 u will find it in the peads section) its Autosomanal dominant.

However in Cowson the book illustrates that it has different patterns of


inheritance (autosomal dominant, recessive and X linked, X linked
dominant types).

2. HYPOCALCIFICATION:

Autosomal dominant OR recessive in ( pink and cowson)

3. HYPOMATURATION: Autosomal dominant

921. You get 39 weeks paid, first 6 weeks you get 90% of you salary,
further 33 weeks you get £128.78 or 90% of your weekly earning,
whichever is lower.
You are also entitled to holidays while on maternity leave at full pay.

They cannot deduct bank holidays from your holiday entitlement while
you're on maternity leave. So you get 28 days paid holiday at the end of
maternity leave.

For more information check the goverment


website: http://www.direct.gov.uk/en/MoneyTaxAndBenefits/BenefitsTax
CreditsAndOtherSupport/Expectingorbringingupchildren/DG_10018741

922. Techniques for behaviour management

Tell, show, do

Self-explanatory, but use language the child will understand.

Desensitization

Used for child with pre-existing fears or phobias. Involves helping


patient to relaxin dental environment, then constructing a hierarchy of
fearful stimuli for that patient. These areintroduced to the child gradually,
with progression on to the next stimulus only when the child isable to
cope with previous situation.

Modelling

Useful for children with little previous dental experience who are
apprehensive. Encourage child to watch other children of similar age or
siblings receiving dental Rx happily.

Behaviour shaping

The aim of this is to guide and modify the child's responses, selectively
re-inforcing appropriate behaviour, whilst discouraging/ignoring
inappropriate behaviour.

Reinforcement

This is the strengthening of patterns of behaviour, usually by rewarding


good behaviour with approval and praise. If a child protests and is
uncooperative during Rx, do not immediately abandon session and
return them to the consolation of their parent, as this could inadvertently
reinforce the undesirable behaviour. It is better to try and ensure that
some phase of the Rx is completed, e.g. placing a dressing.

923. Severe early childhood caries

Aetiology

Frequent ingestion of sugar &/or reduced salivary flow.


Nursing bottle or bottle mouth caries
Associated with frequent consumption of a sugar-containing drink,
especially from a feeding-bottle. Also attributed to prolonged on-demand
breast-feeding, especially at night, due to the lactose in breast milk.
Characteristically, starts with themaxillary 1° incisors, but in more severe
cases the first primary molars are also involved. Themandibular incisors
are relatively protected by the tongue and saliva.

Rampant caries

A term often used to describe extensive, rapidly progressing caries


affecting many teeth in the primary &/or permanent dentition.

Severe early childhood caries may also be caused by the prolonged and
frequent intake of sugar- based medications; however, both
pharmaceutical companies and doctors are more aware of the problem
and the number of alternative sugar-free preparations is increasing.

See p. 126 for list.

Management

•Removal of aetiological factors (education, artificial saliva).

•Fluoride rinses for older age groups (daily 0.05%).

•1°dentition may need to extract teeth of poor prognosis and concentrate


on prevention for permanent dentition.

•2°dentition need assessment of long-term prognosis for teeth.

Final Rx plan should be drawn up in consultation with orthodontist.

Radiation caries Radiation for head and neck cancer may result in
fibrosis of salivary glands andsalivary flow. Patients often resort to
sucking sweets to alleviate their dry mouth, which exacerbates the
problem.

924. Thalasemia is an autosomal dominant disorder (scully 131-2)925.

1.
A 50-year-old man with a prosthetic heart valve requiring a tooth
extraction. There is no other relevant medical history.

No antibiotic

2.
The same patient who requires a further dental procedure three weeks
after the extraction.

No antibiotic

3.
A 55-year-old man who is allergic to penicillin and who is unable to
swallow capsules.

4.
A 5-year-old child attends in the dental emergency clinic at the
weekend, with swelling of the right side of the face resulting from an
abscessed tooth. There is no significant medical history. A.

5.
A 23-year-old woman presents complaining of ‘sore’, red, bleeding
gums andhalitosis. metronidazole

6. A pyrexic 30-year-old man presents with an exacerbation of an apical


infection two days after initiation of root canal therapy.

Amox 500mg

Options list:

A. Amoxicillin 250mg qds. for 5 days


B. Amoxicillin 500mg bd. for 7 days
C.Amoxicillin 1g intravenously
D.Amoxicillin 3g single oral dose
E.Azithromycin 500mg single oral dose
F.Clindamycin 600mg single oral dose
G.Erythromycin 250mg qds for 5 days
H.Metronidazole 200mg tds for 7 days
I.Metronidazole 200mg qds for 7 days
J.Penicillin V 125mg qds. for 5 days
K.Penicillin V 250mg qds. for 5 days
L.Tetracycline 250mg qds. for 5 days

926. muscles of snoring

2) muscled of swallowing

Eating and swallowing are complex neuromuscular activities consisting


essentially of three phases,an oral, pharyngeal and esophageal
phase. Each phase is controlled by a different neurological
mechanism.

The oral phase, which is entirely voluntary, is mainly controlled by the


medial temporal lobes and limbic system of thecerebral cortex with
contributions from the motor cortex and other cortical areas. The
pharyngeal swallow is started by the oral phase and subsequently is co-
ordinated by the swallowing center in the medulla oblongata and pons.
T h e r e f l e x i s i n i t i a t e d b y t o u c h receptors in the pharynx as
a bolus of food is pushed to the back of the mouth by the tongue.
Swallowing is a complex mechanism using both skeletal muscle
(tongue) and smooth muscles of the pharynx and esophagus. The
autonomic nervous system (ANS) coordinates this process in
the pharyngeal and esophageal phase sllowing

927. What's the origin of primary and secondary palate?

primary palate - frontonasal and medial nasal processes

secondary palate-fronto nasal and maxillary process

928.

When a pt suffers LMN lesion on diagnosis:

the tongue deviates to the unaffected side, while in the hypoglossal


lesion it deviated towards affected side.To test the function of the
nerve, a person is asked to poke out his/her tongue. If there is a loss
of function on one side (unilateral paralysis), the tongue will point toward
the affected side, due to unopposed action of the genioglossus muscle
(which pulls the tongue forward) on the side of the tongue that is usually
innervated. This is the result of a lower motor neuron lesion (the
damaged neuron directly innervates the skeletal muscle), and can lead
to fasciculations and atrophy of thetongue.
[3]
The strength of the tongue can be tested by getting the person to poke
the inside of his/her cheek,and feeling how strongly he/she can push a
finger pushed against the cheek - a more elegant way of testing than
directly touching the tongue.The tongue can also be looked at for signs
of lower motor neuron disease, suchas fasciculation and atrophy.
Paralysis/paresis of one side of the tongue results in ipsilateral curvature
of the tongue (apex towardthe impaired side of the mouth); i.e.,
the tongue will move toward the affected side.Cranial Nerve XII is
innervated by the contralateral cortex, so a purely upper motor
neuron (cortex) lesion will cause the tongue to deviate away from the
side of the cortical lesion. Additionally, the fasciculations and atrophy
seen in lower motor neuron lesions are not present.
[3]
Weakness of the tongue is displayed as a slurring of speech. The
tongue may feel "thick", "heavy",or "clumsy." Lingual sounds (i.e., l's, t's,
d's, n's, r's, etc.) are slurred and this is obvious in conversation

929. Fusion time for anterior fontanelle?

6 months
12 months
24 months

Ans. The range is 9- 18 months


. So best answer would be (average)12 months

930. Most appropriate situation to use hand gel to clean hands?

1 After eating and drinking


2 after seeing patient
3 before seeing patient

931.
Which ion is most important for nerve endings?

HCO3
Mg+2
CO3

The ion necessary for action potential as asked in first question is


HCO3. it maintains intercellular nerve potential negative with respect to
extracellular. in your second question, the Na+ ion moves inward and
causes the action potential. It’s Na and K also in addition to these at
terminal end they have voltage gated Ca channels.

932. 1- which cells are present in nerve ending?

Neurons with lots of axons

2- Which cells are important for nerve ending?

933. A boy 14 years old came to ur clinic with avulsed tooth not
accompanied by parents, most appropriate in this scenario ?

1- carry out treatment

2- ask the boy to place tooth in storage media

3- contact parents

934.
Which cells are defective in diabetes mellitus?

Neutrophils

Monocytes

Lymphocyte

Acidophils

935. Parotid gland nerve supply

Although the facial nerve passes through the parotid gland it


does not receive innervation from it.Instead, the parotid gland is
innervated by parasympathetic fibers (of inf. salivatory nucleus)
of the glossopharyngeal nerve arising from the otic ganglion, &
sympathetic fibers originating fromthe middle meningeal plexus.

Vascularization
Branches of the external carotid artery traverse the glandular tissue and
supply the parotid glandwith oxygenated blood. The main branch to
supply the gland is the transverse facial artery, where as numerous local
veins drain the organ. These veins drain into tributaries of external
and internal jugular veins.The maxillary vein and superficial temporal
vein meet to form the retromandibular vein within the parotid gland, but
are not responsible for draining it.Lymphatics comprise pre-auricular
lymph nodes.

936. The Index of Orthodontic Treatment Need

Grade 1 (None)

1 Extremely minor malocclusions including displacements less than 1


mm.

Grade 2 (Little)

•2a Increased overjet 3.6-6 mm with competent lips.


•2b Reverse overjet 0.1-1 mm.
•2c Anterior or posterior crossbite with up to 1 mm discrepancy between
retruded contact position and intercuspal position.
•2d Displacement of teeth 1.1-2 mm.
•2e Anterior or posterior openbite 1.1-2 mm.
•2f Increased overbite 3.5 mm or more, without gingival contact.
•2g Pre-normal or post-normal occlusions with no other anomalies.
Includes up to half a unitdiscrepancy.

Grade 3 (Moderate)

•3a Increased overjet 3.6-6 mm with incompetent lips.


•3b Reverse overjet 1.1-3.5 mm.
•3c Anterior or posterior crossbites with 1.1-2 mm discrepancy.
•3d Displacement of teeth 2.1-4 mm.
•3e Lateral or anterior openbite 2.1-4 mm.
•3f Increased and complete overbite without gingival trauma.

Grade 4 (Great)

•4a Increased overjet 6.1-9 mm.


•4b Reversed overjet greater than 3.5 mm with no masticatory or speech
difficulties.
•4c Anterior or posterior crossbites with greater than 2 mm discrepancy
between retrudedcontact position and intercuspal position.
•4d Severe displacement of teeth, greater than 4 mm.
•4e Extreme lateral or anterior openbites, greater than 4 mm.
•4f Increased and complete overbite with gingival or palatal trauma.
•4h Less extensive hypodontia requiring pre-restorative orthodontic
space closure to obviatethe need for a prosthesis.
•4l Posterior lingual crossbite with no functional occlusal contact in one
or both buccalsegments.•
4m Reverse overjet 1.1-3.5 mm with recorded masticatory and speech
difficulties.
•4t Partially erupted teeth, tipped and impacted against adjacent teeth.
•4x Supplemental teeth.

Grade 5 (Very great)

•5a Increased overjet greater than 9 mm


•5h Extensive hypodontia with restorative implications (more than
one tooth missing in anyquadrant) requiring pre-restorative orthodontics.
•5i Impeded eruption of teeth (with the exception of third molars) due to
crowding,displacement, the presence of supernumerary teeth, retained
deciduous teeth, and any pathological cause.
•5m Reverse overjet greater than 3.5 mm with reported masticatory and
speech difficulties.
•5p Defects of cleft lip and palate.
•5s Submerged deciduous teeth.

Cephalometrics

Most commonly used cephalometric points

:S = Sella: mid-point of sella turcica.


N = Nasion: most anterior point on fronto-nasal suture.
Or = Orbitale: most inferior anterior point on margin of orbit (take
average of twoimages).
Po = Porion: uppermost outermost point on bony external auditory
meatus.
ANS = Anterior nasal spine.
PNS = Posterior nasal spine.
Go = Gonion: most posterior inferior point on angle of mandible.
Me = Menton: lowermost point on the mandibular symphysis.
A = A point: position of deepest concavity on anterior profile of maxilla.
B = B point: position of deepest concavity on anterior profile of
mandibular symphysis.
Frankfort plane = Po-Or.
Maxillary plane = PNS-ANS.
Mandibular plane = Go-Me

SNA= 81Â ° ( Â ± 3 )

SNB= 79Â ° ( Â ± 3 )

ANB= 3Â ° ( Â ± 2 )

1 M a x = 109Â ° ( Â ± 6 )

-Mand= 93° (±6) or 120 minus MMPA

MMPA= 2 7 Â ° ( Â ± 4 )

Facial proportion= 55% (±2)

Inter-incisal angle= 133° (±10)


937.

Cleft lip and palatePrevalence

CLP varies with racial group and geographically.

Occurs in 1:750 Caucasian births, but prevalence M > F.

If unilateral L > R.

Family history in 40% of cases.

Isolated cleft palate occurs in 1:2000 births. F > M.


Family history in 20%.

938. Removable appliance components (Ortho)

Active
Springs, elastics and screws

Fixed appliance componentsBands –


used on molar teeth

Bonds( brackets)
– attached to enamel with composite

Arch wires
- Flexible nickel titanium (NiTi) archwires are used in the intial stages
of Rx and morerigid stainless steel wires for the planned tooth
movements. Tungsten molybdenum, and cobaltchromium alloys are also
popular.

Auxillaries
Elastic rings or wire ligatures are used to tie the archwire to the brackets.
Forces can beapplied to the teeth by auxiliary springs or elastics

939. Force range for tooth movement


Tipping – 30 – 60g
Bodyly – 100-150g
Rotation/extrusion – 50-75g
Intrusion – 15-25g
Optimal tooth movement is 1mm in 4 week period.

940. Classification of functional appliances

Tooth borne passive – Andersen activator ( Overbite reduction)


Tooth borne active – bionator ( expansion)
Tissue borne frankel appliance (Arch expension, managing
h y p e r a c t i v e m e n t a l i s m u s c l e (abnormal soft tissue activity))

941.

What is th Ph of Venous blood

(6.8 ,7.0 ,7.4)?

arterial blood-
7.4

Blood pH:

The bloodstream is the most critically buffered system of the


entire body, far more sensitive thanany other. Arterial and venous
blood must maintain a slightly alkaline pH: arterial blood pH = 7.41and
venous blood pH = 7.36. Because the normal pH of arterial blood is
7.41, a person is consideredto have acidosis when the pH of blood falls
below this value and to have alkalosis when the pH rises above 7.41.

Figure 3. Range of Arterial pH Values


ACIDOSIS pH = 1 to7.40
NEUTRAL pH=7.41
ALKALOSIS pH = 7.42 to14.0

http://www.chemcraft.net/acidph2.html

942. Which is completely derived from Meckles cartridge

Incus
Malleus
Stapes

Malleus and incus from meckel’s cartilage


and stapes from reichet's cartilage.

943.
Commonest microorganism causing food poisioning in UK?

Campylobacter

http://www.nhs.uk/Conditions/Food-poisoning/Pages/Causes.aspx

944.
In which condition are all clotting factors deficient ?

Disseminated intravascular coagulation

945.

Cause of pain 2 years after RCT.

1. INTRA RADICULAR causes,


*Necrotic material being left in the root canal

*Contamination of an initially sterile root canal during treatment

*Persistent infection of a root canal after treatment

*Loss of coronal seal and reinfection of a disinfected and sealed canal


system
*Bacteria left in accessory or lateral canals 2. Extra radicular causes,
*Persistent periradicular infection
*Radicular cysts
*Vertical root fractures

946.
Hiatus Hernia tooth surface loss
we should offer dietary advice but in addition management of dental
erosion presented in a chronic condition reference the guidelines on
dental erosion ( royal college of surgeons of England)
The document mentioned the causes of erosion and Hiatus hernia was
one of the causes.
THE TREATMENT PROPOSED:
1. children:
a. no complain: no ttt
b. sensitivity: cover the eroded surface with composite
c. affecting the appearance: composite on the anterior teeth and
stainless steel crown on posterior teeth

2. Adults:
Asses the intercuspal distance:
If sufficient - USE COMPOSITE TO REDUCE THE SENSITIVITY AND
IMPROVE THE APPEARANCE.

The eroded labial and palatal surface can be restored with veneers or
dentin bonded crowns.

If its not suffiecent ( due to a generalized erosion) - USE DAHL


APPLIANCE.

Periodontology

947.
Microorganisms
Streptococcus mutans group
Several species are recognized within this group,
i n c l u d i n g S . mutans and S. sobrinus. Aerobic. Synthesizes dextrans.
Colony density rises to >50% in presence of high dietary sucrose.
Able to produce acid from most sugars. Most important
organisms in theaetiology of caries.

Streptococcus oralis group


includes S. sanguis, S. mitis, and S. oralis. Account for up to
50% of streptococci in plaque. Heavily implicated in 50% of cases of
infective endocarditis. Streptococcus salivarius group Accounts for
about half the streptococci in saliva. Inconsistent producer of
Dextran.

S. intermedius, S. angiosus, S. constellatus (formerly S. milleri


group) Common isolates fromabscesses in the mouth and at distant
sites.
Lactobacillus
Secondary colonizer in caries. Very acidogenic. Often found in dentine
caries.

Porphyromonas gingivalis
Obligate anaerobe associated with chronic periodontitis and
aggressive periodontitis.

Prevotella intermedia
Found in chronic periodontitis, localized aggressive periodontitis,
(juvenile periodontitis), necrotizing periodontal disease, and areas of
severe gingival inflammation without attachment loss.Prevotella
nigrescens New, possibly more virulent.

Fusobacterium
Obligate anaerobes. Originally thought to be principal pathogens
in necrotizing periodontal disease. Remain a significant periodontal
pathogen.

Borrelia vincenti
(refringens) Large oral spirochaete; probably only a co-pathogen.

Actinobacillus actinomycetemcomitans
Microaerophilic, capnophilic, Gram -ve rod. Particular pathogen
in juvenile periodontitis and rapidly progressive periodontitis.

Actinomyces israelii
Filamentous organism; major cause of actinomycosis. A pe
r s i s t e n t r a r e infection which occurs predominantly in the mouth
and jaws and the female reproductive tract.Implicated in root
caries.

Candida albicans
Yeast-like fungus, famous as an opportunistic oral pathogen; probably
carried asa commensal by most people.

Spirochaetes
Obligate anaerobes implicated in periodontal disease; present in most
adult mouths.Borrelia, Treponema, and Leptospira belong to this family.

948. Plaque
Cocci predominate in plaque for the first 2 days, following which rods
and filamentous organisms become involved. This is associated
with increase in numbers of leucocytes at the gingival margin.

Between 6 and 10 days, if no cleaning has taken place, vibrios


and spirochaetes appear in plaqueand this is associated with clinical
gingivitis.

949. Plaque is the principal aetiological factor in virtually all forms of


periodontal disease. A c t i n o b a c i l l u s a c t i n o m y c e t e m
comitans a capnophilic organism thought to be involved in
t h e aetiology of LAP. is also active against neutrophils.

950. Debris or Oral Hygiene Index

This can be modified for personal use by using disclosing agents.

•0 No debris or stain.

•1 Soft debris covering not more than 1/3 of the tooth surface.

•2 Soft debris covering more than 1/3 but less than 2/3.

•3 Soft debris covering over 2/3 of tooth surface.

951. Pathogenesis of gingivitis and periodontitis

Initial lesion: polymorphs

Early: lymphocytes and polymorphs

Established: lymphocytes and plasma cells

Advanced: plasma cells predominate

952. Basic Periodontal Examination (BPE) Also known as Community


Periodontal Index of Treatment Needs (CPITN).

•0 = No disease,
•1 = Gingival bleeding but no pockets, no calculus, no overhanging
restoration. Rx: OHI.

•2 = No pockets >3 mm, subgingival calculus present or subgingival


retention site, e.g.overhang. Rx: OHI, scaling, and correction of any
iatrogenic problems.

•3 = Deepest pocket 4 or 5 mm. Rx: OHI, scaling, and root planning.

•4 = One or more tooth in sextant has a pocket >6 mm. Rx: scaling and
root planing, &/or flap as required.

•* = Furcation or total loss of attachment of 7 mm or more. Rx: full


periodontal examination of the sextant regardless of CPITN score.

953. Diagnosis Need to distinguish from apical abscess.

Apical abscess
• Non-vital
•TTP vertically
•May be mobile
•Loss of lamina dura on radiograph

Periodontal abscess
•Usually vital
•Pain on lateral movements
•Usually mobile
•Loss of alveolar crest on radiograph

954. Mobility Index:2


Grade 1 = Mobility <1 mm buccolingually
Grade 2 = Mobility 1-2 mm buccolingually
Grade 3 = Mobility of >2 mm buccolingually &/or vertical mobility

955. Furcation involvement


Classification

1st degree: horizontal loss of support not exceeding 1/3 tooth width.
Requires scaling and root planing, possibly with furcation plasty.

2nd degree: horizontal loss of support exceeding 1/3 but not


encompassing the total width of thefurcation area. May require furcation
plasty, &/or tunnel preparation, &/or root resection, &/or extraction. GTR.
3rd degree: horizontal through-and-through destruction in the
furcation area. May require tunnel preparation, &/or root resection, &/or
extraction.

Restorative
956. Assessment of retention
Greatest
→→→→→Poores

Maxilla 6 745312
Mandible 6 754321
Assessment of support
Greatest→→→→→Poorest
Maxilla 6 734512
Mandible 6 7354 21

957. Bridge failuresMost common reasons

•Loss of retention.

•Mechanical failure, e.g. # of casting.

•Problems with abutment teeth, e.g. secondary caries, periodontal


disease, loss of vitality.

958. Root canal therapy

Average working lengths

Refer to table on page 280 in pink book


Remember:

Maxillary
4 74% have >1 canal with> 1 foramina.

5 75% have 1 canal with 1 foramina.

6,7 Assume these teeth have 4 canals (2 MB; 1P; 1DB) until second MB
canal cannot be found.

Mandibular
1, 2 > 40% have 2 canals, but separate foramina are seen in only 1%.

4, 5 May have 2 canals, but these usually rejoin to give 1 foramina.

6, 7 Generally have 3 canals (MB; ML; D), but 1/3 have 4 canals (2 in D
root).

Some endodontists are using a technique involving maintenance of


apical patency, but at present there is little research evidence to support
this approach.

959. Evidence based practice.

Stregnth of evidence of research designs

Systematic reviews and Meta analysisRCT

Chort studiesCase control studies

Cross sectional surveyCase reports

Confidence interval: provides a measure of the precsion or uncertinity of


study results for makinginferences about the population of patients

.970.
67.The light emitted by the polymerization lamp has to be checked
from time to time. The meter used for this only measures light in
the range of:

A. 100-199 nm

B. 200-299 nm
C. 300-399 nm

D. 400-499 nm

Halogen light unit for polymerization of visible-light-curing materials.

•spectral range 400 - 520 nm (blue light) - spectral intensity is well


coordinated with spectralabsorption of all commercially available
composites

•75 W halogen lamp guarantees high curing depth even under extreme
conditions

• built-in light meter for to control the output light intesity

•built-in switch bottons allow accurate time exposure in the range of 20


or 40 seconds, or manual mode with acoustic beep every 10 seconds

•standard package includes 8 mm 55 degree bend dental probe


(universal), the probes can berotated through 360 degree enabliong
precise application of blue light

•Ergonomic design of the handpiece is comfortable to handle and allows


precisemanipulation

971. Patient with xerostomia, will benefit much more from

1. Scaling polishing
2. Flouride rinse

972.1- who checks if there is written protocol put up in the clinic for
radiation protection?

Employer(Legal person)

2- who justifies and optimises radiographs?

Practitioner

3- Whom to contact if there is 20s of extra radiation dose?


4- Radiation dose of 2 bite wings compared to natural?

Radiation dose of 2 bite wings = radiaton dose of a panoramic = 1-4


days of background radiation

http://www.radman.co.uk/resources/dental-radiography-and-x-ray.aspx

973. Which is least likely to be unilocular?

A ameloblastoma
B odontogenic keratocyst
C aneurysmal bone cyst
D myxoma
E osteoporosis

974.
Question 1: In a class II 2 malocclusion, which bridge design would be
contraindicated for a missing lateral upper incisor?

A.Cantilever bridge
B.Maryland bridge

Question 2: Which is the best cantilever bridge design for missing


maxillary canine? Abutment on

A.Both premolars
B.Lateral and central incisor
C.Lateral incisor
D.First premolar

975 .
1.How old patient most likely to have trauma that damages tooth germ of
permanent incisor?

Opt-1, 2 year??

Ans: its 2 years

The reference is master 2 chap 8 page 192 second ed


2. 30 year old man, front tooth missing, with sound abutment, what is the
best treatment between fixed fixed bridge and implant?

Ans: implant

976.
A 9 years-old child who has sustained a fracture of a maxillary
permanent central incisor inwhich 2 mm of the pulp is exposed, presents
for treatment 30 minutes after injury. Which of the following should be
considered?

A.Remove 1-2 mm of the pulp tissue surface, place calcium hydroxide


and fill with resin

B.Remove 1-2 mm of the pulp tissue surface and cover with ledermix

C.Place calcium hydroxide directly on the exposed pulp

D.Pulpotomy using formocresol

E.Pulpectomy and immediate root filling

In a flouridated toothpaste with 0.304% sodium fluoride the amount of


flouride ions is

A.400 ppm
B.1000 ppm
C.1500 ppm
D.4000 ppm

A low fluoride, sorbitol-based toothpaste designed specifically


for children is available (ColgateJunior Toothpaste)
and contains 0.304% MFP (400 ppm fluoride). Data from several
independent s t u d i e s i n d i c a t e t h a t , a l t h o u g h a d o s e -
r e s p o n s e r e l a t i o n s h i p d o e s e x i s t f o r f l u o r i d e l e v e l s i n tooth
paste and caries, use of a 400 ppm fluoride-containing paste by children
under 7 years of age
instead of the standard 1000 ppm fluoride paste
should not increase their caries risk.

http://www.australianprescriber.com/magazine/17/2/49/51/
2.Reversible pulpitis is characterized by

A.Pain lasts longer on hot or cold stimulus than normally


B.Patient can’t localize pain
C.Will have periapical involvement in radiograph

Reversible pulpitis

Symptoms Fleeting sensitivity/pain to hot, cold or sweet with immediate


onset. Pain is usually sharpand may be difficult to locate. Quickly
subsides after removal of the stimulus.
Signs Exaggerated response to pulp testing. Carious cavity/leaking
restoration.
Rx Remove any caries present and place a sedative dressing (e.g. ZOE)
or permanent restoration with suitable pulp protection.

3.Irreversible pulpitis is characterized by

A.There is often a history of spontanous pain


B.Sudden throbbing pain
C.Pain can’t be localised when it reaches the periapical area
D.There is pain which lingers for a short duration after romoval of
stimulus

Irreversible pulpitis
Symptoms Spontaneous pain which may last several hours, be worse at
night, and is often pulsatile in nature. Pain is elicited by hot and cold at
first, but in later stages heat is more significant and cold may actually
ease symptoms. A characteristic feature is that the pain remains after
the removal of the stimulus. Localization of pain may be difficult initially,
but as the infiammation spreads to the periapical tissues the tooth will
become more sensitive to pressure.
Signs
Application of heat (e.g. warm GP) elicits pain. Affected tooth may give
no or a reduced response to electric pulp tester. In later stages may
become TTP.

Rx Extirpation of the pulp and RCT is the treatment of choice (assuming


the tooth is to be saved). If time is short or if anaesthesia proves elusive
then removal of the coronal pulp and a Ledermix dressing can often
control the symptoms until the remaining pulp can be extirpated under
LA at the next appointment.
4.2.2 Mg of NaF contains how many mg of fluoride ions?

A. 0.5mg
B. 1.0 mg
C. 1.5mg
D. 10mg

Oral Rinse:

Sodium fluoride oral rinse is acidulated phosphate sodium fluoride and is


an oral rinse/supplement. Each teaspoonful (5 ml) contains 1.0 mg
fluoride ion (F-) from 2.2 mg sodium fluoride (NaF), in a 0.1 Molar
phosphate solution at pH 4, for use as a dental caries preventive in
children. Cherry, coolmint, bubble gum, grape - sugar and
saccharin free. Cinnamon - contains saccharin, but is sugar free.

Dental Rinse:

Sodium fluoride dental rinse provides 0.2% sodium fluoride in a


mint-flavored, neutral aqueous solution containing 6% alcohol. For
weekly use as caries preventive.

Brush-On Gel:

Self-topical neutral fluoride containing 1.1% sodium fluoride for use as a


dental caries preventive inchildren and adults. This prescription product
is not a dentifrice.

Gel-Drops:

Sodium fluoride (acidulated) gel-drops contain 0.5% fluoride


ion (F-) from 1.1% sodium fluoride(NaF) in a lime-flavored aqueous
solution containing 0.1 Molar Phosphate at pH 4.5. For daily self-
topical use as a dental caries preventive. This form of this drug (neutral)
also contains 0.5% fluoride ion (F-) from 1.1% NaF, but with no acid
phosphate, nor artificial flavor or color, at neutral pH.

Drops/Tablets:

Each ml contains 0.5 mg fluoride ion (F-) from 1.1 mg sodium


fluoride (NaF). For use as a dentalcaries preventive in children.
Sugar free. Saccharin-free. Sodium fluoride lozenge-type chewable
tablets for use as a dental caries preventive in children.S u g a r
free. Saccharin-free. Erythrosine (FD&C red #3) Free.
E a c h 0 . 2 5 m g F t a b l e t ( q u a r t e r - strength) contains 0.25 mg
F from 0.55 mg NaF. Each 0.5 mg F tablet (half-strength)
contains 0.5mg F from 1.1 mg NaF. Each 1.0 mg F tablet (full-
strength) contains 1.0 mg F from 2.2 mg NaF.Each SF 0.25 mg F
tablet (SF for Special Formula: no artificial color or flavor) contains 0.25
mg Ffrom 0.55 mg NaF.

http://www.orgyn.com/resources/genrx/D002244.asp

977.

sensitivity is according to definition number of true positives that are


correctly identified by a test and specificity is the number of true
negatives identified by a test....so its better for a test to be sensitive
rather than just specific...sensitivity of a test is the probability that it will
surely give us a positive result in the end when we compare it to a gold
standard. For eg if drinking sugary drinks
is proved to give you caries then a test
in which we give children who don't brush their teeth sugary d r i n k s i s
a test with high sensitivity...meaning it has a high
likelihood of giving us a positive r e s u l t . . . .
something they call true positive(the patient has cari
e s a n d t h e t e s t i s p o s i t i v e ) On the other hand if you do a
test in which you give children sugary drinks but the children
are brushing their teeth and have a good oral hygiene...then there is a
high likelihood that the children wont get caries...this means that this
test has high specificity meaning the test is producing a true
negative result (the patient does not have caries and the test is
negative) when tested on a sample which was initially normai. So
I think that would mean sensitivity is inversely proportional to
specificity. Moreover, sensitivity is the probability that the test
says a person has a disease and when we do the
test/experiment, we find out it is a fact that the person reallyhas
a disease. S p e c i f i c i t y i s w h e n t h e ( n e w ) t e s t s a y s t h e
person doesn't have the disease and when we do
t h e test/experiment (Gold standard) it is a fact that the person doesn't
have the disease.
978. Which dental hard tissue is mineralised first?

Enamel
Dentine
Cementum
Bone

979.
WHAT IS THE treatment for
- Â Denture stomatitis and resistant angular cheilitis?

FOR DENTURE STOMATITIS

:start with:

1. fluconazole 50 mg onece daily for 7 days


2. miconazol oromucosal gel 2% 24mg/ml
3. Amphotrecin 10mg use 4 times a day for 10 days
4. Nystatine suspension 100,000 UNITS

1 ml after food 4 times a day

RESISTANT CASE OF ANGULAR CHELITIS

USE:

Miconazole 2% plus hydrocortisone 1% cream or ointments use 30gm


apply twice daily to the corner of the mouth for a maximum of 7 days

Unresponsive cases can be treated with hydrocortisone and miconazole


cream or ointment.

Continue treatment until clinical resolution is achieved.

A lack of clinical response might indicate predisposing factors such as a


concurrent haematinic deficiency or diabetes. Refer such cases to a
specialist or the patient’s general medical practitioner.

980. WHAT ARE THE SEPARATORS OF LYMPHNODES ?

Lymphnodes seprators are investing layer of fascia, they prevent


the spread of infection in surrounding tissues.
981. What is the maximum dose of salbutamol?

If you asking about emergency application of sul for an acute episode


of asthma during a dental session the answer is 10 activation of the sul
inhaler

SALBUTAMOL

By mouth (but use by inhalation preferred), 4 mg (elderly and sensitive


patients initially 2mg) 3–4 times daily; max. single dose 8 mg

By inhalation of nebulised solution, adult and child over 5 years 2.5–5


mg, repeated up to 4 times daily or more frequently in severe cases;
child under 5 years 2.5 mg, repeated up to 4 times daily or more
frequently in severe cases. Please refer BNF

982. In which year is GDC going to conduct revalidation?

a. 2010
b. 2011
c. 2012
d. 2013
e. 2014

http://www.gdc-
uk.org/Dentalprofessionals/Revalidation/Pages/Revalidation-Q-and-
As.aspx

983. Which one is not a developmental line of teeth.

Von Ebner
Neonatal line
Perikymata

answer is Perikymata.

incremental line of Retzius(development line of enamel) that do not


complete circle but radiate outto the tooth surface form a groove on
tooth surface are called incremantal lines of PICRRILL.
between these 2 grooves is a convexity known as Perikymata.
Neonatal line ..its accentuated inc line of retzius. it demarcates prenatal
enamel from post natalenamel.

von ebner is incremental (developmental )line of dentine.

984. you are examining a patient whom u suspect as a cracked tooth.


which of the following signs & symptoms and diagnostic test help
confirm ur diagnosis

a)pt has pain wen they bite on something


b)pt has pain wen they release their bite
c)tooth is tender to percussion
d)applying orthodontic band to tooth will result in reduction of the pain
e)transillumunation shows that the light travels thru the tooth

Ans: B, D

985. Which antibiotic is used to manage a super infected herpetic lip


lesion.

Which analgesic cause ringing in the ear after overdose.

Antibiotic for a patient allergic to penicillin who can not swallow pills.

ANS:
1. cefuroxime (2nd generation cephalosporins) normally given in super
infections.

2. asprin as it causes tinnitus

3. It depends upon choices but macrolide azithromycin is normally given.


(oral azithromycinsuspension (>/=10 years: 500 mg; <5 years of age:
200 mg; >/=5 to <10 years of age: 300 mg) given 1 h before the
procedure can be used as an alternative.)

986. Which LA is given for patient with Cardiac Arrythmias?

Ans: prilocaine

987. Periodontal probing force is 0.25N. N stands for Newton.


988. JVP is best described as

1) pulse pressure
2 ) 10 mmHg less than ventricular pressure
3) 10 mmHg more than ventricular pressure
4) 20 mmHg less than ventricular pressure
5) 20 mmHg more than ventricular pressure

989.

1- which is more stretchable?


Veins are the most elastic..n hence most stretchable.

2- Which is carrying least of blood?

Veins
Venues
Arteries
Arterioles
Capillaries

990.

1)best radiograph to asses bone level for Implants or xray for Implants in
mandible ?

2) radiograph for caries in 3year old? or for primary molars?

3) xray for calculi in slivary duct?

Answers

1-PA.
2-Bitewing.
3-Lower true occlusal.

991.

1- what is RPI in partial denture?

2- patient has a nice partial denture with a canine(last teeth) pocketing


less than 2mm and 50% bone support, what treatement is the best:
implant based overdenture
extraction and wait healing to make a new denture
extraction and make an immediate denture
RCT if canine treatable and overdenture

Ans;
1. Rest seat ( mesialy located) -proximal plate- I bar clasp
2. Based overdentue

Explnation:

Its an overdenture

I choose it since the ideal indication is when the canine are the only
teeth remaining in the arch. Since you reduce the tooth length to a dome
shape, you decrease the crown and root ratio thus reducing the stresses
on the teeth. It indicated ideally when the canines are the only remaining
teeth in the arch.

According to the conservative dental approach instead of extracting the


teeth , they can be preserved to:

1. Increase retention
2. offer proprioceptive stimulation
3. transmit the forces to the bone aid in the remodeling and decrease the
rate of bone resorption
4. increase the masticatory force

I choose it since its more conservative and the tooth only had a 2 mm
pocket depth, plus they are canines ( corner stone of the arch with long
roots) check pink book overdenture

992.Antibiotic for chronic sinusitis

Its amoxicillin 250mg for 7 days according to scottish dental prescribing


pdf.

We can also prescribe Doxycycline 100mg.


993.
Which drugs are c/I in lactating mothers

Antihistamines, benzoadepines, aspirin, metronidazole, tetracyclines,


caramezipine

994. CVS and exercise

to understand the changes during exercise, we need to know what is


required by the body in such asituation..

Imagine yourself doing a workout and the physical changes will be


apparent:

1. increased heart rate.


2. increased flushing/warmth/sweating.
3. breathlessness

the other changes in the body would be to increase the flow of blood to
the areas where it is needed the most:

the skeletal muscles and to some extent, the skin.

Now, To increase this flow, we need vasodilation but since the total
amount of blood in the body is limited, we also need vasoconstriction in
areas that do not need blood at that time:

viscera/kidney/etc..

Peripheral resistance is a measure of the resistance to the flow of blood


in any vessel. so obviously,vasodilation would decrease.

PR and vasoconstriction would include it. when exercising, the amount


of vasodilation in the working muscles is much greater than the amount
of vasoconstriction in the non working areas leading to a "NET
DECREASE" in TOTAL peripheral resistance.

Now, cardiac output is the amount of blood pumped by the heart each
minute, which should increase to be able to supply the increased
demands of the body during exercise. cardiac output is a function of
heart rate and stroke volume

C.O=H.R * S.V ,
We know that the heart rate increases during exercise. Stroke volume is
simply the amount of blood pumped by the heart in each stroke i.e after
each systole.To increase the stroke volume we need

1. increased volume of blood in the heart to pump out and


2. increased effectiveness of the heart to pump out this blood i.e
increased contraction.To increase the volume of blood in the heart, the
end-diastolic volume should increase, which is a result of an increased
venous return. and an increase in the heart's contraction is caused by
increased sympathetic stimulation to the heart
(beta 1 receptor action) and to some extent by the frank-starling
mechanism. So, we have a NET INCREASE in cardiac output.
All these changes will occur in moderate to extensive exercising,
however exercising beyond a limit can cause a decrease in the cardiac
output because increased heart rate also decreases the amount of time
available for filling, which would decrease the stroke volume and hence
ultimately the cardiacoutput..

995.

Origin of perio ligament?

It's fibroblast from dental follicle.

996. Wht is th concentration of lignocaine in topical anaesthetic?

10% spray and 4% gel for oral use


5% lidocain in topical, EMLA is 2.5 % lidocain and 2.5% prilocain
997.

1) where more blood vessels seen in which part of pulp ?

i) Close to dentine
ii) Centre of pulp chamber
iii) Above furcation area

2) Cells that give rise to the permenant germ?


3) What is the function of lateral dental lamina? why is it formed?
4) Cells which connect enamel organ to the oral epithelium?
5) Best way of comparing milk teeth and permenant?
Ans:
1. IS CENTRAL PART
2. DENTAL LAMINA
3. GIVE RISE TO THE PERMANENT SUCCESSOR
4. DENTAL LAMINA THEN IT DEGENERATE TO GIVE EPTHELIAL
REST OF SEARS LATERAL LAMINA DOES NOT FORM TEETH...

Lateral lamina can be seen in the cap and bell stages ..... when the tooth
bud grows it drags apart of dental lamina along with it.... and it is just an
extension of the dental lamina which connects It to the tooth bud NO
FUNCTIONAL SIGNIFICANCE....

998.

1-WHICH BUR WOULD YOU USE TO CUT ENAMEL DENTINAL


JUNCTION

2-PICTURE OF RED LESION ON BORDER OF TONGUE IN A PT


WHO HAD HEAVY AMALGAM FILLINGS ON THAT SIDE.WHAT IS
THE DIAGNOSIS

(OPTIONS: SCC,TRAUMATIC ULCER ETC)

3-WHICH ONE IS POLISHED THE BEST?MICROFILLED,GLASS


IONOMER,RESINMODIFIED GLASS IONOMER

4-WHAT IS THE MOST USEFUL FILLING FOR MOLAR 5-MIN.TOOTH


REDUCTION IN GOLD CHAMFER

Ans:

1.ROUND
2. TRAUMATIC ULCER
3. MICRIFILLED
4. AMALGAM5. 0.5mm

999.

1. In 3 year-old child, baby teeth to be extracted.Which ragiograph you


select to check permanent germs.

Panoramic,
periapical,

bimolar,

occlusal,

lateral skull view, etc

2. Trauma to tooth with closed apex. In which one the tooth is more
likely to maintain vitality?

Concussion
Subluxation
Intrusive luxation
Avulsion
Luxation

3. What's the most effective method of diet advise for a child.

a.tell the parents not to give sweets for the child


b. dentist tells the child which are good and bad foods
c. nurse talk to the child
d.send the child to a dietician etc

1000.Origin of the lower lip?

fusion of ectomesenchyme of mandibular processes.

1001.Which odontogenic pathology would contain clear straw


coloured fluid? Options are

1. Odontomes
2. Solitary bone cyst
3. Ameloblastoma
4. Dentigerous cyst
5. Keratocyst.

1002. What is the annual dose limit of radiation for non classified
worker?

Ans. 6mSV
1003. Most abundant immunoglobulin in saliva during infections?

According to online resources (non-medical though i.e. Wikipedia), its


IGA, easy way to remember is most of the body excreted fluid are IGA
eg: tears, blood

1004.

1. What would you do when a patient comes with an asymptomatic root


canal treated tooth with periapical radioluscency treated by some other
dentist?

2. Who is responsible when the dentist is sanctioned

3. What's the maximum time the GDC leaves the dentist physically
impaired away from, his profession?

4. Best solution used for hand scrubbing

5. Urgent referrals-time

6. Surgical removal - which consent?

7. A new dental nurse wants to take radiograph

8. A female nurse denied leave

9. enquring about pts oral hygiene care- open or closed questions, some
questions given so youselect one

10. What happens if the dentist does not pay his annual retention fees
on time?

11. Dental nurse that had needle stick injury, can she go back to work.?

Ans:
1. Since its asymptomatic, just inform the patientt, but do not attempt re
RCT. If in future pt has pain, then go for re RCT.

3. 12 months

5. 2 weeks
6. Written consent

10. Erase from register

1005. Which of the following salivary gland tumours is most likely to


occur bilaterally?

a) pleomorphic adenoma
b)mucoepidermoid carcinoma
c)adenoid cystic carcinoma
d)acinic cell carcinoma (Correct answer)

GOOD LUCK WITH YOUR EXAMS

You might also like